Die (vermutliche) Entdeckung des Higgsteilchens hat ja zu einigen Diskussionen geführt – kann man das Higgsteilchen verstehen? Harald Lesch ist der Ansicht, 99,9% der Menschen könnten das nicht, ich habe mich ganz anders geäußert und behauptet, dass jeder es zumindest “ein bisschen” verstehen kann. Die große Diskrepanz zwischen diesen unterschiedlichen Lagern hat vermutlich weniger etwas mit dem Higgsteilchen zu tun und mehr mit der Frage, was man eigentlich meint, wenn man etwas “versteht”. Anhand des Higgsteilchens möchte ich hier unterschiedliche Aspekte oder Stufen des Verstehens auseinanderdröseln.


(Einige Aspekte habe ich vor langer Zeit schon angesprochen, als ich mir Gedanken gemacht habe, was eigentlich Anschaulichkeit ist.)

Vermutlich hat jeder, der nicht gerade in einer Höhle ohne Stromanschluss wohnt, mitbekommen, dass da etwas entdeckt wurde und dass dieses Etwas zumindest für die PhysikerInnen wichtig ist und ihre Theorien bestätigt. In gewisser Weise ist das wohl schon die erste Stufe des Verstehens:

Erkennen, dass da überhaupt etwas zu verstehen ist.

Auch das ist ja schon nicht ganz selbstverständlich – viele Erkenntnisse der Wissenschaft beruhen ja auch darauf, dass überhaupt erst einmal erkannt wurde, dass es da ein Phänomen gibt, das verstanden werden will.

Gut, es gilt also, das Higgsteilchen zu verstehen. Die nächste Stufe besteht jetzt wohl darin zu

Verstehen, welche Rolle es in der Natur (und der physikalischen Theorie) spielt.

In diese Kategorie gehört meine Kurz-Kurz-Erklärung, aber auch die ganzen Analogien mit Prominenten auf Parties oder mit Kugeln, die durch Honig fliegen. Diese Erklärungen sollen die wichtigste Eigenschaft des Higgsfeldes deutlich machen: Das Higgsfeld verleiht Elementarteilchen ihre Masse.1

Das Higgsteilchen ist eine messbare Anregung des Higgsfeldes, es ist sozusagen eine Störung in dem vollkommen gleichartigen Higgsfeld, die man deswegen sehen kann.

1Dabei wird übrigens meist unterschlagen, dass der größte Teil der Masse, die wir beobachten, nicht vom Higgsfeld erzeugt wird (rühmliche Ausnahme ist der ZEIT-Artikel von dieser Woche). Der Großteil der Masse von Atomen steckt im Atomkern und beruht auf der Energie der Bindung der Quarks innerhalb der Protonen und Neutronen; nur ein relativ kleiner Teil der Masse ist “echte” Masse. Das sollte ich vielleicht bei Gelegenheit auch noch mal ausführlich erklären. Die Aussage, dass unsere Welt ohne Higgsfeld vollkommen anders aussähe, bleibt aber richtig – Atomkerne hätten immer noch eine Masse, Elektronen aber nicht. Chemie wie wir sie kennen wäre dann nicht möglich.

Auch wenn diese Erklärung (ohne die Fußnote) schon ein bisschen Nachdenken erfordert, denke ich, dass sie prinzipiell auch für jemanden nachvollziehbar ist, der nicht Physik studiert hat. In diesem Sinne kann dann eben jeder das Higgsteilchen verstehen.

Dass Menschen wie der Interviewpartner von Lesch sagen, sie würden diese Erklärung selbst nicht verstehen, liegt vermutlich an zwei Dingen: Zum einen ist sie (gerade wenn man Anlogien wie Honig oder Parties verwendet) ganz offensichtlich eher metaphorisch, denn das Universum ist ja nicht wirklich von Honig durchzogen (und Party ist auch nicht überall). Zum anderen – und das ist vermutlich noch wichtiger – ist es eine Erklärung, mit der man wenig anfangen kann.

“Anfangen” ist hier in dem Sinne gemeint, dass man diese Erklärung zwar so schlucken kann, wie sie ist, aber man hat nicht genügend Informationen, um weiterdenken zu können. Wenn ich jemanden, der diese Erklärung bekommen hat, bitte, sich eine Eigenschaft der Higgsfeldes zu überlegen (z.B. bei der Honig-Analogie die Frage: Warum werden dann nicht alle Teilchen so abgebremst, dass sie zur Ruhe kommen?), dann merkt derjenige schnell, dass er darauf keine (oder eine offensichtlich falsche) Antwort hat.

Und hinzu kommt, dass auch immer noch nicht klar ist, warum das so sein soll. Warum können die Teilchen nicht einfach selbst eine Masse haben und brauchen dafür erst ein Higgsfeld? Das bringt uns zur nächsten Stufe des Verstehens:

Verstehen, wozu das Higgsteilchen “erfunden” wurde

Bis in die sechziger Jahre hinein hat niemand ein Higgsfeld gebraucht. Teilchen hatten einfach eine Masse, die als Faktor in der entscheidenden Gleichung der Quantentheorie (der Formel für die Lagrangedichte, falls jemand gern coole Fachausdrücke hört) auftauchte und damit konnte man das Verhalten von Elektronen und anderen Teilchen prima beschreiben.

Dann aber merkte man, dass die Radioaktivität die merkwürdige Eigenschaft hat, dass es Prozesse gibt, die nicht mehr ablaufen können, wenn man das beobachtete System einfach spiegelt. Das ist ungefähr so (Vorsicht, hier kommt wieder eine Analogie) als würden es die Gesetze der Mechanik zwar erlauben, normale Uhren zu bauen, aber solche, die gegen den Uhrzeigensinn drehen, wären einfach unmöglich.

1 / 2 / 3 / 4 / Auf einer Seite lesen

Kommentare (210)

  1. #1 rolak
    14. Juli 2012

    Wieder mal ein schöner post – auch wenn die eingepflegten Fußnoten für mich ziemlich gewöhnungsbedürftig¹ sind. Jedoch wohl für Streichelhandys ein unbedingtes Muß.

    Kann man das Higgsteilchen “verstehen”?

    Na klar, wenn es nur deutlich genug redet…

    _____
    ¹ jedoch eigentlich nicht störend, ich muß nur dem Impuls widerstehen, ganz nach unten scrollen zu wollen.

  2. #2 ZeT
    14. Juli 2012

    Es gibt kein “bisschen Verstehen”. Entweder ich hab es verstanden oder nicht. Ich glaube da wurde einfach die falsche Wortwahl benutzt.

    Besser wäre wohl “Kann man das Higgs verständlich erklären?”

    Um Higgs usw wirklich zu verstehen, muss man sich ja erstmal mit dem SM auseinandersetzen. Und da gehts halt schon los. Ich selbst bin ja auch nur Laie – beschäftige mich aber halt schon seit 15 Jahren mit diesen Themen – aber wie soll der normale Verstand den Welle-Teilchendualismus “verstehen”. Ich weis zwar das es so ist, aber verstanden “warum” – das ist halt eine ganz andere Sache.

    Ich glaube auch, das man der Wissenschaft keinen Gefallen tut, hochkomplexe und eigentlich nicht zu verstehende Verhaltensweisen so radikal runterzubrechen, das es auch ein Laie versteht. Denn dann hab ich es so reduziert das da eigentlich kein Inhalt mehr zum vorschein kommt.

    Schlagt die Tageszeitung auf – in 95% der Fälle steht da Zeug drin was so nicht stimmt. Man könnte natürlich argumentieren das es nötig ist, dies alles so zu simplifizieren das es eben auch ein Laie versteht. Nur was bringt es wenn am Ende das geschriebene so von der Wahrheit abweicht das man im Endeffekt nur falsches weis?

    “Das Gottesteilchen verleiht der Materie die Masse”. Ein Header einer Tageszeitung. Das entspricht nun aber nicht der Wahrheit. Und jetzt versuchen sie (also jetzt nicht sie persönlich – als promovierter Physiker hat man allein dadurch einen Glaubwürdigkeitsvorteil) mal jemandem der das gelesen und geglaubt hat zu erklären, das es nicht das Teilchen selbst sondern das Feld ist, und das Higgsteilchen selbst nur der “Überbringer” also ein Austauschteilchen ist…

    Da steht dann die Aussage eines Hobbywissenschaftlers der eigentlich ITler ist gegen irgendeinen Wissenschaftautor in irgendeiner Tageszeitung. Das einzige was dann passiert ist, das mein Gegenüber zwischen zwei Aussagen hängt. Mit Glück schaut er woanders nach um die Aussagen zu überprüfen – mit Pech glaubt er der Tageszeitung und rennt ab dann mit falschen Habwissen durch die Gegend.

  3. #3 MartinB
    14. Juli 2012

    @ZeT
    Sehe ich ganz anders – mit der gleichen Logik könnte man argumentieren, dass Nicht-Wirtschaftswissenschaftler den Euro-Rettungsschirm nicht verstehen können (wer versteht da schon alle Feinheiten und Details?) oder überhaupt die Funktionsweise von Börsen, oder dass jemand, der nicht jeden Zug von Anand nachvollziehen kann, deshalb das Schachspiel nicht versteht oder oder oder.
    Ich denke es ist zwingend, dass verstehen unterschiedliche Ebenen hat und dass man etwas auch “ein bisschen” verstehen kann und habe das ja hier am Beispiel Higgs auch zu begründen versucht.

    “zu erklären, das es nicht das Teilchen selbst sondern das Feld ist, und das Higgsteilchen selbst nur der “Überbringer” also ein Austauschteilchen ist…”
    Vorsicht – das ist erstmal sachlich falsch, das Higgsteilchen ist nicht das Austauschteilchen, sondern eine Feldanregung. Siehe dazu meine Langfassung.
    Aber ja, das versuche ich. Und zumindest einige Leute schreiben mir unter meine Texte, dass sie dadurch mehr verstanden hätten.

    “…verstanden “warum”.. ”
    Das ist vielleicht ein Punkt hier – in der Physik geht es selten bis nie um das “warum”, sondern letztlich immer um das “wie”.
    Den Welle-Teilchen-Dualismus kann man nur als Tatsache akzeptieren, man kann ihn aber trotzdem im hier gegebenen Sinne “verstehen” – nämlich ein funktionsfähiges Modell bauen, mit dem man sich Sachen überlegen kann.

    “Da steht dann die Aussage eines Hobbywissenschaftlers der eigentlich ITler ist gegen irgendeinen Wissenschaftautor in irgendeiner Tageszeitung.”
    Ist mir jetzt nicht klar auf wen sich das bezieht – wer ist denn der IT-Hobbywissenschaftler?

    @rolak
    An Handys habe ich gar nicht gedacht, ich habe den Trick mit eingestreuten Fußnoten von TetZoo übernommen…

  4. #4 Ralph Ulrich
    14. Juli 2012

    Sehr schöne anschauliche Beschreibung des physikalischen Verstehens!
    Sogar mit der Konsequenzen, wann ist etwas eigentlich gar nicht verstanden!

    Da ich kein Physiker bin, versuche ich immer ein paar Schritte zu überspringen, was prinzipiell natürlich nicht geht 🙁

    Ich hatte im voherigen Blogbeitrag die Frage gestellt, nach dem Verhältnis
    Masse durch starke Atombindungsenergie -zu- Higgsfeld-generierte-Masse

    Was wäre eigentlich, wenn wir die beteiligten Teile der starken Bindungsenergie einfach definieren würden als ein-einziges-neues Teilchen:
    Könnte man vielleicht dadurch den Massemechanismus auf einen einzigen Higgs Mechanismus reduzieren?
    Nur so ne Frage, wäre schöner …

  5. #5 Ralph Ulrich
    14. Juli 2012

    Ähh, vergessen:
    Zwei verschiedene Masse-Mechanismen haben nicht nur die Schwierigkeit ihr genau gleiches Verhältnis zur Gravitation zu erklären, sondern natürlich auch ihr genau gleiches Verhältnis zur Trägheit!

  6. #6 Dr. Webbaer
    14. Juli 2012

    Die (vermutliche) Entdeckung des Higgsteilchens hat ja zu einigen Diskussionen geführt – kann man das Higgsteilchen verstehen?

    Man kann jedenfalls die Theorie vom Higgs-“Teilchen” verstehen, denn sonst gäbe es dieses dementsprechend nicht.

    Streng genommen kann man ohnehin nur die “Theorie von etwas” verstehen oder besser: Die Theorie ist das Verstehen.

    Kleiner Widerspruch vielleicht noch, die Theorienbildung beinhaltet jeweils optional das Beschreiben, Erklären und die mögliche Prognose; ein Verstehen im Sinne einer Theoretisierung muss nicht immer ‘operativ’ und die ‘Vorhersage’ betreffend ausfallen, auch wenn man in der Physiklehre oft nachvollziehbarerweise derart bemüht ist.

    MFG
    Dr. Webbaer (der dank der Bemühungen des Inhaltemeisters sogar das Higgs-Boson ein wenig verstanden hat)

  7. #7 ZeT
    14. Juli 2012

    @Martin

    thx für die Antwort 🙂

    Bezüglich des “Austauschteilchens”. Ich gebe zu, es ist sachlich falsch. Aber wie um alles in der Welt will man einem Laien eine “Feldanregung” so erklären, das er am Ende nicht mit einem Knoten im Gehirn von dannen zieht. Dazu muss ja erstmal der Begriff “Feld” erklärt werdern…(Kornfeld, Vektorfeld, Fussballfeld…)

    Und genau das Problem meinte ich. Sprich: “das Higgs verständlich erklären” oder eben “das Higgs zu verstehen”. Ist in der IT nicht anders (womit auch die Frage nach dem ITler beantwortet wurde). Ich kann verständlich erklären, wieso sich die Software so verhält wie sie es tut – aber ohne Grundwissen meines Gegenübers wird es ihm eigentlich fast nicht möglich sein, das verhalten der Software zu verstehen.

    Mein aktueller Job ist in einem Labor was Lebensmittel auf GVO untersucht. Da ich aber kein Chemiker bin, stösst mein Verständnis irgendwann an Grenzen. Ich weis zwar wie der Ablauf funktioniert, ebenso ist mir bewusst welche chemischen Prozesse dort ablaufen – aber “verstanden” wieso sich das alles so verhält wie es sich verhält… das ist ein Problem. Das führt dann immer dazu, das ich mir passende Bücher organisiere um mich in diesen Bereichen fit zu machen und bei Unklarheiten lass ich mich dann von meinem Laborchef aufklären.

    Ergo “quäle” ich mich durch Fachliteratur, um überhaupt eine Basis zu haben um meinen Laborchef zu verstehen.

    Das hier im Sienceblog die Leute mit euren Erklärungsversuchen zurecht kommen, liegt imho daran, das sich hier auch eher Leute tummeln die sich für dies alles zumindest Ansatzweise interessieren (man weis was ein SM ist, mann hat sich mit diesem Bereich der Wissenschaft zumindest ansatzweise auseinandergesetzt).

    Ich erlebe oft Situationen, wo man Grundwissen voraussetzt. Es mag banal klingen, aber selbst der Aufbau eines Atoms ist vielen nicht geläufig. “Atomkern und da kreist was rum”. Da wird dann aus einem Neutron schnell ein Neutrino, das Proton wird zum Photon usw. usw. da kommt dann auch schnell die Frage auf, was Elementarteilchen eigentlich sind.

    Greetz

  8. #8 Dr. Webbaer
    14. Juli 2012

    Ich kann verständlich erklären, wieso sich die Software so verhält wie sie es tut – aber ohne Grundwissen meines Gegenübers wird es ihm eigentlich fast nicht möglich sein, das verhalten der Software zu verstehen.

    Wegen der Schnittstellen zur Physik (Natur) ähnelt das Verständnis von Software durchaus dem Verständnis der Physiklehre. Binäres Denken macht auch dort nicht happy. – Man kann auch in den Anwendungen der IT teilweise verstehen, eigentlich verstehen alle nur teilweise.

  9. #9 Enkrod
    14. Juli 2012

    Saucooler Artikel, auch wenn ich als Laie nicht behaupten will ihn wirklich zu verstehen.

    Was mich aber zum selben Thema noch interessiert ist: Was kommt jetzt?

    Mit dem (beinahe) sicheren Nachweis des Higg-Boson wurde eine Lücke im Standardmodell geschlossen. Aber das Standardmodell erklärt nur 4% der Masse/Energie des Universums, dunkle Materie macht aber ca. 23% aus und dunkle Energie ca. 73% (Was auch immer diese Begriffe letztlich umfassen). (Quelle: Michio Kaku auf Youtube)

    Gibt es testbare Voraussagen von Theorien ausserhalb des Standard-Models (z.B. die String-Theorie) nach denen die Wissenschaftler des LHC jetzt ausschau halten könnten? Kann der LHC überhaupt die Energien erreichen die vermutlich nötig sind um diese unbekannten “Dunklen Einflüsse” zu beobachten?
    Ist es überhaupt möglich diese Einflüsse zu beobachten die ja eben “dunkel” benannt wurden weil sie nur über ihre Gravitation bzw. die beschleunigte Ausdehnung des Universums indirekt beobachtbar sind?

    Oder kurz gesagt: Was ist der nächste Schritt auf der Suche nach einer “Theorie von allem”?

  10. #10 Dr. Webbaer
    14. Juli 2012

    Was ist der nächste Schritt auf der Suche nach einer “Theorie von allem”?

    Gibt es schon: die Tegmark-Hyopthese. – Ansonsten: Nö, der Physiklehrer oder dementsprechende Forscher kann sich nie sicher sein. Ischt ein wenig so wie mit dem allmächtigen Gott, den es nicht geben kann.

  11. #11 Belzer
    14. Juli 2012

    Ich denke die Frage mit dem Verstehen ist schon etwas komplizierter. Ich bin zwar Laie, aber ich bin davon überzeugt, dass das sogenannte “Verstehen” der Physiker auch kein richtiges “Verstehen” ist. Wie einer der Kommentatoren schon erwähnte ist der Welle-Teilchen-Mechanismus von den Physiker bislang auch nicht richtig verstanden worden. Alle Beteuerungen von Physikern, dass dies kein Problem sei und dass man die jeweilige Theorie nur richtig anwenden müsse, gehen am Kern des Problems vorbei, dass sich beide Erklärungsversuche logisch gegenseitig ausschließen. Man kann zwar so tuen, als ob dies kein größeres Problem sei, was bis zu einem gewissen Grad auch gerechtfertigt ist. Aber man sollte nicht behaupten, dass man das ganze wirklich verstanden hätte. Ich finde das gehört zu Ehrlichkeit dazu. Es ist ein Unterschied, ob man mit mathematischen Gleichungen umgehen kann oder ob man einen Sachverhalt wirklich verstanden hat.

  12. #12 MartinB
    14. Juli 2012

    @Ralph Ulrich
    “Zwei verschiedene Masse-Mechanismen haben nicht nur die Schwierigkeit ihr genau gleiches Verhältnis zur Gravitation zu erklären, sondern natürlich auch ihr genau gleiches Verhältnis zur Trägheit!”
    Nein. Masse ist Energie, und träge Masse ist schwere Masse. Das sind nur drei Begriffe für ein und dieselbe Sache. Das Higgsteilchen ist verantwortlich nur für die Ruhemasse von Teilchen, aber nicht für alle Arten von Energie etc.

    @ZeT
    “Aber wie um alles in der Welt will man einem Laien eine “Feldanregung” so erklären, das er am Ende nicht mit einem Knoten im Gehirn von dannen zieht.”
    Das hängt davon ab, wieviel Aufwand der Laie zu betreiben bereit ist. Wenn er ein paar Stunden oder Tage investieren mag, kann er meine QFT-Serie lesen. Wenn ihm das zu viel ist, dann kann er sich mit der Analogie der Schallwelle begnügen (Eine Feldanregung ist eben eine Störung in einem gleichmäßigen Medium).

    “liegt imho daran, das sich hier auch eher Leute tummeln die sich für dies alles zumindest Ansatzweise interessieren”
    Aber mit schreiben auch Leute Kommentare, die eben sagen, sie hätten vorher nie verstanden, was das Higgsteilchen ist und jetzt hätten sie zumindest eine Idee. Das passiert hier ab und an unter meinen Texten. Wenn die Leute hinterher also mehr wissen und nachvollziehen können als vorher, dann finde ich es sinnvoll, das als eine Zunahme des “Verstehens” zu beschreiben.

    @Enkrod
    “Gibt es testbare Voraussagen von Theorien ausserhalb des Standard-Models (z.B. die String-Theorie) nach denen die Wissenschaftler des LHC jetzt ausschau halten könnten?”
    Oh ja. Zum Beispiel sollten sich in den meisten Modellen mit Supersymmetrie (also auch der Stringtheorie) messbare Effekte zeigen, die sind bisher ausgeblieben. Etwas älterer Artikel dazu hier, viel hat sich seitdem nicht geändert:
    https://www.scienceblogs.de/hier-wohnen-drachen/2011/03/wird-es-eng-fur-susy.php

    “Oder kurz gesagt: Was ist der nächste Schritt auf der Suche nach einer “Theorie von allem”?”
    Fleissig am CERN Daten produzieren und hoffen, dass irgendwas nicht ins Bild des Standardmodells passt. Klingt ein bisschen dünn, ist aber im Moment einfach die beste Möglichkeit. Es gibt sehr viele Ideen, wie eine bessere Theorie aussehen kann, was die dunkle Materie sein könnte usw. Um da etwas zu unterscheiden, braucht man jetzt wirklich wieder mal Daten aus dem Experiment.

    @Belzer
    “Wie einer der Kommentatoren schon erwähnte ist der Welle-Teilchen-Mechanismus von den Physiker bislang auch nicht richtig verstanden worden. ”
    Das kommt darauf an, wie du “verstehen” definierst. Ich kann in den meisten Situationen überlegen, was auf Grund der Quantenmechanik passieren wird, oft sogar ohne großartig rechnen zu müssen (in Sachen Schrödingergleichung habe ich ein Verständnis, das dem oben beschriebenen von Dirac einigermaßen nahekommt). Mit anderen Worten, ich habe eine gute Intuition dafür entwickelt. Was würdest du noch verlangen, damit ich es “verstehe”? Dass ich es auf meine klassische Alltagsanschauung zurückführen kann? Das ist leider unmöglich.

    “Es ist ein Unterschied, ob man mit mathematischen Gleichungen umgehen kann oder ob man einen Sachverhalt wirklich verstanden hat.”
    Stimmt, das habe ich ja oben geschrieben. Das Dirac-artige Verständnis ist ja mehr als bloß die Gleichungen lösen zu können. Wenn es noch eine höhere Form des Verstehens geben soll, dann wüsste ich gern, wie die aussieht. (Siehe auch den verlinkten Text zur Anschaulichkeit.)

  13. #13 Statistiker
    14. Juli 2012

    Oh, der Braunbär ist wieder da…..

    Na, Bärchen, lang genug ausgetobt auf wissenslog, ud jetzt wieder hier rumbräseln?????

  14. #14 MartinB
    14. Juli 2012

    @Statistiker
    Solche Kommentare sind eher überflüssig. Wenn du den Wb nicht so toll findest (so wie ich auch), dann bitte einfach ignorieren (ich lese seine Texte nicht).

    Hinweis an alle: Das ist hier keine Aufforderung, jetzt eine Meta-diskussion anzufangen. Jeden diesbezüglichen Kommentar werde ich löschen. Ihr findet sicher irgendwo ein Forum oder einen offenen Thread, wo ich euch nach Herzenslust über alle möglichen Webtierchen auslassen könnt.

  15. #15 Stefan
    14. Juli 2012

    Es sehr viele Leute, die von Wissenschaft fasziniert sind, und hier spielen Blogs wie von MartinB oder FlorianF eine wichtige Vermittlerrolle, ich selbst habe hier schon einiges gelernt.

    Eine Detailfrage habe ich aber noch:
    Ich habe den Eindruck, dass manche (auch Physiker) das Higgs-Feld und das Teilchen äquivalent verwenden. Meistens liest man, dass es nicht das Teilchen, sondern das Feld ist, welches den Fermionen und Bosonen die Masse verleiht und das Teilchen als Schwingungszustand ist nur der Beweis, dass es das Feld wirklich gibt. Manchmal kann man aber lesen, dass die Higgs-Bosonen doch Wechselwirkungen mit den Elementarteilchen eingehen. Ist das vielleicht eine andere Art von WW oder ein Missverständnis?

  16. #16 Rarehero
    14. Juli 2012

    Als verständnisloser Laie drängt sich mir die Frage auf, warum unterschiedliche Teilchen unterschiedlich mit dem Higgs-Feld wechselwirken. Wenn ich die Abbildungen sehe, dann frage ich mich auch, warum die Teilchen, bildlich gesprochen, an einem der mit “m” markierten Stellen nicht nicht(!) mit dem Higgs-Feld wechselwirken sollten und was passiert, wenn sie es nicht tun. Haben sie dann keine Masse?

  17. #17 SYNTaX
    15. Juli 2012

    wenn ich das richtig verstanden habe könnte das daran liegen, dass man die Bewegen eines Teilchens von A nach B nur als die Summe aller möglichkeiten, also 0 – n wechselwirkungen mit dem Higgs-Feld beschreiben kann?!
    Mhhh, mir fällt gerade auf, dass in meinem Verständnis dafür, die Möglichkeit für den “direkten Weg” immernoch existiert. Die Konsequenz daraus wäre dass alle Teilchen mit einer sehr geringen Wahrscheinlichkeit die Lichtgeschwindigkeit behalten können.
    Also gehe ich davon aus, dass es ein Gesetz gibt, was die Wechselwirkung mit dem Higgs-Feld erzwingt? Oder sorgen dann andere Kräfte dafür, dass die Teilchen gebremst werden?

  18. #18 Joseph Kuhn
    15. Juli 2012

    @ ZeT:

    “(…) wie soll der normale Verstand den Welle-Teilchendualismus ‘verstehen'”.

    Das geht vielleicht viel einfacher als beim Higgsteilchen, weil es, wie mir scheint, mehr mit unseren Alltagsbegriffen zu tun hat als mit Formeln wie der Lagrangedichte. Lesenswert dazu ist das erste Kapitel in dem Buch “Von der Materie” von Jean-Marc Lévy-Leblond. Dort erklärt Lévy-Leblond, wie es dazu kam, Phänomene anhand bestimmter Eigenschaften der Zählbarkeit und der Räumlichkeit als “Teilchen” bzw. als “Welle” zu verstehen, wie es sich damit auf der Quantenebene verhält und warum er deshalb die Redeweise vom “Welle-Teilchen-Dualismus” für unangemessen hält. Das kann man auch als Nichtphysiker “verstehen”.

  19. #19 MartinB
    15. Juli 2012

    @Stefan
    “Ich habe den Eindruck, dass manche (auch Physiker) das Higgs-Feld und das Teilchen äquivalent verwenden. ”
    Das ist eine Begriffsschwierigkeit. Auch Elektronen beschreibt man ja eigentlich mit einem Elektronenfeld, verwendet aber den Begriff Elektron und Elektronfeld meist synonym.
    Beim Higgsfeld wird die Sache jetzt dadurch kompliziert, dass das Higgsfeld eigentlich vier teilchen beschreibt, die man alle als Higgsteilchen beschreiben könnte. Nur eins davon ist das “echte” Higgsteilchen, das man beobachten kann. Die anderen drei sind diejenigen, die letztlich mit dafür sorgen, dass die W- und Z-Bosonen eine Masse haben (Ausführliche Erklärung dazu in meinem langen Higgsfeldartikel.).

    @rarehero
    “warum unterschiedliche Teilchen unterschiedlich mit dem Higgs-Feld wechselwirken.”
    “Warum” im Sinne eines Modells, das die unterschiedliche WeWi erklärt – dafür gibt noch es keine Antwort. Momentan muss man das in die Theorie reinstecken, damit sie zu den Beobachtungen passt.

    “was passiert, wenn sie es nicht tun. Haben sie dann keine Masse? ”
    Hat SYNTaX schon gut beantwortet: Man muss alle Möglichkeiten berücksichtigen. Das bedeutet auch, dass es eine sehr kleine Wahrscheinlichkeit dafür gibt, dass sich z.B. ein Elektron mit Lichtgeschwindigkeit bewegt – die ist aber so klein, dass sie praktisch keine Rolle spielt. (Tatsächlich tragen sogar Prozesse zur Gesamtwahrscheinlichkeit bei, bei denen die Teilchen mit Überlichtgeschwindigkeit fliegen – das führt aber nie zu beobachtbaren Konsequenzen im Sinne einer “echten” Signalübertragung mit Überlichtgeschwindigkeit. )

  20. #20 rolak
    15. Juli 2012

    Das Buch klingt interessant, kenne ich allerdings nicht. Noch nicht, schönen Dank für den Tip, Joseph.
    Die Beschreibung erinnert mich allerdings an einen Stoßseufzer nach der soundsovielten Frage der Art “Quanten: Mit Welle und Teilchen wie ist das denn jetzt? ‘und’, ‘oder’, ‘entweder oder’, ‘abwechselnd’…?”:

    ‘Weder noch’, Quant. Nur ein beim Bewerten der Messung in gewohnte Kategorien sortierendes Denken erzeugt den Scheinwiderspruch.

  21. #21 Joseph Kuhn
    15. Juli 2012

    @ rolak: Das Buch von Lévy-Leblond ist jedenfalls deutlich leichtere Kost als der Sammelband “Die Philosophie der Physik” von Michael Esfeld, auf den C.W. vor einiger Zeit netterweise hinwies. Dort steht zwar in der Einleitung, die Beiträge seien “ohne Spezialkenntnisse in Philosophie und Physik verständlich”, darüber, was “Spezialkenntnisse” in der Physik sind, können die Meinungen aber offensichtlich etwas auseinandergehen.

  22. #22 Roland Tluk
    15. Juli 2012

    @Martin Bäker
    Schöner Artikel.

    Schade nur, dass die meisten Medien stumpf die dpa-Meldung kopiert und eingefügt haben. Verstanden haben die meisten Journalisten das nicht.

    Auf der anderen Seite ist das Higgs-Teilchen nicht bewiesen. Dennoch ist die einfache Beschreibung des Problemfelds gelungen.

  23. #23 Dr. Webbaer
    15. Juli 2012

    Dort erklärt Lévy-Leblond, wie es dazu kam, Phänomene anhand bestimmter Eigenschaften der Zählbarkeit und der Räumlichkeit als “Teilchen” bzw. als “Welle” zu verstehen, wie es sich damit auf der Quantenebene verhält und warum er deshalb die Redeweise vom “Welle-Teilchen-Dualismus” für unangemessen hält.

    Die Menschen pflegen andauernd mehrere sich oft auch widersprechende Sichten (“Theorien”) auf eine Sache oder einen Sachverhalt. Eine Besonderheit im Sinne eines speziellen Dualismus kann der Schreiber hier nicht erkennen.

    Was aber der o.g. “Dualismus” zeigt, ist dass es in der Modernen Wissenschaft nicht mehr um Wahrheit oder Verifizierung geht. Die Wissenschaft schafft kein Wissen, ‘Scientia’ ist sprachlich klarer.

    MFG
    Dr. Webbaer

  24. #24 rolak
    15. Juli 2012

    Danke für die beruhigende Aufmunterung, Joseph, als das von Dir angesprochene andere Werk geliefert wurde, d.h. nachdem ich es auf dem Weg ‘in die Stadt’ vom Statiönchen abgeholt hatte, habe ich als hoffentlich einen Eindruck ergebende Unterwegslektüre Kap8, Brigitte Falkenburg, ‘Was sind subatomare Teilchen?’ gewählt. Da machten mir eher der imho überstrukturierte Text, die gräßlich vielfältigen Akronyme und mangelndes Faktenwissen auf dem Gebiet der Philosophie zu schaffen.
    Liegt allerdings noch nicht oben auf dem (in sich nicht unbedingt statisch sortierten) ToRead-Stapel der Fachliteratur…

    können die Meinungen … auseinandergehen

    Nun ja, die Meinung über den semantischen span von ‘Spezialkenntnisse’ dürfte standortabhängig sein. Ist allerding ein Kommunikationsproblem eher läßlicher Art, bis zum Realitäts-Test ein falsches Bedeutungsbild beim Hörer aufbauend. Im Gegensatz zu Themen, bei denen Meinungsträger wie die bekannten Königskinder durch tiefe Gräben getrennt zu sein scheinen.

    Doch zurück zu den nichtgebackenen Teichen: Quantenmechanische Effekte gibt es der Folklore nach auch in der IT – seit heute morgen triezt mich ein lästiger Heisenbug 😉

  25. #25 Jürgen K.
    15. Juli 2012

    Hallo Martin,
    Vielen Dank dafür, dass Du uns die Physik ein wenig näher bringst. Sie ist so komplex, dass man oft gar keinen Einstieg findet. Das kann für den interessierten Laien sehr frustrierend sein. Ich kann zwar nicht behaupten, dass ich verstanden habe, was es jetzt genau mit dem Higgs-Teilchen auf sich hat, aber ich glaube verstanden zu haben, was “euch” Physiker zu der Theorie bewegt. Das ist doch schon mal was.

    Also noch mal Danke für Deinen Blog.

  26. #26 haarigertroll
    15. Juli 2012

    Nur mal so interessehalber, ob ich das halbwegs richtig verstanden habe: Das Higgs-Feld ist dafür verantwortlich, dass Teilchen eine (Ruhe-)Masse, also auch Trägheit haben.
    Wenn es jetzt eines fernen Tages gelänge, das Higgs-Feld künstlich zu beeinflussen, (bisschen Sci-Fi hier) könnte man damit dann so Sachen wie beispielsweise Trägheitsdämpfer für Raumschiffe realisieren um bislang für unmöglich gehaltene Beschleunigungen zu realisieren?
    Bzw. kann man das Higgs-Feld überhaupt so wie das elektrische Feld beeinflussen? Kann hierbei das Higgs-Teilchen analog zum elektrischen Ladungsträger Elektron eine Rolle spielen?

  27. #27 Joseph Kuhn
    15. Juli 2012

    @ rolak: Die vielen Abkürzungen im Text von Brigitte Falkenburg fand ich auch nicht sehr lesefreundlich, aber ich meine eher Texte mit Passagen wie dieser:

    “Zur Angabe der globalen Struktur benötigt man eine Konnektion. Sie ist definiert als Differentialform mit Werten in der Lie-Algebra der Strukturgruppe. Da man sich die Lie-Algebra als isomorph zum Tangentialraum der Lie-Gruppe denken kann, gibt die Konnektion eine Regel, wie man den Tangentialraum des Prinzipalfaserbündels in einen ‘horizontalen’ Anteil (isomorph zum Tangentialraum des Basisraums) und einen ‘vertikalen” Anteil zerlegen kann”

    Vielleicht braucht man zum Verständnis solcher Passagen keine “Spezialkenntnisse in Philosophie und Physik”, aber die Wissensseite im Feuilleton liest sich dann doch etwas einfacher.

  28. #28 MartinB
    15. Juli 2012

    @haarigertroll
    Nette Idee – ein Higgsreduktor als Trägheitsdämpfungsfeld für Raumschiff Enterprise.
    Problem ist zweierlei: Erstens würde es nicht viel nützen, weil (siehe die Fußnote) der Großteil der atomaren Masse aus den Kernbausteinen in der Bindungsenergie steckt. Und zweitens würden unsere Atome ihre Struktur massiv ändern, wenn man die Elektronenmasse reduziert, das wäre vermutlich nicht sehr gesund. Man könnte aber sicher einen fiesen Disruptor als Waffe bauen.

    @Joseph
    Nö, Spezialkenntnisse in Physik braucht man da nicht, es reicht vermutlich eine Master-Mathe-Vorlesung “Gruppentheorie”….

  29. #29 Dr. Webbaer
    15. Juli 2012

    Falls es jemanden interessiert:

    Brigitte Falkenburg, ‘Was sind subatomare Teilchen?’ gewählt. Da machten mir eher der imho überstrukturierte Text, die gräßlich vielfältigen Akronyme und mangelndes Faktenwissen auf dem Gebiet der Philosophie zu schaffen.

    Vor dieser Dame kann Dr. W nur warnen, die fehlende Einfachheit als Schutzfunktion verstehend…

    MFG
    Dr. Webbaer

  30. #30 Niels
    15. Juli 2012

    @MartinB
    Ohne eine zusätzliche Vorlesung in Differentialgeometrie hätte man dann aber immer noch Schwierigkeiten…

    Ich hab ein bisschen ein Problem mit deinen beiden letzten Stufen des Verstehens.
    Da würde ich keine Rangordnung aufstellen, das sind für mich zwei verschiedene Dinge.
    Ich kenne Physiker, die nach vierzig Jahren Beschäftigung mit einer Theorie intuitiv vorhersagen können, ob ein bestimmter Ansatz klappt oder nicht.
    Ich glaube aber nicht, dass sie deswegen die Theorie auch selbst herleiten könnten, wenn sie die Herleitung vorher noch nie gesehen hätten.

    Das selbst anhand der Grundidee entwickeln halte ich für sehr viel schwieriger.
    Trotzdem folgt auch daraus nicht, dass man die Theorie deswegen gleich intuitiv versteht.

    Das sind für mich wie gesagt zwei paar Schuhe, ohne dass eins davon ein “tieferes” Verständnis beweist als das andere und ohne dass das eine dem anderen unbedingt vorausgehen muss.

  31. #31 rolak
    15. Juli 2012

    es reicht vermutlich

    @ MartinB @ Joseph @ me: 🙂 Das dürfte einen der von mir zu entdeckenden Haken trefflich beschrieben haben. Doch solange ich wenigstens halbwegs den Überblick zu behalten glaube, also nicht andauernd das gefühlte Gesamtverständnis wegbricht, weil zuviel Nochnichtverstandenes auf die lange Bank geschoben wurde, so lange lese ich gerne weiter. Dank einer gewissen Zähigkeit dürfte bis zum ‘es reicht’ vermutlich einige Zeit vergehen.

  32. #32 Frank Ludwig
    15. Juli 2012

    Endlich mal ein wenig mehr über die netten kleinen Higgs Teilchen, das man auch verstehen kann. Ja ich schreibe verstehen, weil ich sowas auch verstehe, auch wenn ich sicherlich nicht zu den 0,01% der Menschheit gehören mag.

    Super Erklärung, und sehr nette Analogieen.

    Gruß Frank The Tank
    von Frankies Testwelt
    https://www.frankiestests.blogspot.com

  33. #33 Joseph Kuhn
    15. Juli 2012

    @ MartinB: … ist hier mehr Physik drin?

    “Für die relativistische Quantentheorie spielt die Poincaré-Gruppe eine wichtige Rolle, da ihre irreduziblen unitären Darstellungen zur Klassifizierung der Elementarteilchen dienen. Die physikalische Annahme dahinter ist, dass der Zustandsraum eines elementaren Objekts keine relativistischen invarianten Unterräume besitzen soll (…). Nun lassen sich die Darstellungen der Poincaré-Gruppe durch die Eigenwerte ihrer Casismir-Operationen charakterisieren. Dem liegt zugrunde, dass die Aussage, ein physikalisches System besitze eine gewisse Symmetrie, äquivalent ist zu der Aussage, dass der Hamiltonoperator mit allen Operatoren der Symmetriegruppe kommutiert, was klar ist, da (…)”

    Ich muss gestehen, mir ist das nicht klar. Wie beim Higgs-Teilchen kann man sich jetzt fragen, wie viel davon muss man als Nichtphysiker verstehen, um überhaupt etwas von relativistischer Quantentheorie zu verstehen und was lässt sich davon für Nichtphysiker verständlich machen?

  34. #34 MartinB
    15. Juli 2012

    @Niels
    Ja, interessante Frage. Aber die Physiker, die du kennst, können zwar vielleicht die Theorie nicht selbst herleiten, sie können aber doch vermutlich innerhalb der Theorie Dinge herleiten oder haben das mal getan – wie sonst kämen sie zu ihrer Erfahrung. Insofern bin ich mir nicht sicher, ob das dann nicht zwei etwas unterschiedliche Dinge sind, um die es geht – einmal das Herleiten der Theorie, einmal das Herleiten von Lösungen mit Hilfe der Theorie.
    Auf der anderen Seite kann es einem aber auch passieren, dass man intuitiv weiß, was die Lösung für ein problem ist, und dann eine ganze Weile nachdenken muss um das auch für andere nachvollziehbar zu begründen. Insofern hast du vielleicht trotzdem recht.

  35. #35 MartinB
    15. Juli 2012

    @Joseph
    Ja, das ist eine Sicht der Dinge.
    Letztlich sagt das aber nur folgendes aus: Da Quantenfelder (wie zum Beispiel das Elektronfeld) sich für unterschiedliche Beobachter (im Sinne der RElativitätstheorie, also solche, die sich mit unterschiedlichen Geschwindigkeiten bewegen oder deren Bezugssysteme zueinander verdreht sind) sinnvoll ineinander umrechnen lassen müssen (es kann nicht sein, dass das, was für mich wie ein Elektron aussieht, für dich ein Quark oder ein Photon ist, nr weil du ein anderes Koordinatensystem verwendest), müssen diese Quantenfelder gewisse Bedingungen erfüllen.
    Daraus lassen sich ziemlich klare Bedingungen ableiten, die alle Quantenfelder erfüllen müssen – insbesondere folgt daraus, dass Quantenfelder (und damit alle Teilchen) einen eindeutigen Spinwert haben müssen.

    Am besten fleißig meine Quantenfeldtheorieserie lesen (oben unter Artikelserien).

    Der ganze mathematische Ballast dient dazu, das wasserdicht zu machen und zu zeigen, wie man damit was beweisen kann, aber die Grundidee ist simpel.

    Ein bisschen kann man das mit der Physik und der Mathematik vielleicht mit der Juristerei vergleichen – Gesetzestexte sind ja auch in ihrer ganz eigenen Sprache abgefasst und ohne Jurastudium nicht wirklich in allen Details zu erfassen, trotzdem kann man auch als Laie eine Idee bekommen, was in den Gesetzen drinsteht. Ist bei der Mathematik sicher extremer, aber schon ein bisschen ähnlich.

  36. #36 Stefan
    15. Juli 2012

    @MartinB

    Danke für die Antwort, jetzt ist mir das Ganze wieder ein bißchen klarer.

    OT:
    Die österreichische Tageszeitung “Die Presse” hat heute einen Artikel über Wissenschaftsblogs veröffentlicht. Dabei wurde gerade dieser Blog lobend erwähnt.

    https://diepresse.com/home/techscience/internet/1265083/Was-bringt-das-Web-20-der-Wissenschaft?_vl_backlink=/home/techscience/index.do

  37. #37 Wurgl
    15. Juli 2012

    Die Frage die sich nun aufdrängt ist: Kann man dieses Higgsfeld irgendwie beeinflussen und lässt sich daraus eine Waffe bauen.

    Warum Waffe? Ganz einfach weil aus allem was der Mensch so erfindet erstmal eine Waffe gebaut wird. Danach kommt eine zivile Anwendung.

  38. #38 Stefan
    15. Juli 2012

    Neue Entdeckungen treiben die Innovation an. Dieser Prozess scheint schwer aufzuhalten, wenn man bedenkt, wie sehr Neugierde und Forscherdrang in der menschlichen Natur verankert sind. Vielleicht werden uns unsere Innovationen irgendwann umbringen, das hat aber nicht mit den neuen Erkenntnissen von CERN im Speziellen zu tun.

    Besseres Verständnis von Verbrennungsprozessen und der Thermodynamik sind einher gegangen mit der Verbreitung von Verbrennungsmotoren, hohe Ölpreise und bessere Technik machen die Förderung von unkonventionellen fossilen Rohstoffen rentabel. Wir brauchen keine Waffen um unsere Zivilisation zu bedrohen.

  39. #39 adenosine
    16. Juli 2012

    So eine Wechselwirkung kann man sich ja noch plausibel machen. Aber was ist der Trick, dass die Higgswechselwirkung nur bei Beschleunigung von Teilchen wirkt und nicht bei konstanter Geschwindigkeit?

  40. #40 MartinB
    16. Juli 2012

    @Stefan
    Boah, ich werd berühmt…

    @Wurgl
    Siehe mein Kommentar oben an haarigertroll

    @adenosine
    Nein, die Wechselwirkung wirkt immer, aber bei konstanter Geschwindigkeit ändert sie eben nichts an der Geschwindigkeit. Der MAssenterm macht sich aber auch da bemerkbar, denn er beeinflusst den Zusammenhang zwischen Energie E und Impuls p. Für ein Teilchen ohne Masse ist
    E=p c
    Für ein teilchen mit Masse ist
    E^2=p^2 c^2 + m^2 c^4
    Auch da wirkt also der Higgsmechanismus.
    Details dazu, wie der Masseterm wirkt, gibt’s so in den Teilen 5-8 meiner QFT-Serie.

  41. #41 Irfy
    16. Juli 2012

    Ich hätte eine ganz extreme Frage.

    Bisher ist ja die Quantengravitation noch nicht gelöst. Hat es schon Überlegungen gegeben, dass das Higgs-Feld quasi nebenbei auch noch die Gravitation erzeugt?
    Oder ist das ausgeschlossen?

    Ist mir beim Durchlesen des Artikels so in den Sinn gekommen und ich kann mir nicht vorstellen, dass da nicht auch Experten darüber zumindest nachgedacht haben.

  42. #42 MartinB
    16. Juli 2012

    @lrfy
    Nein, das ist ausgeschlossen. Das Higgsfeld ist ein konstantes Hintergrundfeld. Seine Anregungen sind die sehr massiven Higgsteilchen. Das Teilchen der Gravitation müsste aber masselos sein. Außerdem hat das Higgsfeld Spin 0, das Graviton muss Spin 2 haben.

    Nicht ausgeschlossen ist allerdings meines Wissens, dass es irgendeinen Zusammenhang zwischen Higgsfeld und dunkler Energie gibt – bisher hat da allerdings niemand eine sinnvolle Theorie erstellen können.

  43. #43 Irfy
    16. Juli 2012

    Danke,

    dachte mir das die Idee keine neue ist und es einfache Antworten gibt

  44. #44 Name auf Verlangen entfernt
    16. Juli 2012

    Ich interesssiere mich besonders dafür, was damit gemeint ist:

    “Das Higgsfeld intuitiv verstehen”

    “Wie stark ist das Higgsfeld?”

    Das sind alles Fragen, die man nur mit Hilfe der mathematischen Formulierung der Theorie beantworten kann?

    Nun denn: wohlan! Hier die ultimative Troll-Frage an den Herrn Professor (die er ja selbst gestellt hat: Wie stark ist denn genau das Higgs-Feld?

    Intuition wird offensichtlich in der Hirarchie des Verstehenes weit über die reine Mathematik gestellt – das macht Rudolf Steiner auch so.

  45. #45 Erik der Unlesbare
    17. Juli 2012

    -> Kommentar MartinB: 14.07. 12 17:47 an @Belzer
    “Wenn es noch eine höhere Form des Verstehens geben soll, dann wüsste ich gern, wie die aussieht.”
    Warum wollen sie eine höhere Form des Verstehens kennenlernen? Wollen sie nach dem Kennenlernen diese vielleicht anwenden?
    Ich kann verstehen warum sie diesen Blog schreiben. Mich unterstützen sie mit ihren wissenschaftlichen Beiträgen.
    Mich verstehen können sie nicht. Die Eulersche Identität in die Physik von Raum und Zeit können sie nicht einordnen. Sie ist es, welche den Welle-Teilchen-Dualismus und die Quantelung der Elementarteilchen beschreiben und erklären kann.

    Ich traue mich nicht weiter zu schreiben, weil sie von meinen Texten nur 5 Zeilen lesen können und dann eine Elementarkraft in ihrem Körper sie weiter treibt, ohne mich verstehen zu können. Aber dennoch. . ..
    Ich lade sie ein zu einem Gedankenexperiment: Lassen sie uns beide unsere Gedanken zum Welle-Teilchen-Dualismus und zur Quantelung der Elementarteilchen in ihrem Wissenschaftsblog darstellen. Ich sage voraus: Wenn wir mit der Quantelung der Elementarteilchen beginnen, werden wir mit der Erläuterung des Welle-Teilchen-Dualismus enden. Beginnen wir mit dem Welle-Teilchen-Dualismus, werden wir mit der Quantelung der Elementarteilchen enden.

    Ich werde dabei lernen mein intuitives Wissen so darzustellen, das alle es verstehen können.
    Sie haben geschrieben, sie wollen eine höhere Form des Verstehens kennen lernen? 

    Ich möchte ein wenig vorweg greifen: die Lösung des obigen “Rätsels” liegt in der Beschreibung der Quantelung von Zeit. Sie verbindet beide Phänomene. Nein, sie verbindet diese nicht, sondern durchdringt beide Phänomene.

  46. #46 MisterX
    17. Juli 2012

    @Erik der Unlesbare: GTFO

  47. #47 MartinB
    17. Juli 2012

    @MT
    Ausnahmsweise mal eine Antwort: Die Stärke des Higgsfeldes beträgt 246GeV. Dass Sie mit dieser Antwort nichts anfangen können, ist Ihr Problem.

    @Erik
    Warum du hier allerdings immer wieder kommentierst, wenn du inzwischen doch merken solltest, dass niemand hier auf deine texte antwortet (was meiner Vermutung nach daran liegt, dass die meisten Leute – wie ich auch – sie nicht lesen), erschließt sich mir auch nicht.

    @MisterX
    Leute schmeißt hier nur einer vom Blog, das bin ich 😉

  48. #48 SCHWAR_A
    17. Juli 2012

    @MartinB:
    “Die Stärke des Higgsfeldes beträgt 246GeV”

    Müßte dieser Vakuumenergie-Erwartungs-Wert denn nicht auch die Energie der Störung im Higgsfeld, also die Energie des Higgs-Teilchens sein?

    Herzliche Grüße.

  49. #49 Erik der Unlesbare
    17. Juli 2012

    @MartinB
    Ich entwickle die Weltformel, oder mit anderen Worten: Ich beantworte mir die Fragen, die sie oben gestellt haben. Stück für Stück.
    Dabei helfen sie mir! Dank dafür.
    Sie verstehen nur nicht mit welchen Methoden ich das mache. Darüber haben sie ja oben auch geschrieben.

  50. #50 snyder
    17. Juli 2012

    Am Rande:

    Wer sein Verständnis auf englisch erweitern kann/möchte (80 min., “easy understandable”) hat dazu Gelegenheit zB. durch dieses Video einer ganz nett besetzten Veranstaltung der UC Berkeley:

    Professors Beate Heinemann, an experimental physicist and a member of the ATLAS experiment at the LHC in CERN, Switzerland, and Lawrence Hall, a theoretical physicist and former Director of the Berkeley Center for Theoretical Physics, explained what the Higgs is, why it was predicted and how it was proven to exist. They were joined by panel members Professor Marjorie Shapiro, also a member of the Atlas experiment, Miller Fellow Josh Ruderman and PhD student and ATLAS member Louise Skinnari.

  51. #51 Vastehnix
    17. Juli 2012

    Ehrlich, ich bin bemüht etwas zu verstehen. Dank meiner offensichtlich beschränkten intellektuellen Fähigkeiten gelingt mir das jedoch leider nicht. Ich warte also so lange ab, bis das “Higgs-Teilchen” im schon sehr gut gefüllten Mülleimer der physikalischen Theorien verschwindet.
    Da mein bescheidenes Wissen eher ästhetischer Natur ist, kann ich von diesem Gesichtspunkt her für mich sagen: ästhetisch ist das gesamte Quantentheoriezeugs nicht. Es erinnert eher an eine Dach auf 3 Beinen, von denen keines vernünftig verankert ist und die in einer äußerst wagemutigen Konstruktion so ausgesteift und miteinander verbunden sind, dass das Dach zu halten scheint. Spätestens beim Betreten des Bereiches unter dem Dach stürzt die Konstruktion jedoch in sich zusammen.
    Unangenehm ist mir, dass auf der einen Seite von Theorie gesprochen wird, jedoch andererseits immer brav das Verb sein in seinen verschiedenen Konjugationen verwendet wird.
    Ach ja, und dann immer die vermaledeite Graviation – stets am Rande erwähnt.

  52. #52 MartinB
    17. Juli 2012

    @SCHWAR_A
    “Müßte dieser Vakuumenergie-Erwartungs-Wert denn nicht auch die Energie der Störung im Higgsfeld, also die Energie des Higgs-Teilchens sein?”
    Nein, warum sollte das so sein? Schau dir das Sombrero-Bild an, das eine ist ein Maß dafür, wie tief der Sombrero unterhalb der Nullinie ist, die Masse des Higgsteilchens ist ein Maß für die Krümmung in der Krempe.

    @Vastehnix
    Du warst der, der ein Troll werden wollte, oder? Sehr schwacher Versuch, sorry.

  53. #53 SCHWAR_A
    17. Juli 2012

    @MartinB:
    “Schau dir das Sombrero-Bild an, das eine [der VEW] ist ein Maß dafür, wie tief der Sombrero unterhalb der Nullinie ist, die Masse des Higgsteilchens ist ein Maß für die Krümmung in der Krempe.”

    AAAh! Das hatte mir gefehlt! Danke.

    Herzliche Grüße.

  54. #54 Dr. Webbaer
    17. Juli 2012

    Ich entwickle die Weltformel, oder mit anderen Worten: Ich beantworte mir die Fragen, die sie oben gestellt haben. Stück für Stück.
    Dabei helfen sie mir! Dank dafür.
    Sie verstehen nur nicht mit welchen Methoden ich das mache. Darüber haben sie ja oben auch geschrieben.

    Offensichtlich mit persönlichen Methoden. Hat ja niemand etwas dagegen, aber muss man das Verlautbaren [1] bevor die Weltformel von Ihnen bereitgestellt worden ist?

    Unangenehm ist mir, dass auf der einen Seite von Theorie gesprochen wird, jedoch andererseits immer brav das Verb sein in seinen verschiedenen Konjugationen verwendet wird.

    Theorien sind Sichten und erlauben bspw. den Generellen Präsens und die Behauptungsform. Meinen Sie, dass Theoretiker mehr das Sollen referenzieren, äh, sollen?

    MFG
    Dr. Webbaer

    [1] ‘Warum du hier allerdings immer wieder kommentierst, wenn du inzwischen doch merken solltest, dass niemand hier auf deine texte antwortet (…)’ – FYI: das schreibt Herr Bäker nur, weil er nicht alle Kommentare liest.

  55. #55 MartinB
    17. Juli 2012

    @SCHWAR_A
    Sorry, ich muss mich korrigieren – der Vakuumerwartungswert ist natürlich der Radius der Sombrero-Krempe; die Tiefe ist die Energiedichte.

  56. #56 SCHWAR_A
    17. Juli 2012

    @MartinB:
    …??…
    wie paßt das denn so zu den Teilchen Higgs, Top, etc.?
    Wie muß ich mir denn zB. Top im Sombrero vorstellen?

    Herzliche Grüße.

  57. #57 roel
    17. Juli 2012

    @MartinB Erstmal, super Artikel! Habe bisher nur mitgelesen, möchte jetzt aber eine kurze ot-Bemerkung plazieren.

    “Sorry, ich muss mich korrigieren”: Das ist von sciencebloggern eher selten zu lesen und zwar nicht, weil es nie etwas zu korrigieren gäbe*. Das macht diesen Blog so lesenswert**. Wenn mal etwas nicht 100% stimmt und es Dir oder einem der versierten Kommentatoren auffällt, wird es korrigiert.

    *Das soll jetzt nicht heißen, dass andere scienceblogger permanent etwas zu korrigieren hätten.

    **natürlich nicht nur das, sondern auch die interessanten und gut erklärten Beiträge.

  58. #58 MartinB
    17. Juli 2012

    @Schwar_a
    Verstehe nix. Das Top-Quark hat mit dem Sombreropotential doch nichts am Hut (pun intended).

  59. #59 Name auf Verlangen entfernt
    17. Juli 2012

    @ MartinB: Nein, das haben Sie missverstanden: daß Sie aus Teilchenkollisionen meinen auf´s Feld Rückschlüsse ziehen zu können und vice versa ist bekannt – daß diesen Feldern ein Energiewert – z.B. 246GeV – zugeordnet wird (gemäß der Kraft, die nötig ist, einer Teilchen-Wirkung des Feldes habhaft zu werden) auch (wobei natürlich die Frage nach der Masse/Energieäquivalenz bei masselosen Teilchen, die selbst jedoch Masse verleihen sollen, unbeantwortet bleibt) – mich würde vielmehr interessieren: da behauptet wird, das Universum expandiere – und andererseits nun ebenfalls behauptet wird, das Higgs-Feld sei Äther-gleich und überall vorhanden – wie schafft es das Universum nur, bei beschleunigter Ausdehnung den Energiewert des Feldes konstant zu halten? Wird das Higgs-Feld mit beschleunigt? Und was passiert, wenn es sich beständig ausdehnt und beschleunigt? Kann es dann einen konstanten Wert haben? Schließlich sind ja – Homogenität und Isotropie – Ränder und Mittelpunkt des Universums überall, nicht?

    Fernerhin: ist das Higgs-Feld überall, so ist es offenbar mit dem, was wir uns unter Raum vorstellten, bevor Einstein eine vierte Dimension aus dem Hut zauberte, identisch? Ist daher der (Einstein-freie) Raum – jener alte Newton-Kontainer – das Higgs-Feld selbst – oder wie unterscheidet es sich davon?

  60. #60 Joseph Kuhn
    17. Juli 2012

    @ MartinB:

    “Ein bisschen kann man das mit der Physik und der Mathematik vielleicht mit der Juristerei vergleichen – Gesetzestexte sind ja auch in ihrer ganz eigenen Sprache abgefasst und ohne Jurastudium nicht wirklich in allen Details zu erfassen, trotzdem kann man auch als Laie eine Idee bekommen, was in den Gesetzen drinsteht.”

    Das sehe ich auch so. Zu dem Esfeld-Buch, aus dem ich zitiert habe, muss man gerechterweise dazu sagen, dass dort in der Einleitung nicht nur steht, dass man ohne Spezialkenntnisse der Physik auskommt, der Satz geht weiter: “(…) dies ist jedoch kein Einführungsbuch. Eine gewisse Vertrautheit mit den Themen, die man durch das Studium einschlägiger Einführungen erwerben kann, wird vorausgesetzt.” Also vielleicht eher was für Dich als für mich.

  61. #61 Niels
    17. Juli 2012

    @SCHWAR_A

    Wie muß ich mir denn zB. Top im Sombrero vorstellen?

    Gar nicht.
    Der Sombrero stellt das Higgs-Potential dar. Dieses hängt von zwei Parametern ab.
    Wenn man einen Querschnitt des Sombreros betrachtet (also nur mit reellen und nicht mit komplexen Zahlen arbeitet), sieht das für das Higgs-Potential dann so wie hier auf dem rechten Bild aus:
    https://www-zeus.physik.uni-bonn.de/~brock/feynman/vtp_ws0506/chapter09/higgs_pot.jpg
    Das ist eine bestimmte Parabel vierter Ordnung, lässt sich also allgemein folgendermaßen darstellen:
    V(x) = -A*x^2 + B*x^4
    Wir müssen nur noch statt x das Higgs-Feld Φ einsetzen. (Üblicherweise arbeitet man mit den Parametern μ^2 und λ statt A und B, das ist aber nur eine Konvention und ich spar uns das mal.)
    Den Φ-Beitragwert für die zwei Minima erhält man durch eine einfache Kurvendiskussion. (Erste Ableitung Nullsetzen usw.)

    Dieser Φ-Wert ist der Vakuumerwartungswert (VEV) des Higgs-Feldes. Er hängt natürlich nur von den Parametern A und B ab. Wenn man diese Parameter kennt, kann man also den VEV berechnen.
    Wenn man umgekehrt allerdings nur den VEV kennt, wie das vor der Entdeckung des Higgs-Teilchens war, kennt man nur das Verhältnis der beiden Parameter. Man weiß nicht, wie das Potential genau aussieht.

    Die Masse des Higgs wiederum hängt nur von B und dem VEV ab, wobei der VEV ja wie gesagt nur von A und B abhängig ist. Die Form des Higgs-Potentials bestimmt die Masse des Higgs-Teilchens also vollständig.
    Wenn man Higgs-Masse und VEV kennt, kennt man deswegen auch die genaue Form des Higgs-Potentials und umgekehrt.

    Das top-Quark und die anderen Elementarteilchen (ausgenommen nur die Bosonen der elektroschwachen WW) merken dagegen überhaupt nicht von diesem Potential.
    Für sie ist nur wichtig, ob der VEV ungleich Null ist oder nicht.
    Wenn der VEV ungleich Null ist, koppeln sie dem Standardmodell zufolge mit einer jedem Teilchen eigenen unterschiedlichen Kopplungskonstante an diesen Wert. Daraus ergibt sich dann ihre Masse.
    Welche Form das Higgspotential hat, wie steil es ist, usw. ist dabei völlig unerheblich.

    @Name auf Verlangen entfernt
    Eigentlich interessiert sie das Thema doch gar nicht, oder?
    Sonst hätten sich auch mal die Texte über den Kommentaren gelesen und würden nicht behaupten, dass

    die Frage nach der Masse/Energieäquivalenz bei masselosen Teilchen, die selbst jedoch Masse verleihen sollen, unbeantwortet bleibt

    Wie auch immer, vielleicht interessiert es ja jemand anderen.

    wie schafft es das Universum nur, bei beschleunigter Ausdehnung den Energiewert des Feldes konstant zu halten?

    Das schafft es ganz genau so, wie es das auch bei der Vakuum-Energiedichte aller anderen Felder schafft.
    https://de.wikipedia.org/wiki/Vakuumenergie
    Das bisschen, was noch durch das Higgs-Feld dazu kommt, spielt eigentlich keine größere Rolle.
    So eine Energiedichte mit einem konstanten Wert beobachten wir ja auch, wenn wir die Expansion des Universums untersuchen. Das ist die berühmte dunkle Energie.
    Allerdings ist der beobachtete Wert der dunklen Energie ungefähr 110 Größenordnungen kleiner als die theoretischen Vorhersagen der Quantenfeldtheorien.
    Wir wissen bisher noch nicht, wie sich dieses Problem lösen lässt.

    Reicht das, um auch in Zukunft auch die verrückten, verblendeten Physiker schimpfen zu können?
    Das Ganze wird in Anspielung auf die Ultraviolett-Katastrophe bei der Schwarzkörperstrahlung auch Vakuum-Katastrophe genannt.
    Das ist ein kostenloser Servive, da Sie beim Trollen doch so gerne mit kaum verstandenen Buzzwörtern um sich werfen.

    Zum Einstein-freien Raum kann ich nicht viel beitragen, weil ich keine Ahnung habe, was genau Sie wobei falsch verstanden haben oder absichtlich falsch verstehen wollen.
    Das Higgs-Feld ist jedenfalls ein Skalarfeld und damit natürlich lorentzinvariant. Deswegen muss man sich auch keine Sorgen um Einstein machen.
    Noch ein Tipp: Behaupten Sie alternativ lieber, die kosmische Hintergrundstrahlung sei ganz eindeutig der neue Äther.

  62. #62 Bell
    18. Juli 2012

    @MartinB:

    Nette Idee – ein Higgsreduktor als Trägheitsdämpfungsfeld für Raumschiff Enterprise.
    Problem ist zweierlei: Erstens würde es nicht viel nützen, weil (siehe die Fußnote) der Großteil der atomaren Masse aus den Kernbausteinen in der Bindungsenergie steckt. .

    Das irritiert mich jetzt etwas.
    Ist ein Photon träge? Ist es nicht die Ruhemasse ausschließlich, die zur Trägheit führt?

    ===========================================

    Vorsicht – das ist erstmal sachlich falsch, das Higgsteilchen ist nicht das Austauschteilchen, sondern eine Feldanregung. Siehe dazu meine Langfassung

    Kommt das Higgsteilchen als Teilchen denn überhaupt in der Natur vor?

    Oder ist es mehr ein Phänomen …. wie etwa der Sprite über manchen Gewitterblitzen, der ja auch detektierbar ist?

  63. #63 MartinB
    18. Juli 2012

    @Niels
    Danke, dass du dir die Mühe machst, auf MT zu antworten.

    @Bell
    Ja, Photonen sind träge. Weil die Verwirrung hier jetzt öfters aufgetaucht ist (wieso verleiht das Higgsfeld Masse, hat aber nicht direkt was mit Gravitation zu tun etc.?) werde ich demnächst mal was über die unterschiedlichen Aspekte des Massebegriffs schreiben.

    “Kommt das Higgsteilchen als Teilchen denn überhaupt in der Natur vor? ”
    Zunächst mal gilt in der E-Teilchentheorie: Wenn ich ein Teilchen erzeugen kann, dann muss es ein zugehöriges Feld geben, von dem dieses Teilchen die Anregung ist. Wenn ich also ein Higgsteilchen finde, dann muss es ein Higgsfeld geben.
    Ansonsten kann man aber davon ausgehen, dass z.B. durch Kollisionen von Teilchen der kosmischen Strahlung auch Higgsteilchen entstehen, denn diese Strahlung hat eine sehr hohe Energie.

  64. #64 Geislwind
    18. Juli 2012

    Musste mich in den letzten 2 Jahren relativ intensiv mit Elektrotechnik auseinandersetzen und meine Erkenntnis dabei war:
    Gewisse Sachen kann man nicht “verstehen”, man GEWÖHNT sich nur an sie.;-)

  65. #65 MartinB
    18. Juli 2012

    @Geislwind
    Ja, das hat uns unser Quantenmechanik-Prof auch erzählt…
    Ich glaube aber eher, man ändert seine Idee, was es bedeutet, etwas zu verstehen.

  66. #66 Name auf Verlangen entfernt
    18. Juli 2012

    @ Niels: Ihre Unverschämtheiten beiseite gelassen, reicht interessierten Laien Ihr Rumfuchteln mit physikalischen Definitionen nicht (erklären Sie in Zukunft in verständlichen Worten, die Barrikade aus fachchinesischer Schwurbelei sparen Sie sich; wir Bürger könnten uns sonst genötigt fühlen, mal das Budget für physikalische Grundlagenforschung zu überprüfen … ).

    Es kann ja nicht von “allen Feldern” die Rede sein – das Gravitationsfeld durchdringt das Vakuum völlig mühelos und wird in seiner “Wechselwirkung” mit dem Vakuum – weil das Standartmodell dafür keinen Plan hat – ausgeklammert.

    “So eine Energiedichte mit einem konstanten Wert beobachten wir ja auch, wenn wir die Expansion des Universums untersuchen. Das ist die berühmte dunkle Energie.”

    Was Ihnen bei der Ontologisierung metrischer Prozesse entgangen ist, die Sie Relativitätstheorie nennen, ist die Realität. Für die Konstanz von c wurde t variabel gedacht. Selbstredend gibt es dafür nicht den geringsten Anhaltspunkt in der Wirklichkeit, und wir haben das gerade an diesem Ort bis zum Abwinken durchdiskutiert und jedem Laien ist bekannt, daß Ihr Kaiser – die SRT – mindestens kosmologisch gesehen nackt ist: i.e.: falsch. Den virtuellen Lendenschurz nennen Sie “Dunkle Materie” und “Dunkle Energie”.

    Und natürlich wird die Frage nach der Masse/Energieäquivalenz bei ansonsten (angeblich) masselosen Teilchen weder in oberem Text noch sonst irgendwo im Gebäude der theoretischen Physik überhaupt ansatzweise zufriedenstellend berührt.

    Das hier ist zwar auch reiner Scientismus, aber wenigstens etwas ehrlicher:

    https://www.zeit.de/wissen/2012-07/higgs-teilchen-cern-kommentar

    Das mag auch daran liegen, daß es falsch ist, das Higgs-Feld (so es das gibt) als lorentzinvariant zu bezeichnen. Woher wollen Sie denn das wissen? Man zweifelt, ob das Higgs überhaupt “entdeckt” sei – Sie aber wollen schon Eigenschaften des Higgs-Feld kennen?

  67. #67 Manea-K
    18. Juli 2012

    Für die Konstanz von c wurde t variabel gedacht. Selbstredend gibt es dafür nicht den geringsten Anhaltspunkt in der Wirklichkeit(…)

    Wenn es nicht so traurig waere…

    Falls es jemanden interessiert: https://prl.aps.org/abstract/PRL/v91/i19/e190403

    https://www.nature.com/nphys/journal/v3/n12/full/nphys778.html

  68. #68 Name auf Verlangen entfernt
    18. Juli 2012

    @ Manea-K: Ja, traurig. Mal nicht das Myonen oder GPS-Märchen? Oder der Flugzeug-Bluff? Nicht ein “Beweis”, der entweden hinkt oder – Merkur-Perihel – locker anders interpretierbar ist. Ihr müßt euer Denken ändern, behauptet zumindest die Kosmologie, anstatt die Schmierenkomödie mit der “Dunklen Materie” für hohe Wissenschaft zu verkaufen.

  69. #69 Manea-K
    18. Juli 2012

    Ach so? Dann interpretieren Sie doch mal das nature-paper “locker anders”. Ich bin gespannt… 🙂

  70. #70 SCHWAR_A
    18. Juli 2012

    @Niels:
    Vielen Dank für die Super-Erklärung!

    Ich habe mit der “Sombrero”-Potentialgleichung (4) ein wenig herumgerechnet und hätte da doch noch die eine oder andere Frage:

    1. Das Minimum ergibt sich bei
      Φ² = m²·2/λ.
    Ist dieses Minimum eigentlich gleichzusetzen mit
      Φ² = m_Higgs²,
    also mit der quadrierten Higgs-Masse?

    2. Was bedeutet dann der Parameter m² bzw. µ² bei Dir? Ist das die quadrierte Planck-Masse?

    3. Wie kommt man von Masse^4 auf Potential? Was wird da an h, c, 2π etc. alles weggelassen, bzw. was muß ich wieder wie ergänzen?

    4. Es wird irgendwo gesagt, λ sei die “Selbstkopplung” im Higgsfeld. Müßte dieses λ nicht seeeehr groß sein?

    Vielen Dank im Voraus,
    Herzliche Grüße.

  71. #71 SCHWAR_A
    18. Juli 2012

    @Niels:
    in Frage 1. meinte ich nicht die Higgs-Masse, sondern die dem VEV entsprechende Masse, also
      Φ² = (VEV/c²)².

  72. #72 Name auf Verlangen entfernt
    18. Juli 2012

    @ Manea-K: Das mach ich jetzt mal nicht; bisher waren die faulen Tricks angeblicher Beweise durchschaubar – und mir ist nicht einsichtig (Stichwort Neutrinomessungen), was man sich da zusammengelogen hat. Inhaltlich zeigt Ihnen gern Frank Wappler – warum es nicht möglich ist, die SRT so zu fasifizieren oder verifizieren. Mir als Laien reicht es auch völlig, Lee Smolin (2006) zu lesen, der ganz frei die Möglichkeit einräumt: ” … daß nämlich das Relativitätsprinzip der Bewegung falsch sei, was bedeutet, dass wir zwischen absoluter Bewegung und absoluter Ruhe unterscheiden können.” (Zukunft der Physik, S. 308)

    Warum die chronogeometrische Distanzdefinition – der einzig mögliche Maßstab – die Distanz nicht findet, sondern vorab metrisch-unverrückbar definiert – und damit jeder Dilatation oder Kontraktion von Länge und Zeit widerspricht, kann ich herleiten, ist hier schon vorgeführt: aber das machen wir jetzt nicht, das wäre ja albern.

  73. #73 Manea-K
    18. Juli 2012

    Hihi.
    Halten wir also fest: Es gibt mindestens ein Experiment, das nicht “locker anders interpretierbar” ist.
    Fuer den Rest ist mir meine Zeit zu schade. Ich geh dann mal wieder Ihre Steuergelder verplempern.

  74. #74 Niels
    18. Juli 2012

    @SCHWAR_A
    Schöner Fund.

    Ist dieses Minimum eigentlich gleichzusetzen mit
    Φ² = m_Higgs²

    Laut deinem Link gilt, dass der (Vacuum expectation value) VEV= v² = -2*m²/λ ist.
    Für die Masse des Higgs-Bosons gilt
    mHiggs = Wurzel[λ]*v,
    also ergibt sich duch Einsetzen für die Masse des Higgs:
    mHiggs²= -2*m²

    Das Minimum ergibt sich bei
    Φ² = m²·2/λ.[…]
    in Frage 1. meinte ich nicht die Higgs-Masse, sondern die dem VEV entsprechende Masse, also
    Φ² = (VEV/c²)²

    Ich habs nicht nachgerechnet, aber im Link steht, die Minima liegen bei
    Φ² = v²/2 = -m²/λ.
    Das steht bei mir oben anders, sorry.

    Was bedeutet dann der Parameter m² bzw. µ² bei Dir?

    m and λ are the mass and self-interaction coupling constant of the scalar field.

    […]
    So long as m²>0 , this model is just a self-interacting scalar field, whose quanta are particles and antiparticles of mass m .
    […]
    Things are different, however, if m² is smaler 0
    […]
    Now canonical quantization can proceed as normal in terms of the fields φ1,2 . The model describes two kinds of particles, the φ1 quanta of mass λ√v and the massless φ2 quanta, with cubic and quartic couplings. The φ2 quanta, corresponding to a spatial variation of the phase angle, are the Nambu-Goldstone bosons
    Den Rest dazwischen sollte man natürlich auch lesen.
    Was besseres fällt mir dazu auch nicht ein.
    Mit der quadrierten Planck-Masse hat m überhaupt nichts zu tun.
    Ich kannte das Potential nur mit dem Parameter µ, aber welchen Buchstaben man da reinschreibt ist ja völlig egal.

    Wie kommt man von Masse^4 auf Potential? Was wird da an h, c, 2π etc. alles weggelassen, bzw. was muß ich wieder wie ergänzen?

    Ich weiß gerade nicht, wie du auf Masse^4 kommst. Aber es gilt wie immer in der theoretischen Physik c = hquer = 1.

    Es wird irgendwo gesagt, λ sei die “Selbstkopplung” im Higgsfeld.

    Das sieht man z.B. an mHiggs=Wurzel[λ]*v .

    Müßte dieses λ nicht seeeehr groß sein?

    Im Vergleich wozu?
    mHiggs und der Vakuumerwartungswert v sind doch bekannt. Den Betrag von λ kannst du also ausrechnen.

  75. #75 SCHWAR_A
    18. Juli 2012

    @Niels:
    Frage 2 und 4 haben sich gerade aufgelöst:
      m_H = µ √2.
    Dadurch ist die Kopplung λ in der Nähe von 1…

    Herzliche Grüße.

  76. #76 SCHWAR_A
    18. Juli 2012

    @Niels:
    …und Frage 1 auch:
    Das Minimum ist ja gerade der VEV…

  77. #77 Niels
    18. Juli 2012

    @MartinB
    Danke, dass du dir die Mühe machst, auf MT zu antworten.

    War das Sarkasmus? Ich fand die Frage ganz gut, deswegen die Antwort.
    Dadurch haben wir jetzt leider wieder ein paar Tage lang den üblichen Salat.
    Ignorieren ist eben doch die bessere Idee.

  78. #78 MartinB
    18. Juli 2012

    @Niels
    Nein, nicht so richtig sarkastisch.
    Ich lese MT nicht mehr wirklich (außer wenn’s kurz ist), sondern überfliege allenfalls. Aber für neue Leser mag das komisch aussehen, falls da tatsächlich mal ne realistische Frage steht.

  79. #79 Name auf Verlangen entfernt
    18. Juli 2012

    @ Niels: mal abgesehen davon, daß Sie auf meine Fragen überhaupt nicht geantwortet haben – was Sie offenbar nicht merken in ihrem Formel-Delirium: anworten Sie rasch noch auf diese Frage, und wenn Sie sie beantworten können (aber verständlich), dann sind Sie mich Troll sofort los:

    Warum glauben Sie, das Higgs-Feld sei ein Skalar-Feld und Lorentz-invariant? Irgendwelche Anhaltspunkte oder Beweise?

  80. #80 Niels
    19. Juli 2012

    @SCHWAR_A
    Hast du übersehen, dass ich dir schon geantwortet hatte?

    @Name auf Verlangen entfernt

    Das hier ist zwar auch reiner Scientismus, aber wenigstens etwas ehrlicher:
    https://www.zeit.de/wissen/2012-07/higgs-teilchen-cern-kommentar

    Ich bin mir ziemlich sicher, dass Sie nicht verstanden haben, worum es in diesem Artikel eigentlich geht.
    Vielleicht würde sich nochmaliges Lesen lohnen?

    mal abgesehen davon, daß Sie auf meine Fragen überhaupt nicht geantwortet haben

    Auf “Wenn der Mond aus Käse ist, warum ist die Erde dann nicht aus Wurst” kann man nun mal ziemlich schlecht antworten.
    Wenn man außerdem weiß, dass Erklärungen den Fragesteller überhaupt nicht interessieren, weil dieser niemals in Betracht ziehen wird, dass der Mond möglicherweise doch nicht aus Käse besteht…

    1. wobei natürlich die Frage nach der Masse/Energieäquivalenz bei masselosen Teilchen, die selbst jedoch Masse verleihen sollen, unbeantwortet bleibt

      Welches masselose Teilchen verleiht denn Ihrer Meinung nach welchen anderen Teilchen Masse? Und wo wurde das von MartinB behauptet? Zitate, bitte.

    2. wie schafft es das Universum nur, bei beschleunigter Ausdehnung den Energiewert des Feldes konstant zu halten?

      Darauf hab ich geantwortet.

    3. Wird das Higgs-Feld mit beschleunigt?

      Das Higgs-Feld wird natürlich nicht beschleunigt. Der Witz ist vielmehr, dass die Energiedichte des Higgs-Feldes trotzt der Expansion konstant bleibt.

      Genauso wie es sich auch bei den Vakuum-Energiedichten aufgrund aller anderern Quantenfelder verhält.
      Das sind nach heutigem Verständnis einfach intrinsische Eigenschaften des Raumes.

      Ihre Frage macht allerdings deutlich, dass Sie offenbar leider nicht verstanden haben, was es überhaupt bedeutet, wenn man davon spricht, dass das Universums (beschleunigt) expandiert.

    4. Schließlich sind ja – Homogenität und Isotropie – Ränder und Mittelpunkt des Universums überall, nicht?

      Homogenität und Isotropie bedeuten nicht, dass “Ränder” und “Mittelpunkt” “überall” sind.
      Schauen Sie sich doch bitte einmal an, was diese Begriffe wirklich bedeuten.

    5. ist das Higgs-Feld überall, so ist es offenbar mit dem, was wir uns unter Raum vorstellten, bevor Einstein eine vierte Dimension aus dem Hut zauberte, identisch?

      Nein, da besteht keine Identität.
      Eigentlich sollte es selbstverständlich sein, dass ein Feld nicht mit einem Raum identisch sein kann.

    6. Ist daher der (Einstein-freie) Raum – jener alte Newton-Kontainer – das Higgs-Feld selbst – oder wie unterscheidet es sich davon?

      Jetzt könnte ich Ihnen einfach die Definitionen von “Raum” und von “Feld” verlinken.
      Wenn etwas in der Mathematik unterschiedliche Namen hat, ist es extrem wahrscheinlich, dass es dann auch zwei unterschiedliche Dinge sind.
      Der absolute Raum Newtons hat mit dem Higgs-Feld keinerlei Gemeinsamkeiten.
      Dei Frage nach dem Unterschied ist ein bisschen sinnlos.
      Was unterscheidet einen Apfel vom EU-Rettungsschirm?

      Außerdem ist es unmöglich, mit Hilfe des Higgs-Feldes ein absolutes Bezugssystem zu definieren. Darauf habe ich allerdings schon erklärt.

    7. Es kann ja nicht von “allen Feldern” die Rede sein – das Gravitationsfeld durchdringt das Vakuum völlig mühelos

      Es gibt gar kein “Gravitationsfeld” so wie Sie sich das vorstellen. Die Gravitation wird momentan am Besten mit Hilfe der allgemeinen Relativitätstheorie beschrieben. Der Natur und der Physik ist es dabei egal, dass es Menschen gibt, die diese Tatsache anscheinend niemals mit ihren speziellen Weltbildern in Einklang bringen können.

      Welche Felder ich meine, hätten Sie herausfinden können, wenn sie dem Wikipedia-Link zur Vakuumenergie gefolgt wären.
      Das dort und in meinem Text der Begriff Quantenfeldtheorie häufig auftaucht, hätte sogar bei flüchtigem Überfliegen einen Hinweis liefern müssen.

    8. jedem Laien ist bekannt, daß Ihr Kaiser – die SRT – mindestens kosmologisch gesehen nackt ist: i.e.: falsch. Den virtuellen Lendenschurz nennen Sie “Dunkle Materie” und “Dunkle Energie”.

      Hier wäre es z.B. sehr nützlich, wenn Sie den Unterschied zwischen SRT und ART kennen würden. (Na ja, oder wenn Sie wenigstens verstanden hätten, was die SRT überhaupt aussagt.)
      Es wissen nämlich sogar sehr viele Laien, dass diese Dinge überhaupt nichts mit der SRT zu tun haben.

    .

    anworten Sie rasch noch auf diese Frage, und wenn Sie sie beantworten können (aber verständlich), dann sind Sie mich Troll sofort los:
    Warum glauben Sie, das Higgs-Feld sei ein Skalar-Feld und Lorentz-invariant? Irgendwelche Anhaltspunkte oder Beweise?

    Das Higgs-Feld ordnet jedem Punkt des Raumes eine reelle Zahl (einen Skalar) zu.
    (Wenn Sie wissen wollen, wie man darauf kommt, würde ich vorschlagen, dass Sie die Artikel, unter die Sie ihre Kommentare posten, auch wirklich lesen.)
    Ein Skalar-Feld ist definiert als eine Funktion, die jedem Punkt eines Raumes eine reelle Zahl (einen Skalar) zuordnet.
    Deswegen ist das Higgs-Feld ein Skalar-Feld.

    Eine Physikalische Größe, die sich bei einer Lorentz-Transformation nicht ändert, heißt lorentzinvariant.
    Skalare ändern sich unter Lorentz-Transformationen nicht.
    Das Higgs-Feld ist wie erwähnt ein Skalar-Feld.
    Deswegen ist das Higgs-Feld lorentzinvariant.

    (Wenn Sie sich jetzt beschweren, dass ich bei meiner Antwort auf Ihre Frage, warum das Higgs-Feld die per Definition vorgegebenen Forderungen für ein Skalar-Feld und für Lorentzinvarianz erfüllt, mit physikalischen Definitionen rumfuchtele, dann qualifizieren Sie sich erfolgreich für den Großen Clownsorden mit Eichenlaub, Schwertern und Brillanten.)

  81. #81 SCHWAR_A
    19. Juli 2012

    @Niels:
    “Hast du übersehen, dass ich dir schon geantwortet hatte?”

    Oh, tatsächlich übersehen… Vielen Dank.

    “Ich weiß gerade nicht, wie du auf Masse^4 kommst.”
    “es gilt wie immer in der theoretischen Physik c = hquer = 1.”

    V(Φ) scheint eine Funktion der Masse hoch 4 zu sein:
      Φ² = v²/2 = -m²/λ
      V(Φ) = -m²Φ² + λ/2Φ^4 = -m^4/λ + ½m^4/λ = -½m^4/λ
    Das macht mir Probleme… V(Φ) ist potentielle Energie, also [Nm], λ ist eine dimensionslose Zahl (Kopplung): Da fehlt mir was, um von [kg^4] auf [Nm] zu kommen…

    Herzliche Grüße.

  82. #82 MartinB
    19. Juli 2012

    @Niels
    pwned.

  83. #83 Name auf Verlangen entfernt
    19. Juli 2012

    @ Niels: ich werde nicht auf alle Ihre Argumente eingehen (ich lebe nicht von Ihren Steuergeldern und habe Gott sei Dank einen arbeitsreichen Tag vor mir).

    Ich habe Sie nicht nach der Definition eines Skalar-Feldes gefragt, oder danach, was Lorentz-invariant bedeutet. Wenn ich das nicht wüßte (oder keine Vorstellung davon hätte, was allgemein mit ART oder SRT (oder sonst einer RT – VSL z.B.) gemeint ist – würde ich hier nicht kommentieren.

    Ich wollte wissen, wie Sie wohl dazu kommen, dem Higgs-Feld bestimmte Eigenschaften zuzuschreiben, was Sie berechtigt, diese Eigenschaften als gesichert anzunehmen, während das dazugehörnde “Teilchen” noch nicht einmal “entdeckt” ist. Ich wollte nicht wissen, warum die Banane krumm ist.

    “Das Higgs-Feld ordnet jedem Punkt des Raumes eine reelle Zahl (einen Skalar) zu.” – nein, tut es gewiss nicht. Selbst wenn das Higgs Feld existierte, ein Skalar-Feld wäre und daher Lorentz-invariant – ordnet das Higgs-Feld niemandem und nichts irgendwelche Zahlen zu. Allenfalls machen Sie das in einem Modell. Der Raum hat auch keine Punkte – mal ganz abgesehen von der permanenten Zugrundelegung des “absoluten” Raumes an ein relativistisches Modell, in dem “Raum” laut Theorie so überhaupt nicht existiert. Das macht übrigens auch M. Bäker ständig. Und bei solchen Überlegungen geht es nicht um Semantik, sondern um Sinn (Ontologie).

    Das Higgs-Feld kann ebenso ein riesiges Vektorfeld sein – so es das Higgs-Feld überhaupt gibt.

    Und wenn es irgendein Feld gibt – dann natürlich das Gravitationsfeld. Das Ihre RTs die Gravitation erklärte widerlegt ja – oft genug gesagt – gerade die Masseberechnung expandierender Galaxien.

    “Der absolute Raum Newtons hat mit dem Higgs-Feld keinerlei Gemeinsamkeiten.
    Dei Frage nach dem Unterschied ist ein bisschen sinnlos.”

    Nicht, wenn die Relativitätstheorien ihrerseits falsch und sinnlos sind.

    Nun, dann sagen Sie doch schlicht, was diesen absoluten Newton-Raum vom Higgs-Feld unterscheiden sollte – vor allem, wenn es ein Skalar-Feld wäre – welche Eigenschaft hätte das Higgs-Feld nicht – die dem absoluten Raum ebenfalls zugeschrieben wurde/wird?

    Eben: gar keine. Und genau darum geht es mir: einmal zu zeigen, wie abstrus sich Ihre (und Ihrer Kollegen) Argumentation dem großen Retro-Akt der Äther-Renaissance nähert – ohne im Geringsten verstanden zu haben, worum es sich handeln könnte – wie auch, mit einer falschen Gravitations-Theorie und dilatierenden Gedanken.

  84. #84 georg
    19. Juli 2012

    Niels· 19.07.12 · 01:02 Uhr
    @Name auf Verlangen entfernt

    mal abgesehen davon, daß Sie auf meine Fragen überhaupt nicht geantwortet haben

    Auf “Wenn der Mond aus Käse ist, warum ist die Erde dann nicht aus Wurst” kann man nun mal ziemlich schlecht antworten. Wenn man außerdem weiß, dass Erklärungen den Fragesteller überhaupt nicht interessieren, weil dieser niemals in Betracht ziehen wird, dass der Mond möglicherweise doch nicht aus Käse besteht…

    Großes Kino! Ich bin begeistert. Danke dafür.

    mfg georg

  85. #85 Karl R.
    19. Juli 2012

    Hallo hier wieder mal ein Greenhorn …
    Das Porblem mit dem Verständnis des Higgsteilchens/feldes sehe ich darin dass das ganze einfach in viel Meta-Inormationen eingebettet sein muss um es zu verstehen oder zumindest ein einfaches Bild davon zu bekommen.

    Z.b: diese Feld-Masse-Teilchen Wirkung tritt ja nicht bei Proton/Neutron (woher haben die die Masse?) sondern nur beim elektron auf. Und das Masse im klassischen Sinn von einer Feldwirkung abhängen soll geht mir nicht in den Kopf;

    Anderes Beispiel : Photonen haben eine Masse wenn auch geringfügig – wie kann es dann seine dass Die sich mit Lichtgeschwindigkeit bewegen? na jetzt kommts : man unterscheidet zwischen Ruhemasse und Mass in Bewegung …

    Für den normal sterblichen – sind wir da schon zu weit weg?
    ich glaube schon. Aber ich finde diese Blogs trotzem grossartig und die Bemühung es verständlich darzustellen ist beeindruckend und das verdient hohes Lob!
    Ein paar Dinge hab ich verstanden? …
    – Das Higgsfled (Die Matrix :)) ist omnipräsent – darum Gottes(Feld)?
    – Das Higgsteilchen kommt so nicht in der Natur vor sondern durch eine mit sehr viel Aufwand erzeugten Feldstörung des Higsfeldes
    – Der Begriff Masse scheint sich nochmals neu zu definieren , nicht nur das Masse im Verhältnis zur Energie sondern auch als Wechelwirkung von Teilchen zu einem Feld – so ist die Masse dann nicht mehr ein eigentümlich dem Teilchen zu geordnete Eigenschaft sonder definiert sich in der Wechselwrikung zum Higgsfeld

    So bis dahin
    LG
    /Karl

  86. #86 Lavie
    19. Juli 2012

    [i]Ich wollte wissen, wie Sie wohl dazu kommen, dem Higgs-Feld bestimmte Eigenschaften zuzuschreiben, was Sie berechtigt, diese Eigenschaften als gesichert anzunehmen, während das dazugehörnde “Teilchen” noch nicht einmal “entdeckt” ist.[/i]
    @Name auf Verlangen entfernt
    Wo sollte da das Problem sein? Aus einer Theorie sollten sich normalerweise Eigenschaften ableiten lassen, auch ohne das diese Theorie jemals experimentell bestätigt wurde.
    Und soweit ich diesen Blog durchgelesen habe, hat nie jemand behauptet, dass man das Higgs-Teilchen entdeckt hat. Zudem lese ich aus Ihren Beiträgen dass selbst wenn das Higgs-Teilchen eines Tages nachgewiesen wird, Sie trotzdem das ganze in die Tonne kloppen. Ist doch so, oder? Was soll also Ihre Bemerkung?
    @MartinB
    Ich habe deinen Blog über die Definition eines Trolls gelesen. Denkst du nicht, dass Name auf Verlangen entfernt all diese Eigenschaften mit sich bringt? Wenn ja, verstehe ich nicht ganz weshalb solche Leute in diesem Blog ihre Berechtigung haben.

  87. #87 Lavie
    19. Juli 2012

    “Z.b: diese Feld-Masse-Teilchen Wirkung tritt ja nicht bei Proton/Neutron (woher haben die die Masse?) sondern nur beim elektron auf.”
    @Karl R.
    Diese Frage wurde schon im Eröffnungspost beantwortet.

  88. #88 MartinB
    19. Juli 2012

    @Karl
    Ja, diese Begriffsverwirrung mit dem Wort “Masse” dürften viele so erleben – ich bastle gerade an einem text zum Thema, da wird auch der Unterschied zwischen masse und Ruhemasse eine Rolle spielen. Letztlich wäre es vermutlich für Laien viel verständlicher, wenn alle leicht unterschiedlichen Aspekte der Masse eigene Namen haben.

    Proton und Neutron ahben den größten Teil ihrer Masse aus ihrer bindungsenergie, nur einen kleinen Teil aus der Ruhemasse der Quarks.

    @Lavie
    Sicher ist MT ein Troll, aber solange Trolle nicht zu lästig werden, sperre ich sie nicht – jeder kann sie ja nach Herzenslust ignorieren. Sonst fühlt sich MT noch als Märtyrer, obwohl er in Wahrheit mehr der Pausenclown ist (und für den einen oder anderen Lacher sind seine Abstrusitäten ja manchmal gut).

  89. #89 Adent
    19. Juli 2012

    @MartinB
    Lol, inzwischen beantwortet MT sich seine Fragen ja selbst, weil er die Physik hinter den richtigen Antworten (z.B. von Niels) nicht versteht. Ist schon ein Brüller der gute….
    Was mir nur immer wieder sauer aufstösst ist seine dogmatische Sichtweise, dass er a) immer Recht hat und b) andere hinzuzieht, die ihm (angeblich) beipflichten und dies dann c) der bösen Physik, die ja so grundlegend falsch liegt wie nur irgendetwas gegenüberstellt. Netter rhetorischer Trick, bei seinem inflationärem Gebrauch aber langsam langweilig.

  90. #90 MartinB
    19. Juli 2012

    @Adent
    Ja, deswegen überfliege ich seine Kommentare nur noch.

    @alle
    Können wir die Meta-Diskussion über Trolle etc. bitte beenden?
    Wer meint, dass irgendwer hier gesperrt gehört oder sich anderweitig über den blog beschweren will, kann mir gern eine mail schicken.

  91. #91 Karl R.
    19. Juli 2012

    Hmm sperren würde ich Trolle nur wenn sie ausdrücklich nicht in Verbindung zum (wahrscheinlich vorhandenen, noch zu beweisenden) Higgsfeld stehen, aber ….
    🙂

  92. #92 SCHWAR_A
    19. Juli 2012

    @Niels & MartinB:
    Bezüglich der “negativen” Masse des VEV v habe ich eine Idee für die Interpretation –
    sie wirkt in entgegengesetzter Richtung:
      V(Φ) = λ/2 · (2Φ^4 – Φ²v²)
    Der erste Term Φ^4 stellt die innere Energie dar, die von einem Punkt aus abgestrahlt werden will, der zweite Term stellt die durch das Higgsfeld verursachte, rückwärtslaufende Energie dar (tatsächlich einen Impuls), die der Abstrahlung entgegen wirkt (also auch hier quasi “Masse verleiht”).
    Die Energie, bei der die beiden Flüsse im Gleichgewicht sind, also gleichviel abstrahlt wie auch wieder zurückläuft, stellt gerade v²/2 dar.

    Herzliche Grüße.

  93. #93 Niels
    19. Juli 2012

    @Name auf Verlangen entfernt

    Und genau darum geht es mir: einmal zu zeigen, wie abstrus sich Ihre (und Ihrer Kollegen) Argumentation dem großen Retro-Akt der Äther-Renaissance nähert

    Genau das ist das Problem und der Grund, warum Ihre Fragen normalerweise nicht beantwortet werden.

    Übrigens müssen Sie sich keine Sorgen machen, bei der Produktion dieser Beitrage kamen keine Steuergelder zu Schaden.
    Schuld war ein schwerer Anfall von Prokrastination. Die Motivation war also sozusagen “Zeitverschwendung”, nicht Geldverschwendung.

    @MartinB
    Zur “Entdeckung” des Higgs:
    Weißt du, ob man bei den Entdeckung des Top-Quarks auch so übervorsichtig war?
    Von der erstmaligen “Entdeckung” bis zur ganz genauen Vermessung dieses Teilchens hat es doch auch ein paar Jahre gebraucht.
    Würde mich eigentlich wundern, wenn man dabei und auch bei den anderen erstmaligen Messungen neuer, theoretisch vorhergesagter Teilchen immer monate- oder gar jahrelang vom “unbekannten Teilchen, dass vermutlich das X-Teilchen ist” gesprochen hätte.

  94. #94 Lavie
    19. Juli 2012

    “Würde mich eigentlich wundern, wenn man dabei und auch bei den anderen erstmaligen Messungen neuer, theoretisch vorhergesagter Teilchen immer monate- oder gar jahrelang vom “unbekannten Teilchen, dass vermutlich das X-Teilchen ist” gesprochen hätte.”

    Würde mich ganz und gar nicht wundern. Ich sehe beim neuen Signal überhaupt keine “Übervorsichtigkeit”. Im Gegenteil wäre es unverantwortlich ohne die experimentell bestimmten Eigenschaften frühzeitig die Entdeckung eines Higgs-Bosons bekannt zu geben. Dass das Internet dabei seine ganz eigenen Gesetze hat, haben wir ja bei den überlichtschnellen Neutrinos erlebt. Die von den Medien nie vorhandene Sensation (was Opera immer ausdrücklich betont hat) wurde in einer Eigendynamik zu einer solchen hochgeschaukelt und führte schlussendlich zum Rücktritt des Opera-Sprechers.

    Von den ersten Anzeichen des Top-Quarks bis zum Ausschluss statistischen Fluktuationen wird jedenfalls sicher auch einige Zeit vergangen sein. Der Unterschied zu heute ist nur der, dass wir in einer viel schneller lebigen Zeit leben und dass Teilchenphysik im heutigen Internetzeitalter inzwischen schon fast Popstar Status erreicht hat. Wer hat denn von der Entdeckung des Top-Quarks damals schon Notiz genommen (ausgenommen Physiker und ein paar Freaks)?

  95. #95 Name auf Verlangen entfernt
    19. Juli 2012

    @ Niels: also, meine Fragen werden normalerweise beantwortet, da kann ich mich gar nicht beklagen. Und das ist auch nicht der Grund für sonstige Kontroversen.
    Wie schaft man es nur, so konsequent und mit allem danebenzuliegen?

    Solange es entscheidende Beobachtungen gibt, die nahelegen, daß eine Theorie bullshit ist, ist es schlicht wissenschaftlich, diese Theorie in ihren Grundfesten anzuzweifeln, anstatt sich Dunkle Energien herbeizufanatasieren. Seid ihr wohl Wissenschaftler – gibt es kein Ehrgefühl mehr? Wir sprechen von 96%. Das ist kein kleiner Fehler, der umfasst das Allermeiste. Daß sich hier die Schulterklopfer die Klinke in die Hand geben, wundert keinen. Es gibt aber andernorts Wissenschaftler, an denen nagt zu Recht der Zweifel, und es wundert schon ein bischen, wie hier in Reih und Glied marschiert wird, zumal mit welcher unfassbaren Arroganz. Kreativ ist das nicht und Steuergelder kostet es – und Leute, wie Sie offensichtlich auch den Verstand. Wir werden es zu verhindern wissen, daß Ihr nächster Milliarden-Wahn finanziert wird. Doch bleiben Sie sich treu – Sie scheinen ja recht jung – das Leben wird´s von selbst richten.

  96. #96 Niels
    20. Juli 2012

    @Lavie

    Von den ersten Anzeichen des Top-Quarks bis zum Ausschluss statistischen Fluktuationen wird jedenfalls sicher auch einige Zeit vergangen sein.

    Es geht aber weder beim Higgs noch beim Top um “ersten Anzeichen”.
    Die “ersten Anzeichen” des Higgs gab es übrigens spätestens im Dezember 2011.
    Siehe z.B. hier:
    https://blog.vixra.org/2011/12/02/higgs-rumour-anaylsis-points-to-125-gev/
    Some clarification at NEW is that ATLAS has a 3 sigma excess at 126 GeV while CMS has a smaller excess at 126 GeV, perhaps 2 sigma, both in diphoton channels. These are close enough to combine to give a 3.5 sigma. That would be enough to claim an “observation” but is well short of “discovery”.

    Es geht ebenfalls nicht um statistischen Fluktuationen.
    Niemand bezweifelt ernsthaft, dass ein neues Teilchen gefunden wurde. Es ist sogar klar, dass dieses Teilchen ein Boson ist.
    https://press.web.cern.ch/press/PressReleases/Releases2012/PR17.12E.html
    The results are preliminary but the 5 sigma signal at around 125 GeV we’re seeing is dramatic. This is indeed a new particle. We know it must be a boson and it’s the heaviest boson ever found

    Es geht darum, dass bei diesem neuen Teilchen noch nicht alle möglichen Zerfallskanäle genau untersucht wurden.
    Deswegen ist es prinzipiell richtig, wenn man sagt, dass man nicht genau weiß, ob das neue Teilchen genau wie das vorhergesagte Higgs zerfällt. Auch wenn alle bisherigen Ergebnisse darauf schließen lassen.
    Meine Frage war, ob man bei anderen Teilchen, etwa den Ws, dem Z oder dem Top, auch so vorsichtig bei der Wortwahl war. Den Nobelpreis für die drei genannten Bosonen gab es zum Beispiel ganz erstaunlich kurz nach den ersten Messungen.

  97. #97 Erik der Unlesbare
    20. Juli 2012

    @ Lavie 19.07.12   11:45 Uhr
    “…,  verstehe ich nicht ganz weshalb solche Leute in diesem Blog ihre Berechtigung haben.”
    Das Phänomen des Welle-Teilchen-Dualismus finden sie nicht nur in der Quantenmechanik. Soziales Verhalten von,  in gegenseitiger Abhängigkeit stehender Menschengruppen, zeigen Verhaltensmuster wie sie der Welle-Teilchen-Dualismus beschreibt.  
    MarkusTermin (steht für Welle-Charakter) ist emotional und intuitiv stark auf die Wissenschaftler (stehen für Teilchen-Charakter) ausgerichtet.  Dabei wirft er aus seinem “Gedanken-Vakuum” Wissen in Richtung des Teilchen-Charakters , welches die Wissenschaftler noch nicht mit ihrer Mathematik “greifen” können. Name auf Verlangen entfernt kann aber nicht ohne sie Wissenschaftler sein, denn der Impuls des Wiederspruchs ist es der neues Wissen oder Gedanken-Gut aus dem NICHT (Vakuum) herausziehen kann. 
    Wenn sie als Wissenschaftler weiter ihre Mathematik in ihren Köpfen bewegen, werden sie die Zusammenhänge  der QM weiter entwickeln. Irgendwann wird das Wissen über Materie und Zeit (Teilchen-Charakter) sich  dem Wissenserwerb von Name auf Verlangen entfernt (Welle-Charakter) annähern.
    Die gegenseitige Abhängigkeit von Wissenschaftlern und Trollen besteht im übertragenen Sinne im Informationsaustausch, welcher im Universum durch Formenvielfalt von Energie verwirklicht wird. Evolution im Universum basiert auf Formen wie Elementarteilchen, Atome, Moleküle, Zellen, Lebewesen, Sprache, Denken …
    Diese Prozesse im Grossen und im Kleinen zu beobachten finde ich sehr interessant.

  98. #98 MartinB
    20. Juli 2012

    @Niels
    Meine Erinnerung an die Top-Entdeckung ist nur noch düster, aber wozu gibt’s denQuell der Weisheit (aka wikipedia):
    “In October 1992, the two groups found their first hint of the top, with a single creation event that appeared to contain the top. In the following years more evidence was collected and on April 22, 1994, the CDF group submitted their paper presenting tentative evidence for the existence of a top quark with a mass of about 175 GeV/c2. In the meantime DØ had found no more evidence than the suggestive event in 1992. A year later on March 2, 1995, after having gathered more evidence and a reanalysis of the DØ data (who had been searching for a much lighter top), the two groups jointly reported the discovery of the top with a certainty of 99.9998% at a mass of 176±18 GeV/c2”
    Da war man auf jeden Fall genauso vorsichtig.

    Beim W und Z ging es meiner Ansicht nach deswegen schneller, weil zum einen die Massen schon vorher grob bekannt waren, wenn ich mich recht entsinne. Außerdem ist der Zerfall von W/Z sicherlich charakteristischer als der des Higgs, das in beliebige Teilchen-Antiteilchen zerfallen kann.

  99. #99 Lavie
    20. Juli 2012

    “Irgendwann wird das Wissen über Materie und Zeit (Teilchen-Charakter) sich dem Wissenserwerb von Name auf Verlangen entfernt (Welle-Charakter) annähern.”
    @Erik
    Ich hoffe nicht.

  100. #100 Name auf Verlangen entfernt
    20. Juli 2012

    @ Lavie schreibt: “Und soweit ich diesen Blog durchgelesen habe, hat nie jemand behauptet, dass man das Higgs-Teilchen entdeckt hat.”

    @ Niels schreibt: “Niemand bezweifelt ernsthaft, dass ein neues Teilchen gefunden wurde. Es ist sogar klar, dass dieses Teilchen ein Boson ist.”

    Frans Klinkhamer – Physiker an der Uni Karlsruhe sagt: “Sicher ist, dass es Kosmologen in größte Bedrängnis bringt. Denn das Higgs-Feld füllt das Universum mit großen Mengen an Energie. »Das würde eine riesige abstoßende Kraft hervorrufen, was aber nicht mal ansatzweise zu den Beobachtungen passt«.

    Klaus Desch, Cern-Physiker von der Universität Bonn sagt:
    »Wir verstehen an der Brücke zwischen Mikro- und Makrokosmos etwas Grundlegendes nicht.«

    Richard Panek meint, der Begriff “Dunkle Materie” könnte auch als Bankrotterklärung in die Geschichte eingehen.

    https://www.zeit.de/2012/29/Physik-Higgs-Teilchen

    Harald Lesch meint: “Beschäftigen wir uns lieber mit realen Dingen.”

    https://www.sueddeutsche.de/panorama/harald-lesch-ueber-higgs-boson-das-versteht-kein-mensch-1.1402850

    Robert Gast schreibt: “Rätselhaft ist etwa, wieso noch kein Überschuss an sogenannten W-Bosonen aus den Teilchenkollisionen in dem unterirdischen Kreisbeschleuniger hervorging. Ohne ihn werden Zweifel bestehen bleiben, ob man es nicht mit etwas ganz anderem als dem Higgs zu tun hat.”

    https://www.zeit.de/wissen/2012-07/higgs-teilchen-cern-kommentar

    Von all diesem Meinungen will man jedoch bei Science-Blogs – die jeder Wissenschaftsredakteur in Deutschland gewiss aufruft, bevor er was schreibt, keine Ahnung haben. Und ein Troll ist, wer sich eine Aussage jenseits des Standartmodells erlaubt:

    B. v. Haller jedoch, früher am CERN, räumt die Möglichkeit ein, es könnte zu einem “konkreten Hinweis auf eine Physik jenseits des Standardmodells” werden – als gäbe es das nicht schon durch Beobachtungen des Universums selbst.

    Zeilinger spricht von einer grundsätzlichen “Neuformulierung unserer Ideen von Zeit und Raum.” (FAZ 12, Okt 2008)

    Bernard d’Espagnat – quasi CERN-Mitbegründer und einer der wenigen Denker dort weiß, daß am CERN nicht die Wirklichkeit erforscht wird. (FAZ 2. März 2008).

    @ Erik: Ich bin hier nicht just for fun, oder um mich als Laie wissenschaftskundig aufzuspielen, sondern um aufzuzeigen – deswegen die oberen Zitate – daß man bei den SB (der Skeptiker Haupt-Nebenorganisation) eine gewissermaßen stalinistische Variante der Naturwissenschaften propagiert.

    Naturwissenschaftler – sofern man einen Typus verallgemeinern kann – beanspruchen die “Realität” für alle verbindlich zu deuten – da nutzt es, dem unbefangenen Leser den entsprechenden Charaktertypus vorzuführen und vor allem die Lückenhaftigkeit des Weltbilds aufzuzeigen, das dem Anspruch keinesfalls gerecht werden kann. Wissenschaft an sich ist sinnvoll und klasse – bescheiden eingeordnet in den Kreis anderer Methoden, von denen sie – da magst Du Recht haben – nur lernen kann – den Menschen wirklich nützliches Wissen zu finden. Jede/r jede sollte sich für Naturwissenschaft interessieren und den Fachleuten auf die Finger gucken. Das ist auch gar nicht schwer, wenn man sich vom kryptischen Formel-Gemurmel nicht einlullen läßt und sich selbst – wie auch beim Higgs gut möglich, ein Urteil zutraut.

    Das geht auch beim Aspect-Experiment – das bei allen theoretischen Erwägungen hier grundsätzlich ignoriert wird – als hätte es das nie gegeben, als wären etwa Zeiten und Entfernungen, die sich aus dem Standartmodell für´s Universum ergeben, gesicherte Tatsachen … “Baby-Galaxie mit dem Alter von 3,5 Milliarden Jahren, etc.”, und man hätte hier gerade noch mit ein paar kleinen Fragen zu tun, ein Teilchen zu entdecken, dann wäre alles geklärt und Gott überflüssig (Stephen Hawking).

  101. #101 MartinB
    20. Juli 2012

    @MT
    “Von all diesem Meinungen will man jedoch bei Science-Blogs – die jeder Wissenschaftsredakteur in Deutschland gewiss aufruft, bevor er was schreibt, keine Ahnung haben.”
    Schauen Sie mal auf das zweite Wort dieses textes. Sie müssen nicht mal alles lesen (geschweige denn verstehen), aber zumindest den ersten Satz könnten Sie doch angucken.

    Auch das Problem der Nullpunktsenergie (das sich bei allen Quantenfeldern stellt, nicht nur beim Higgs) wird hier auf diesem Blog überrraschenderweise nicht totgeschwiegen, siehe z.B.
    https://www.scienceblogs.de/hier-wohnen-drachen/2012/05/qft-fur-alle-das-vakuum.php

    Dort kann man übrigens des Satz lesen (wenn man denn lesen kann)
    “Meine persönliche Meinung ist eher die, dass das hier eins der vielen kleinen offenen Probleme der Quantenfeldtheorie ist, das darauf hindeutet, dass diese nicht der Weisheit letzter Schluss ist. ”

    “Das geht auch beim Aspect-Experiment – das bei allen theoretischen Erwägungen hier grundsätzlich ignoriert wird”
    Warum sollte ich das Aspect-Experiment, das die Bellschen Ungleichungen bestätigt (die hier schon mehrfach thematisiert waren) ignorieren? Das Aspect-Experiment ist eine der besten Bestätigungen der QM und der Problematik Lokalität/Realität, die es gibt.
    Siehe z.B. hier
    https://www.scienceblogs.de/hier-wohnen-drachen/2011/05/quantenmechanik-nichtlokalitat-und-unscharfe.php
    (auch wenn der Name Aspect da nicht explizit auftaucht, geht es da ja genau um solche Fragen)

    Zum Troll macht Sie nicht die tatsache, dass Sie die Physik und das Standardmodell attackieren – zum Troll macht Sie die Tatsache, dass Sie statt die texte hier zu lesen lieber so viele Strohmänner bauen, dass dieser Blog bald besonderer Brandschutzmaßnahmen bedarf.

    Und natürlich die “Arrogance of ignorance” die mit solchen Sätzen deutlich wird
    “Das ist auch gar nicht schwer, wenn man sich vom kryptischen Formel-Gemurmel nicht einlullen läßt und sich selbst – wie auch beim Higgs gut möglich, ein Urteil zutraut. ”
    Um so komischer (oder trauriger), dass Sie andererseits Nicht-Astrologen natürlich kein Urteil über die Astrologie zugestehen.

    Stalinistisch ist allenfalls Ihr Versuch, die Realität hinzulügen (der eine löscht einen unliebsamen Herrn Trotski von einem Foto, der andere unliebsame Kommentare von seinem Blog oder falsche Daten für den Untergang der Titanic…)

    PS: Standart (oder eigentlich Standarte) ist ne kleine Fahne am Auto
    https://de.wikipedia.org/wiki/Standart

  102. #102 Name auf Verlangen entfernt
    20. Juli 2012

    @ Martin Bäker: nun freilich, darüber schreiben Sie – aber wo liest man bei Ihnen, daß der Realismus der Quantentheorie auf Kosten des Realismus der Relativitätstheorie geht, und umgekehrt: daß sich beide Theorien ausschließen? Und das folglich alle Entfernungs- und Zeitangaben im Universum und im Standard-Modell (vielen Dank für die Korrektur des “De”) – wie es geflissentlich als Realität verkauft wird – keinesfalls stimmen können, wenn wir die Quantentheorie und deren Nicht-Lokalität anerkennen – und nichts anderes bleibt uns nach EPS, Bell, Aspect und Zeilinger wohl übrig?

    Im Gegenteil – zweifelt jemand an Einstein, berichten Sie “er müsse dann schon verdammt viel erklären”.

    Dabei täte es der Physik gut, offen zuzugeben, daß sie sich in einer völlig schizophrenen Situation befindet.

    Was machen Sie stattdessen? – : die Astrologie als Strohpuppe aufbauen – den Feind des Unbegreiflichen und vermeintlich Irrationalen ins Äußere verlegen.

    Selbstverständlich kann man über Astrologie diskutieren, oder deren Ergebnisse kritisieren – wir bitten darum; was aber gar nicht geht, ist deren Existenz anzuzweifeln. Ich kann auch nicht sagen: es gibt keine Physik. Natürlich gibt es die, und so auch Astrologie.

    Um nun zu behaupten – der Gegenrede zuvorkommend – Astrologie sei komplett irrational – ist der eigene Standpunkt wohl überhaupt nicht rational genug fundamentiert – von welchem Platz aus wollten Sie das beurteilen? – dem der Nicht-Lokalität? – wir sind zwar keine Naturwissenschaft im eigentlichen Sinne – aber genau so objektiv, wie mindestens jede andere “weiche” Wissenschaft.

    Würden Sie sich mit derselben Energie um ein Verständnis für die Astrologie bemühen, wie ich mich um Physik bemühe, könnte ich wohl nur den Hut vor Ihnen ziehen. Das ist aber nicht der Fall, das halten Sie nur für lächerlich.

    Stattdessen applaudieren Sie hier regelmäßig ihrem vollignoranten Astronomen, der sich für Astrologie-bewandert hält, und noch nicht mal weiß, was ein Tierkreis ist – und treten in einem Portal auf, wo bestallte Wissenschaftskräfte nichts anderes zu tun haben, als “Skeptiker”-Versammlungen zu promoten, die auf dem selben Niveau herum dümpeln.

    Ich möchte hiermit auch ausdrücklich jede Meta-Diskussion über Astrologie beenden – Sie haben das erwähnt. Ich werde aus Gründen der Fairness zu keinem Kommentar weiter Stellung nehmen, der in diesem Sinn völlig off-topic ist.

  103. #103 MartinB
    20. Juli 2012

    “wo liest man bei Ihnen, daß der Realismus der Quantentheorie auf Kosten des Realismus der Relativitätstheorie geht, und umgekehrt: daß sich beide Theorien ausschließen? ”
    Nirgends, wäre ja auch falsch.

    “zweifelt jemand an Einstein, berichten Sie “er müsse dann schon verdammt viel erklären”. ”
    Und habe das ja auch mit hinreichend vielen Argumenten (das sind diese Dinger, mit denen Sie so Probleme haben, nicht zu verwechseln mit leeren Behauptungen) belegt.

    “Was machen Sie stattdessen? – : die Astrologie als Strohpuppe aufbauen”
    Ich habe noch nie über Astrologie geschrieben (außer in Kommentaren) – aber es ist schon amüsant, wenn Sie sich herausnehmen, die theoretische Physik beurteilen zu wollen, aber umgekehrt einem Astronomen nicht zugestehen, sich über Astrologie zu äußern.

    “Würden Sie sich mit derselben Energie um ein Verständnis für die Astrologie bemühen”
    Das habe ich in der Anfangszeit dieses Blogs sogar versucht (erinnern Sie sich?) – habe von Ihnen aber auf meine entsprechenden Fragen wenig bis gar keine Antworten bekommen.

    “Stattdessen applaudieren Sie hier regelmäßig ihrem vollignoranten Astronomen”
    citation needed wie man im Englischen sagt.

    “Sie haben das erwähnt.”
    Ich habe das nur erwähnt, um deutlich zu machen, wie sehr Sie mit zweierlei Maß messen.

  104. #104 rolak
    20. Juli 2012

    Nirgends, wäre ja auch falsch.

    1) Danke für den vergnüglichen WE-Start.
    2) Mist – Kaffeeflecken wegwischen…

  105. #105 Name auf Verlangen entfernt
    20. Juli 2012

    “wo liest man bei Ihnen, daß der Realismus der Quantentheorie auf Kosten des Realismus der Relativitätstheorie geht, und umgekehrt: daß sich beide Theorien ausschließen? ” Nirgends, wäre ja auch falsch.”

    O.k. – vielleicht sollte man aber doch in einer Diskussion erwähnen, daß es ernstzunehmende Forscher gibt, die das anders sehen, anstatt (es zuzulassen) eine andere Meinung als “crank” etc. abzutun? – :

    Cyril Branciard:“Ich glaube, dass die Geschwindigkeit in Wirklichkeit unendlich ist.”

    https://www.spiegel.de/wissenschaft/mensch/mysterioeses-quantenphaenomen-einsteins-spuk-ist-tausende-male-schneller-als-das-licht-a-572068.html

    https://library.thinkquest.org/C007354/Htm-Dateien/Widersprueche.htm

    https://homepages.uni-tuebingen.de/th.mueller/eigenes/arbeit.pdf

    “Würden Sie sich mit derselben Energie um ein Verständnis für die Astrologie bemühen” Das habe ich in der Anfangszeit dieses Blogs sogar versucht (erinnern Sie sich?) – habe von Ihnen aber auf meine entsprechenden Fragen wenig bis gar keine Antworten bekommen.”

    Das erste mag stimmen – da bin ich ungerecht – es mag an dem Umfeld liegen – Leute, wie @rolak beispielsweise – was natürlich meinerseits die Sache hochschaukelt und dann zu Verzerrungen führt. Außerdem werden von außerhalb die Kampagnen in SB oft als konzertierte Aktion wahrgenommen. Dass Sie keine Antworten bekommen haben allerdings stimmt nicht – und es sind immer Antworten im Umfeld einer Horde, die gleichzeitig jedes Geschütz auffahren, um zu diffamieren und überhaupt nicht daran denken, sich mit etwas wirklich zu beschäftigen. Da geht schon mal was unter.

    Der Astronom darf sagen, was er will – dann jedoch noch nicht einmal zu wissen, was der Tierkreis ist, aber gleichzeitig präjudizierend über Astrologie zu schreiben, wäre vergleichbar mit jemandem, der nicht weiß, daß Geschwindigkeit ein Quotient aus Weg und Zeit ist. Er kann also eben nicht ernst genommen werden. Es wäre ja auch merkwürdig, zu behaupten, der Astronom hätte jemals daran gedacht, die Sache ernsthaft zu untersuchen.

    “citation needed wie man im Englischen sagt.” Ihre Kommentare – insbesondere bei Freistetters sind ja auch gemeint – und der Ton, in dem Sie gehalten sind – die such ich jetzt aber nicht alle raus, oder?

  106. #106 Dr. Webbaer
    20. Juli 2012

    @Termin
    Die Physiklehre ist ein wenig soft geworden, was die “Teilchen”-Physik und die Quantenphysiklehre betrifft, aber – lassen Sie sich das mal gesagt sein – es gibt hier aus esoterischer Sicht nichts zu saugen.

    Stellen Sie als exponierte Meinungskraft Ihrer Esoterik doch erst einmal eine Wissenschaft auf, selbst die sog. Frauen- oder Vorurteilsforschung hat es geschafft ein wissenschaftliches Gebilde zu formen.

    MFG
    Wb

  107. #107 MartinB
    20. Juli 2012

    @MT
    Dass der “Kollaps der WF” überlichtschnell ist, ist ja nichts wirklich neues. Das sorgt trotzdem nicht für einen Widerspruch zwischen QM und RT, egal wie oft Sie das behaupten.

    “Dass Sie keine Antworten bekommen haben allerdings stimmt nicht”
    Das sehe ich anders, siehe hier:
    https://www.scienceblogs.de/hier-wohnen-drachen/2010/12/physik-und-geist-dialog-uber-zwei-weltsysteme.php

    Alles was ich auf meinen sehr langen Kommentar zum Thema “Qualität der Zeit” bekommen habe, war ein lapidares “nein” und die Behauptung, ich würde von etwas ablenken. Positive Antworten zu Ihrer Weltsicht sind Sie schuldig geblieben, es kamen lediglich Aussagen, was die Naturwissenschaft angeblich alles falsch machen.
    Und ja, ob Sie es glauben oder nicht, damals war ich wirklich ernsthaft an Ihrer Weltsicht interessiert, weil ich die Möglichkeit durchaus in Betracht zog, dass hier ein intelligenter Mensch zu vollkommen anderen Schlüssen über die Welt kommt als ich. Inzwischen habe ich Sie leider als jemanden kennen gelernt, der Dinge verbiegt, falsch darstellt, der andere beleidigt, wenn ihm die Argumente ausgehen etc.

  108. #108 Name auf Verlangen entfernt
    20. Juli 2012

    @ Martin Bäker: Komisch, ich hab da mal nachgelesen – finde zwei äußerst nette Diskussionsbeiträge von mir selbst, ganz spontan: leben wir in verschiedenen Universen? – :

    “Wie bereits diskutiert, ist hier der Punkt, wo Sie sich vorbeimogeln. Der Diolog ist sonst fein und interessant! Warum vorbeimogeln? … weil es tatsächlich nichts anderes ist: Mathematik muß der Logik gehorchen, nicht die Logik der Mathematik. Egal, wie Sie die “Regel” nennen, ihre Referenz bleibt die Mathematik und damit die Logik und damit der Geist. Gemäß ihrem Dialog hätte der Geist – an diesem Punkt konsequent den Sieg davontragen müssen.”

    12.12.10 · 14:09 Uhr
    @ MartinB: Eigentlich war ich dran mit dem Frage-stellen, aber bitte:
    Ich schreibe: “Den Null-Punkt ohne Raumausdehnung stellt die konstante Lichtgeschwindigkeit dar.”

    Zum letzteren Punkt – weil er ein wenig topic ist – finde ich heute sogar Beistand durch wissenschaftliche Metaphysiker, die im Wesentlichen dasselbe sagen:

    https://www.science20.com/alpha_meme/fundamental_nature_light-75861

    Wie dem auch immer sei: sollten Sie beleidigt worden sein, bitte ich ausdrücklich um Entschuldigung. Ich bin auch nicht so oft “amüsiert”, wie Sie.

    Immerhin machen Sie zu Recht darauf aufmerksam, daß man einige Zeit miteinander verbracht und gestritten hat. Können sich jetzt natürlich Steinbock-mäßig auf die Würde Ihrer Person beziehen – oder aber man trifft sich mal per Realität oder skype und findet einen neuen Anfang, der allen was bringt und aus Feindschaften neue Inspiration schafft. Das hätte auch Freistetter bei seinem Besuch in Nürnberg neulich offen gestanden – alles hätte geklärt werden können – stattdessen schreibt er, ich soll ihn bloß im Planetarium bei seinen Skeptiker-Spinnern in Ruhe lassen, er hätte “keine Lust auf Astrologie” – ich dräng mich da nicht auf.

    @ WB: Ja, da haben Sie wieder Recht, alter Bär, aber so richtig suck´n sind eigentlich Sie selbst mit Ihrer bärigen Hinterlist. Als hätte ich mich hier bemüht, irgendeine Esoterik in den Vordergrund zu bringen und daraus einen Vorteil zu ziehen. Man kann keine Wissenschaft aufstellen mit Astrologie (tut mir echt leid, ich werde meinem Vorhaben untreu, darüber nichts zu sagen), weil Astrologie selbst der Urgund der Wissenschaft ist. Wir werden uns also nicht unter dem Dach der Wissenschaft einfinden, sondern die Wissenschaft wird wieder heimkehren ins ursprüngliche Haus der Astrologie – die damals nicht so hieß und auch in Zukunft – sich wandelnd – etwas anderes sein wird.

    Frauenforschung ist übrigens keine “Vorurteils-Forschung” – eher umgekehrt – nicht wahr? Handelt es sich um Ihren originären Beitrag zur Beschneidungs-Debatte?

  109. #109 Niels
    20. Juli 2012

    https://library.thinkquest.org/C007354/Htm-Dateien/Widersprueche.htm

    Wir haben also herausgefunden, das Volumen eines Schwarzen Loches der Masse eines Photons der Wellenlänge λ ist größer als das Volumen eines Photons VPhoton ≈ λ³.
    Da das Photon kleiner ist als das Schwarze Loch, aber die gleiche Masse hat wie das Schwarze Loch, folgern wir daraus:
    das Photon muss auch ein Schwarzes Loch sein.

    LOL

  110. #110 Ralph Ulrich
    20. Juli 2012

    Zitat:
    “Da das Photon kleiner ist als das Schwarze Loch, aber die gleiche Masse hat wie das Schwarze Loch, folgern wir daraus: das Photon muss auch ein Schwarzes Loch sein.”

    Was können wir daraus lernen:
    Für massive Materie gilt das Pauli Prinzip: Nichts kann da sein wo schon was ist. Schwarze Löcher werden also von einer Lichtnatur sein.

    Was aber der Author nicht bedacht:
    Man kann die Ausdehnung eines Photons in Wahrheit nicht sinnvoll angeben, denn wir leben in einem anderen ZeitRaum Kontinuum. Für ein koheräntes im Quantenzustand befindiliches – also lebendiges – Photon, besteht seine Ausdehnung vom Ursprung bis zum Untergang, weil es bei Lichtgeschwindigkeit nur einen Moment der Zeit gibt.

  111. #111 Ralph Ulrich
    20. Juli 2012

    Kann man also sagen:
    Schwarze Löcher sind Klumpen von eingefrorenem Licht. Eingefroren, weil ja die Zeit in ihnen durch die starke Gravitation nicht mehr vergeht.

  112. #112 rolak
    21. Juli 2012

    Nun lach nicht, Niels, wer einen so schön dicken Multiplikationspunkt hat, muß anderweitig dünne Stellen haben – und nicht nur die Logik sieht dort arg löchrig aus.

    Klar, Ralph,der Klimawandel kommt von der Abwärme der ganzen Licht-Tief­kühl­truhen.

  113. #113 Ralph Ulrich
    21. Juli 2012

    @rolak, nun lach nicht über mich. Ich wollte nur zeigen, dass es eine Platitüde ist, zu behaupten, dass das kleinste schwarze Loch ein Photon ist. Schließlich was bleibt übrig bei der Hawking Zerstrahlung kleiner schwarzer Löcher!

  114. #114 rolak
    21. Juli 2012

    Ich lach ja gar nicht, Ralph, nicht nach Lesen der Vorlage. Photonenvolumen=λ³, das ist doch ein preiswürdiger Hammer – jetzt ist nämlich auch endlich klar, warum die Radios früher so groß waren: Bei Langwelle mußten da kubikkilometergroße Photonen reinpassen, form follows function, logo.

    Jedoch sollte selbst der elfhundertelfte Aufguß dieses terminschen Unsinnverschleuderns nicht von dem eigentlichen, ziemlich interessanten thread-Thema ablenken.

  115. #115 MartinB
    21. Juli 2012

    @MT
    “finde zwei äußerst nette Diskussionsbeiträge von mir selbst”
    Ja, und auf meine Nachfrage zu genau diesen kam – nichts.

    Aber, wie gesagt, das war damals, als ich noch dachte, bei Ihnen könnte es sich um jemanden handeln, der auf ehrliche und intellektuell redliche Weise zu einer anderen Ansicht über die Welt gekommen ist. Inzwischen kenne ich Ihre Methoden besser und habe an einem Meinungsaustausch mit Ihnen kein Interesse.

    Nicht zu vergessen die Unlogik Ihrer Ansichten – wer die drei Aussagen “nein, die Physiker können methodenbedingt die SRT nicht widerlegen”, “wenn die Physiker die SRT widerlegen würden, würden sie das niemals zugeben” und “OPERA hat die SRT widerlegt” alle drei gleichzeitig behaupten kann, der lebt wohl wirklich in einem anderen Universum in dem eine seltsame mehrwertige Logik herrscht (Vermutlich wie bei Pu der Bär “Wenn ich mich irre, will ich nicht Pu der Bär heißen. Ich heiße aber Pu der Bär und das beweist, dass ich recht habe”.)

    ” sollten Sie beleidigt worden sein,”
    Sowas nennt man wohl “not-pology”. Da kann (angesichts der Tatsache, dass Siemir angelegentlich “Wahnvorstellungen vorgeworfen haben, wenn ich mich recht entsinne), wohl von “sollten” nicht die Rede sein, und die nette Passivkonstruktion spricht auch Bände – nicht mal entschuldigen können Sie sich ehrlich.

    Deswegen diskutiere ich nicht mehr, sondern antworte nur noch, wenn ich der Ansicht bin, dass etwas – für die anderen Leser hier – der Richtigstellung bedarf. Sie wurden gewogen und für zu leicht befunden.

  116. #116 Ralph Ulrich
    21. Juli 2012

    @MartinB,
    Ich habe zufrieden vernommen, dass sie einen neuen Artikel planen über das Verhältnis der Massen aus dem Higgsmechanismus und den starken Bindungen. Ich versthe nämlich nicht, wie das überhaupt kein Problem werden könnte, dass diese verschiedenen Massemechanismen in Punkto
    Trägheit zu Gravitation
    immer in einem gleichen harmonischen Verhältnis bleiben. Es sind doch noch ander Teilchen bekannt, die vielleicht eine noch höhere Bindungsenergie aufweisen. Diese hätten dann bei gleicher Masse und gleicher Gravitation eine geringer Trägheit, wenn sie einen geringeren Masseanteil des Higgsmechanismus hätten.

  117. #117 MartinB
    21. Juli 2012

    @Ralph
    Ich habe ja schon mehrfach erklärt, dass es generell nicht die Masse, sondern der Energiegehalt ist, der Trägheit und Schwerkraft bestimmt. Für den ist es vollkommen egal, ob das Bindingsenergie von Quarks oder Wechselwirkung mit dem Higgsfeld oder sonst etwas ist.

  118. #118 Ralph Ulrich
    21. Juli 2012

    @MartinB
    Das ist erstmal eine “sture” Behauptung 🙂

    Ok, lassen wir die Gravitation mal weg, für die es “extra” – unentdeckte Mechanismen geben mag, bei der allgemein Energien den Raum krümmen.

    Aber was könnte zu der Vermutung Anlass geben,
    – dass einerseits für die fünf Prozent Higgsfeld-Energie-Masse es ein ganz eigenes Feld im Raum gibt, das so stark ist, dass es wahrscheinlich 70 Prozent der Gesamtenergie ausmacht in Form von dunkler Energie
    – dass aber die 95 Prozent der starkten Bindungsenergie der sichtbaren Normal-Materie an dieses Feld gar nicht koppeln

    … aber trotzdem die gleichen Trägheitseffekte in gleichem Masse herbei geführt werden?

    Oder könnte es sein, dass spezielle Phänomene im Universum, die unterschiedliche Anteile an Higgsfeld zu Bindungsenergie haben, dass diese ein von uns ganz unerwartetes Verhältnis von Schwerkraft zu Trägheit aufweisen?
    Haben zB Neutronensterne das gleiche Verhältnis Bindungsenergie/Higgsfeldenergie wie unsere normal Protonen/Neutronen Materie?

  119. #119 Ralph Ulrich
    21. Juli 2012

    Ausserdem – fällt mir gerade ein – muss auch noch die dunkle Materie an das Higgfeld koppeln. Sonst gäbe es nicht diese dunkle Materie Felder zweier Galaxien, die träge weiter auseinander fliegen obwohl sich ihre sichtbare Materie schon (durch Reibung?) ineinander verhakt hat.

  120. #120 Ralph Ulrich
    21. Juli 2012

    Das Pauli Prinzip, dass Elektronen auf Schalen um den Kern sich wolkig ausbreiten, ohne denselben Raum einnehmen zu können, wird doch beim elektro-magnetischen Feld so erklärt, dass es eben an einem Punkt im Raum nur den einfachen Elektronenwert annehmen kann. Vielleicht hat das Higgsfeld auch so eine Ausschlußeigenschaft: An einem Punkt kann maximal ein Higgsteilchen existieren. Vielleicht ist das ein Grund, dass Raum gekrümmter – was heisst dichter – an Massen sein muss. Dann würde das Higgsfeld indirekt zur Raumbeschaffenheit beitragen und auch die Gravitation zumindest mitbestimmen.

  121. #121 MartinB
    21. Juli 2012

    “aber trotzdem die gleichen Trägheitseffekte in gleichem Masse herbei geführt werden?”
    Wie oft muss ich es denn eigentlich noch wiederholen?
    Träge Masse = Schwere Masse = Energie.
    Woher die Energie kommt, ist für die Trägheit und die Schwerkraft unerheblich.
    Es ist wirklich und wahrhaftig so simpel.
    Die Masse, die vom Higgsfeld kommt, liefert einen Teil der Energie.

    Um es mit einer Analogie zu erklären: Sie wundern sich gerade ganz fürchterlich, warum das Geld, das ich für meine Arbeit bekomme und das, das ich fürs Bloggen bekomme, und das, das ich vielleicht als Vortragshonorar bekomme, alle gleichermaßen in Euro messbar sind und dass jeder dieser Euros genau dieselbe Kaufkraft hat.

  122. #122 Lavie
    21. Juli 2012

    @MartinB
    “Woher die Energie kommt, ist für die Trägheit und die Schwerkraft unerheblich.”
    Unerheblich? Von einem Photon geht auch Schwerkraft aus?

  123. #123 SCHWAR_A
    21. Juli 2012

    @Niels & MartinB:
    Koppeln eigentlich Photonen auch an das Higgsfeld? Und wenn nein, warum nicht?

    Herzliche Grüße.

  124. #124 MartinB
    21. Juli 2012

    @Lavie
    Ja, natürlich. Photonen erzeugen ein Schwerefeld, so wie jede andere Energie auch.

    @SCHWAR_A
    Nein, tun sie nicht. Von den ursprünglichen 4 masselosen Bosonen der elektroschwachen WeWi vereinen sich drei mit drei Komponenten des Higgsfelds, das vierte ist das masselose Photon.

  125. #125 SCHWAR_A
    21. Juli 2012

    @MartinB:
    “das vierte ist das masselose Photon.”

    …also: “masselos” als Begründung für “koppelt nicht an Higgsfeld”?

  126. #126 MartinB
    21. Juli 2012

    @SCHWAR_A
    Zum einen ist das hier nicht exakt dieselbe Kopplung wie z.B. bei den Fermionen, sondern eine etwas andere (denn um aus einem masselosen en massives Vektorboson zu machen, braucht man einen zusätzlichen Freiheitsgrad, siehe Teil 2 der Langfassung der Higgserklärung).
    Zum anderen ist es natürlich die Frage, was die Begründung wovon ist. Wenn du das Standardmodell als fundamental ansiehst, dann ist “ist entkoppelt vom Higgsfeld” die Begründung dafür, dass das Photon masselos ist, wenn du umgekehrt mit der Phänomenologie startest, dann kannst du sagen “weil das Photon masselos ist, darf es nicht ans Higgsfeld koppeln bzw. kein Higgsteilchen fressen”. Das ist in meinen augen Geschmackssache, denn die Physik beantwortet keine “Warum”-Fragen.

  127. #127 Name auf Verlangen entfernt
    21. Juli 2012

    Die feinen Unterschiede im Denken anderer fallen Ihnen nicht auf, und Sie merken auch nicht, wenn Ihre Fragen nicht innerhalb Ihres eigenen – tatsächlich verwirrten Erwartungshorizonts zu klären sind – Sie müssen halt in einem Gespräch auf Rede und Widerrede achten – das machen Sie überhaupt nicht, streuen aber dann und wann kleine Sticheleien ein, wie “schwach”, “amüsant” und jetzt wieder dieser blöde Spruch mit dem Wiegen.

    Was soll diese Aufplusterei?

    ” … Naturwissenschafts-Skeptiker und (!) Sympathisanten, wie mich, ist es eine klar erkennbare Tatsache, daß ihr “System” aufgrund einer logischen Verwirrung an zentraler Stelle ihr und vieler Denken ganz grundsätzlich auf den Kopf stellt – eine Art Gehirnwäsche – ohne, daß Ihnen das bewußt wäre – ” und wenn Sie eine Antwort bekommen, die Ihnen nicht gefällt und sie Ihrerseits aufmerksam darauf gemacht, werden, daß Sie nicht beim Thema sind, fällt Ihnen das gar nicht auf.

    Das gilt auch für Ihre Art, die Wahrheit zu verdrehen. Das erste ist wohl Wappler – das zweite gerade durch die “lockerer-Kabel-Affäre” gut bestätigt – und das dritte von mir grundsätzlich als völlig wurscht allenfalls psycholanalytisch interessant dargestellt – wie es ja auch ist.

    Nehmen Sie sich den Satz Ihres Kollegen Hubert Gönner zu Herzen:

    “Die Einsteinsche Relativitätstheorie zeigt denselben Mangel wie die Newtonsche: auch in ihr treten keine Relativ-größen, wie etwa die Relativgeschwindigkeit auf, sondern auf das lokale Feld bezogene Absolutgrößen.”

    Mehr muss ja eigentlich gar nicht gesagt sein. Was das für Konsequenzen und Schlussfolgerungen nach sich zieht – dafür habt ihr Physiker euch offenbar das Denken völlig abgewöhnt – und an dessen Stelle den mathematischen Physikalismus gesetzt. Sie sollten verstehen, daß man sich so nicht austauschen kann. Mann kann auch schlecht jemanden ernst nehmen der diskutiert und dann sagt, er würde es nicht tun.

    Auf wessen Seite ist da der Widerspruch? Auf meiner nicht: Ich rede mit Ihnen und sag auch, daß ich´s tue.

  128. #128 Dr. Webbaer
    22. Juli 2012

    Sie wurden gewogen und für zu leicht befunden. (Dr. Bäker)

    (…) und jetzt wieder dieser blöde Spruch mit dem Wiegen.
    Was soll diese Aufplusterei? (Termin)

    Bibelfestigkeit, Dr. Bäker beweist diese. – Ansonsten haben Sie natürlich recht, Leistungsbewertungen fordert ein Kommentator idR nicht an, sollten aber nur auf Wunsch erstellt werden.

    Wenn Sie Lust haben können Sie auch gerne das Gönner-Zitat belegen wie erklären.

    MFG
    Dr. Webbaer

  129. #129 Name auf Verlangen entfernt
    22. Juli 2012

    Das Gönner-Zitat gibt´s in dem Büchleich “Einsteins Relativitätstheorien”, C.H.Beck 2005, S. 71 – es stellt den logischen Grundwiderspruch der RT-Theorien dar – wie in SB von mir immer so und nie anders vertreten – und im Übrigen auch fast der einzige aber fundamentale, den ein Laie ohne physikalische Fachkenntnisse berechtig anbringen kann. Im Zitat habe ich ein Wort vergessen, es ist hier fett gedruckt und erklärt das Zitat auch:

    “Die Einsteinsche Relativitätstheorie zeigt denselben Mangel wie die Newtonsche: auch in ihr treten keine Relativ-größen, wie etwa die Relativgeschwindigkeit auf, sondern auf das lokale metrische Feld bezogene Absolutgrößen.”

    Die RTs arbeiten – vereinfacht ausgedrückt – mit einem riesigen Zollstock – der nach heutigen Messungen (alle unterscheiden sich historisch um bis zu 10%) 299 792 458 Meter lang ist und die “dimensionslose” Geschwindigkeit des Lichts in einer Sekunde pro Sekunde darstellt: c = 1.

    Unbefangene Leser kratzen sich schon hier am Hinterkopf. “Dimensionslos”? Wie kann eine Geschwindigkeit, die sich von A nach B bestimmt, “dimensionslos” sein?

    Der grundsätzliche Widerspruch ist jedoch der in diesem Sinn absolute metrische Maßstab, der selbst zu dem Ergebnis führen soll, daß alle anderen Größen ihm gegenüber relativ sein sollen. Das ist vergleichbar mit einer Linse, durch die man blickt, die alles um einen herum verzerrt – selbst jedoch nicht verzerrt wird, wenn sie angeguckt wird. Einstein setzt damit den Newtonschen Raum-Container voraus, er entzieht den Resultaten seiner eigenen Theorie von vornherein die Grundlage.

    Von diesem Grundwiderspruch wird durch die Frage, ob Lichtgeschwindigkeit addierbar oder höher als “c” sein könnte nur abgelenkt.

    Dieser Grundwiderspruch ist so beschähmend einfach logisch zu verstehen, daß man es nicht anders, denn als Wahngebilde bezeichnen kann, wie darauf eine Wissenschaft aufgebaut wurde, ein riesiger, superteurer babylonischer Turm gewissermaßen.

    Warum hat man das gemacht? Der Mensch verträgt keine Welt ohne physikalischen Geozentrismus und nichts anderes sind die RTs gleich welcher Spielart – sie extrapolieren eine terrestrische Messgröße ins All und sorgen durch den Irrtum, diese Größe sei selbst nicht den Konsequenzen der Schlussfolgerungen aus der Theorie unterworfen, für ein grundsätzlich nahezu unentwirrbares Weltbild und machen aus dem Universum eine Disney-Projektion ihrer eigenen falschen Vorstellungen.

    – Einsteins Wirkung ist die eines Tricksers, der gut ausgebildete Praktiker zurück ins Mittelalter führt – er beendet die Zeit der Aufklärung und läutet eine Zeit des wissenschaftlich-pseudo-religösen Dogmatismus ein, der an die Stelle der Kirche tritt.

  130. #130 MartinB
    22. Juli 2012

    @MT
    Habe das Buch von Gönner zwar nicht, aber aus der Tatsache, dass erselbst die Relativitätstheorie für korrekt hält, könne wir wohl schließen, dass Sie (wie üblich) hier einen Satz so aus seinem Kontext gerissen haben und dass Herr Gönner Ihnen nicht zustimmen würde, was Ihre Schlussfolgerungen (ich verwende das Wort hier mal im weitest denkbaren Sinne) angeht. Soll ich ihm eine mail schreiben und ihn fragen, was er von Ihrem Kommentar hier hält?

  131. #131 Name auf Verlangen entfernt
    22. Juli 2012

    @ MartinB: wenn Sie mögen. Das Problem ist halt, daß man aus der Naturwissenschaft das Denken selbst verbannt hat und an dessen Stelle das mathematische Kombinieren und Beobachten allein gestellt. Beobachtungen müssen jedoch interpretiert werden – auch gibt es ideologische Selektion – was nun beobachtet werden soll, und was nicht (manchmal kostet sie Milliarden … ) – und bei dieser Interpretation braucht es wieder das Denken, das Kant vorher leider über Bord geschnissen hat. Mit einem Satz: Physik ist ohne Philosophie und Erkenntistheorie weder begreifbar, noch überhaupt wirklich zu haben.

    Ein paar Sätze oben steht bei Gönner auch ein weiterer ketzerischer Satz:

    “Je nach Beobachtungssituation wird die kosmologische Konstante aus dem (Zauber-) Hut der Theoretiker gezogen oder wieder darin versteckt.” “Zauberhut” ist kein Zusatz von mir.

    Warum Gönner seine Reputation natürlich nicht riskiert und die Konklusio seines Büchleins in einem einzigen nahezu letzten Satz verpackt, erklärt Lee Smolin in dem Kapitel: “Wie Wissenschaft wirklich funktioniert” in der dt. Übersetzung seines bekannten Buches auf S. 446:

    “Gegenwärtig verhält es sich so, dass Menschen die anders sind – die aus dem ein oder anderen Grund Grund einflussreichen älteren Physikern ein Gefühl des Unbehagens einflößen – , nicht eingestellt werden.” So einfach erklärt er das.

    Zeilinger wird auch Stein und Bein auf Einstein schwören – und dennoch bei jedem Interview sagen, daß wir eine grundlegend andere Sicht auf Zeit und Raum brauchen.

    Gönner ist nicht der Einzige, der die logisch widersprüchliche Situation aussitzt und sicher nach bestem Gewissen verdrängt.

    Gewiss – in “Nature” würde z.B. sowas von einem weiteren Ihrer Kollegen nicht erscheinen, eher in dieser Art Bilder-Zeitung: die Aussagen sind dennoch nachvollziehbar:

    https://www.pm-magazin.de/r/natur/%C2%BBwarum-ist-der-urknall-ein-irrtum%C2%AB

    Da Sie ja offen sind, könnten Sie auch einmal von Kirchhoff lesen: “Räume, Dimensionen, Weltmodelle”. Daraus zitiert – was auch meiner Meinung entspricht:

    “Alles Relative bedarf des Absoluten. Das Relative kann nicht als universell gültig gesetzt oder postuliert werden, ohne daß verständlich gemacht wird, auf welchen absoluten Pol das Relative bezogen wird. Fehlt dieser absolute Pol, kollabiert auch die Relativitätsbehauptung. “Alles ist relativ” ist ein unsinniger Satz, weil er, wenn er wahr ist, sich selbst aufhebt.” (siehe oben, S. 269)

  132. #132 MartinB
    22. Juli 2012

    @MT
    “Gönner ist nicht der Einzige, der die logisch widersprüchliche Situation aussitzt und sicher nach bestem Gewissen verdrängt.”
    Wirklich mal wieder amüsant. Wenn Gönner etwas sagt, dass Ihnen in den Kram passt, dann ist ihm natürlich unbedingt zu folgen, wenn er andere Schlussfolgerungen zieht als Sie, dann liegt es natürlich nur daran, dass er (habe ich das richtig verstanden???) Angst um seinen Job hat (laut seiner Internetseite ist er 2002 “retired”???) und die Widersprüche (die nur ein genialer Name auf Verlangen entfernt sehen kann) aussitzt.

    Ihre Mischung aus Cherry-picking, quote mining und arrogance of ignorance (ich weiß gerade nicht, wie die Ausdrücke auf deutsch heißen) könnte man fast schon als Lehrbuchbeispiel nehmen – falls ich mal einen zweiten Teil des Trollhandbuchs schreibe “Wie man unsauber argumentiert” werde ich mich sicher hier bedienen.

  133. #133 SCHWAR_A
    22. Juli 2012

    @MartinB:
    “Nein, tun sie [Photonen] nicht [an Higgsfeld koppeln].”
    “Um aus einem masselosen ein massives Vektorboson zu machen, braucht man einen zusätzlichen Freiheitsgrad”

    Heißt dann also ‘an ein Higgsfeld koppeln’ automatisch, daß ein Vektorboson erzeugt werden muß, oder könnte eine Kopplung auch eine andere Wirkung haben, zB. daß lediglich die Vorwärts-Energie eines Photons im Higgsfeld abnimmt weil ein kleiner koppelnder Anteil reflektiert wird? So ein Photonen-Kopplungsfaktor könnte ja auch seeehr klein sein, zB. das Verhältnis VEV²/E_pl²…

    Herzliche Grüße.

  134. #134 Erik der Unlesbare
    22. Juli 2012

    @ Name auf Verlangen entfernt
    “Zeitiger wird auch Stein und Bein auf Einstein schwören – und dennoch bei jedem Interview sagen, daß wir eine grundlegend andere Sicht auf Zeit und Raum brauchen.”

    Am Ende des youtube-Beitrages kommt eine entscheidende Aussage von Zeilinger:  sinngemäss sagt er: Wenn sich die Wellenfunktion im Universum ausbreitet und sich auf Grund einer Messung die Wellenfunktion im gesamten Universum verändert, dann ist das schon ein herausfordernder Gedanke… – sein Lächeln dabei gibt eigentlich die aussagekräftigere Begründung für sein Scheitern und den Ruf nach einer neuen Sicht auf Zeit und Raum.

    “Dem Menschen ist es nicht erlaubt, in die Schöpfung Gottes einzugreifen!”
    Wer erteilt diese Erlaubnis?
    Wer erteilt die Erlaubnis, das Higgs verstehen zu dürfen? 
    Wer erteilt die Erlaubnis, das Higgs erklären zu dürfen?
    Ist es Gott selber der die Erlaubnis erteilt oder ist es ein absolutistisches Denken?
    Vor diesem Dilemma stehen die Teilchen-Charakter-Physiker und die Welle-Charakter-“Trolle”. Galileo hatte das gleiche Problem!

    Vor diesem Problem steht jeder, der mit dem Higgs, mit dem von Gott verdammten Teilchen, in Berührung kommt.

  135. #135 Name auf Verlangen entfernt
    22. Juli 2012

    @ Martin Bäker:“dann liegt es natürlich nur daran, dass er (habe ich das richtig verstanden???)” …

    Nein haben Sie nicht (und Sie wundern sich, wenn die Antworten knapp ausfallen?) – ich spreche nicht von “Job”, sondern “Reputation” – warum verdrehen Sie immer alles?! Wovon allerdings Smolin deutlich spricht, daß man in der theoretischen und praktischen Physik einfach keinen Job bekommt, wenn man oder frau nicht an Urknall oder Einstein glaubt.

    Wie soll ich Ihre Methode nennen? “Sich dumm stellen?” Nehmen Sie Studenten zur Doktorarbeit an, die an der Relativitätstheorie zweifeln? Das wissen die schon, wenn Sie bei Ihnen anfangen. Da sind Zweifel schnell verflogen. Smolin nennt das “positive Diskreminierung” – keine Ahnung, was daran positiv sein soll – es ist eine Meinungsdiktatur der Urknalltheoretiker & Relativisten im Auftrag des technischen Kapitals – wenn wir politisch werden wollten. Wenn den Relativisten & Urknalltheoretikern das keiner von außen sagt, sagt`s niemand. Da hat Frank Wappler schon irgendwie Recht. Gaddafi hat auch nicht geglaubt, daß er ein Tyrann ist – er hielt sich für den gnädigen Landesvater der in Libyen Wasser und Sozialhilfe organisierte. Wo ist Wappler überhaupt – habt ihr den gesperrt, oder hat er keine Lust mehr?

  136. #136 MartinB
    22. Juli 2012

    @SCHWAR_A
    ich weiß nicht mal, was eine “Vorwärts-Energie” sein soll, insofern kann ich die Frage nicht beantworten.
    Koplung an ein Higgsfeld bedeutet, dass es einen Wechselwirkungsterm gibt (oder ein feynmandiagramm mit einem passenden Vertex).
    Was da die Planck-Masse zu suchen haben soll, erschließt sich mir genau gar nicht.

  137. #137 MartinB
    22. Juli 2012

    @MT
    Sie haben das mit der Reputation so begründet:
    “”Gegenwärtig verhält es sich so, dass Menschen die anders sind – die aus dem ein oder anderen Grund Grund einflussreichen älteren Physikern ein Gefühl des Unbehagens einflößen – , nicht eingestellt werden.” So einfach erklärt er das.”
    Und siehe da, da steht etwas von “nicht eingestellt werden” – und nach dem was da steht, wäre Gönner ja wohl eher der “einflussreiche ältere Physiker”.
    Und Sie wundern sich, warum ich Sie nicht ernst nehme?

    “Nehmen Sie Studenten zur Doktorarbeit an, die an der Relativitätstheorie zweifeln? ”
    Ist für Materialwissenschaftler ziemlich irrelevant,insofern würde ich die Frage nicht stellen. Sollte jemand an der Existenz von Atomen zweifeln (die vergleichbar gut belegt sind wie die SRT), dann würde ich ihn nicht einstellen, nein.

    ” Smolin nennt das “positive Diskreminierung” – keine Ahnung, was daran positiv sein soll – es ist eine Meinungsdiktatur der Urknalltheoretiker & Relativisten im Auftrag des technischen Kapitals – wenn wir politisch werden wollten. ”
    Smolin (der selbst Relativist ist, um diese Bezeichnung zu übernehmen (und daran ändert es nichts, dass er die DSRT für eine interessante Alternative hält)) bezieht sich dabei natürlich auf die Stringtheorie, nicht auf die SRT – aber entstellen Sie ruhig weiter die Meinung anderer…

    Da ich den Rest des Tages besseres zu tun habe, werde ich auf weiter Kommentare von Ihnen nicht antworten.

  138. #138 SCHWAR_A
    22. Juli 2012

    @MartinB:
    …laß das Wort ‘Vorwärts-‘ einfach weg…
    und die Planck-Energie ist nur als Beispiel verwendet, damit der Photon-Higgsfeld-Kopplungswert schön klein wird.

    Also:

    Könnte eine Higgsfeld-Kopplung auch eine andere Wirkung haben, zB. daß lediglich die Energie eines Photons abnimmt, weil ein sehr kleiner koppelnder Anteil reflektiert wird?

    “Feynmandiagramm mit einem passenden Vertex”

    Gibt es denn keines, bei dem ein Photon in zwei aufgeteilt wird, eines davon mit fast derselben, aber etwas kleinerer Energie, das andere mit genau der Energie-Differenz?
    (Über die Richtung sagt ein FD ja nichts aus…)

    Herzliche Grüße.

  139. #139 MartinB
    22. Juli 2012

    @SCHWAR_A
    “Könnte eine Higgsfeld-Kopplung auch eine andere Wirkung haben, zB. daß lediglich die Energie eines Photons abnimmt, weil ein sehr kleiner koppelnder Anteil reflektiert wird?”
    Verstehe ich mal wieder nicht. Was meinst du mit “reflektiert”? Am Higgsfeld kannst du nichts reflektieren (sonst wäre die Impulserhaltung verletzt, denn das Higgsfeld ist ja selbst überall konstant).

    “Gibt es denn keines, bei dem ein Photon in zwei aufgeteilt wird, eines davon mit fast derselben, aber etwas kleinerer Energie, das andere mit genau der Energie-Differenz?”
    Nein, dazu müssten Photonen an Photonen koppeln, das tun sie aber nicht. Als prozess höherer Ordnung könnte man das konstruieren (Photon wird zu Elektron-Positron, Elektron gibt ein Photon ab, Elektron und Positron rekombinieren zu einem zweiten), aber als beobachtbaren Prozess gibt es das ncht, weil es Energie- und Impulserhaltung verletzen würde.

    @MT
    Damit ich Ihnen nicht Unrecht tue (habe gerade noch einmal nachgedacht):
    Wenn Sie fragen
    “”Nehmen Sie Studenten zur Doktorarbeit an, die an der Relativitätstheorie zweifeln? “”
    Was meinen Sie mit “zweifeln”? Das wird bei Ihren Äußerungen generell nie deutlich. Meinen Sie, dass Sie die Existenz von Phänomenen wie Zeitdilatation, Lorentz-Kontraktion etc. leugnen? Wenn ja, dann gibt es keine Hoffnung für Sie.

    Meinen Sie aber lediglich, dass Sie das Bild von der Raumzeit, so wie es die SRT zeichnet, nicht als der Weisheit letzter Schluss akzeptieren, sondern dass Raumzeit in Wahrheit etwas anderes ist und die SRT nur eine Näherung an dieses wahre Bild ist – dann ist das sicher denkbar und wird von vielen Physikern als Ansicht so geteilt. In dem Fall müssen Sie lediglich die Idee aufgeben, Sie seien ein besonderer “Wissenschaftskritiker” – denn diese Ansicht ist nicht ungewöhnlich und auch nicht physik-kritisch.

  140. #140 SCHWAR_A
    22. Juli 2012

    @MartinB:
    “sonst wäre die Impulserhaltung verletzt”

    Für jeden Punkt des Higgsfeldes ist die Impulssumme 0:
      Es trifft der Impuls p ein, der geht als p – λ·p weiter und es geht zusätzlich ein Impuls λ·p rückwärts, es kommt also quase ein Impuls -λ·p zusätzlich an.

    Damit trifft in Summe p – λ·p ein und es geht ab p – λ·p.

    Ich vermute mal, die Heisenberg’sche Unschärfe hilft hier bei der Betrachtung von Ursache und Wirkung…

    Herzliche Grüße.

  141. #141 MartinB
    22. Juli 2012

    @SCHWAR_A
    Wie üblich verstehe ich nicht, was du mir sagen willst, sobald du anfängst, formeln zu benutzen.

  142. #142 SCHWAR_A
    23. Juli 2012

    @Niels:
    ….Hiiiilfe…. verstehst Du denn, was ich meine? Wo drücke ich mich bloß so merkwürdig aus, daß es Martin nicht versteht?

    Vielen Dank im Voraus,
    Herzliche Grüße.

  143. #143 MartinB
    23. Juli 2012

    @SCHWAR_A
    Ich verstehe schon, dass du irgendwie Impuls auf das Higgsfeld übertragen willst um aus einem Photon zwei zu machen.
    Ich verstehe nur nicht, warum du das willst, da das Photon nun mal nicht ans Higgsfeld koppelt.

  144. #144 MartinB
    23. Juli 2012

    @MT
    Erfreulicherweise habe ich jetzt eine Kopie des Gönner-Buches.
    Falls es noch eines Belegs bedarf, dass Sie Zitate verdrehen, hier das Gönner-Zitat im Original
    ” Die Einsteinsche Gravitationstheorie zeigt denselben Mangel
    wie die Newtonsche: Auch in ihr treten keine Relativ-Größen
    wie etwa die Relativgeschwindigkeit auf, sondern auf das lo-
    kale metrische Feld bezogene Absolutgrößen.”
    Man beachte, dass da nicht “Einsteinsche Relativitätstheorie” steht sonder “Einsteinsche Gravitationstheorie” – kleiner aber feiner Unterschied (im Kontext geht es hier um die kosmologische Konstante, nicht um c als Absolutgeschwindigkeit).

    Zitate nicht nur aus dem Zusammenhang reißen, sondern sogar fälschen – das ist nun nicht mehr amüsant, sondern nur noch armselig.

  145. #145 Name auf Verlangen entfernt
    23. Juli 2012

    @ Ist ja gut – und wer dann auf dieser Banane ausrutscht, weiß schon wieder nicht, warum nur die ART (die übrigens der SRT widerspricht) eine Gravitationstheorie ist – als würde das im Ergebnis irgendwas ändern. Ich schlage vor, wir unterhalten uns weiter, wenn Sie mal wieder einen schönen Post über Physik und Geist machen.

    Guten Wochenstart!

  146. #146 MartinB
    23. Juli 2012

    “Ist ja gut ”
    Nein, ist es nicht. Zitate verfälschen ist unredlich. Mich weiter mit Ihnen zu unterhalten liegt nicht in meinem Interesse und ich denke gerade darüber nach, ob ich Sie angesichts dieser Aktion lieber gleich hier als Kommentator sperren sollte.

  147. #147 Name auf Verlangen entfernt
    23. Juli 2012

    Vielleicht lenken Sie mal nicht ständig von der Sache ab. Ich hab den Eindruck, eine Menge Leute mit spitzen Ohren können dieser einfachen Logik folgen.

  148. #148 vastehnix
    23. Juli 2012

    @MartinB
    Das war gar kein Troll-Versuch sondern eine Meinung, die gar nicht kommentiert werden wollte.

    Aber unabhängig davon eine hoffentlich trollfreie sachliche Frage:
    Inwieweit haben beim angeblich gelungenen messtechnischen Nachweis des Higgs-Bosons am CERN eigentlich “virtuelle Messkrücken” eine Rolle gespielt? Soll heißen: ein direkter Nachweis dürfte wohl kaum möglich sein. Also musste irgendwie mittelbar “gemessen” worden sein. Diese mittelbare “Messung” wiederum dürfte unter Zuhilfenahme von aufeinander aufbauenden Erkenntnissen/ Theoriegebäuden aus vorangegangenen mittelbaren “Messungen” erfolgt sein. Die Wahrscheinlichkeit, dass
    a) die Messtheorie an das Messziel angepasst wurde
    oder
    b) irgendwo in den vorangegangenen Messungen und den Messungen zu Grunde gelegten Theorien ein grober Fehler vorhanden ist (wahrscheinlich mit Ursache a) verknüpft)
    dürfte doch recht hoch sein. Wann kann das Higgs-Boson als “gefunden” gelten? Meiner Meinung nach frühestens erst dann, wenn mindestens eine weitere Messung unter Zuhilfenahme eines möglichst vollständig anderen Messprinzips zum gleichen Ergebnis führt. Bis dahin ist Skepsis doch ganz unabhängig von ästhetischen Erwägungen durchaus angebracht, gelle?

    Und dann noch Frage Nr.2: Wo genau findet sich ein Higgs-Teilchen örtlich betrachtet oder muss ich mir die Higgsteilchen als “Weltäther” vorstellen? Habe ich doch richtig verstanden: das Higgs-Boson hat eine große Ruhemasse und ist von einem “Feld” umgeben, wobei dieses “Feld” nicht von Teilchen gebildet wird, sondern ein echtes “Feld” ist. Das “Higgs-Feld” verleiht Elektronen Ruhemasse. Unter der Annahme, dass die Ruhemasse von Elektronen überall im Universum gleich groß ist, wäre zu schlussfolgern, dass die Higgs-Bosonen dann auch ganz gleichmäßig im Universum (als Theoriekrücke mal davon ausgehend es gibt nur eins) verteilt sind. Nun gibt es schlaue Leute, die jahrtehntelang behauptet haben (und immer noch behaupten), dass Universum würde sich mehr oder weniger gleichmäßig ausdehnen. In diesem Falle würden sich bei gleichmäßiger Verteilung der Higgs-Bosonen” auch die (mittleren) Abstände der Higgsteilchen vergrößern und damit auch die “mittlere Higgs-Feldstärke” und somit die Ruhemasse der Elektronen im Verlaufe der Zeit (Zeit im Sinne einer Theoriekrücke ausgehend von einem geradlinigem Verlauf der Zeit) immer kleiner werden. Habe ich denkbare Folgen der “verstanden?

    Danke vorab für die Antworten!

  149. #149 MartinB
    23. Juli 2012

    @vastehnix
    “Wann kann das Higgs-Boson als “gefunden” gelten? Meiner Meinung nach frühestens erst dann, wenn mindestens eine weitere Messung unter Zuhilfenahme eines möglichst vollständig anderen Messprinzips zum gleichen Ergebnis führt. Bis dahin ist Skepsis doch ganz unabhängig von ästhetischen Erwägungen durchaus angebracht, gelle?”
    tatsächlich eine gute Frage – da Unmengen von Daten anfallen, muss man natürlich immer filtern und diese Filterung würde eventuell vollkommen unerwartete prozesse einfach rausfiltern.
    Dass man relativ sicher sein kann, dass es sich um das Higgsteilchen handelt, liegt aber eben genau daran, dass man hier nahezu exakt (es gibt einige Abweichungen, die es zu klären gilt) die Zerfälle sieht, die man auch erwartet. Und wenn es aussieht wie ein Higgsund zerfällt wie ein Higgs, dann wird es vermutlich auch eins sein (was aber nicht ausschließt, dass morgen jemand eine geniale Theorie erfindet, in der es kein Higgsteilchen gibt, sondern ein anderes Teilchen mit genau den beobachteten Eigenschaften.)

    “Wo genau findet sich ein Higgs-Teilchen örtlich betrachtet oder muss ich mir die Higgsteilchen als “Weltäther” vorstellen? ”
    Das Higgsteilchen ist örtlich im Detektor bei CERN entstanden. Das Higgsfeld ist (analog zu einem Äther) überall im Universum.

    “In diesem Falle würden sich bei gleichmäßiger Verteilung der Higgs-Bosonen” auch die (mittleren) Abstände der Higgsteilchen vergrößern und damit auch die “mittlere Higgs-Feldstärke” und somit die Ruhemasse der Elektronen im Verlaufe der Zeit (Zeit im Sinne einer Theoriekrücke ausgehend von einem geradlinigem Verlauf der Zeit) immer kleiner werden.”
    Nein – das Higgsfeld ist keine Substanz, die immer dünner wird, wenn man das Universum weiter ausdehnt. Es hat (und behält) immer denselben Wert, unabhängig von der Expansion des Universums.

  150. #150 Dr. Webbaer
    23. Juli 2012

    Nein, ist es nicht. Zitate verfälschen ist unredlich.

    Kann man auch anders adjektivieren.

    MFG
    Dr. Webbaer (der’s geahnt hat)

  151. #151 SCHWAR_A
    23. Juli 2012

    @MartinB:
    Habe ich das (Kapitel 6.5 und 6.6) eigentlich richtig verstanden:
     Nach dem Standard-Modell mit Higgsfeld bleibt die Neutrino-Masse 0?

    OK, der Text ist schon ca. 4-5 Jahre alt…
    Hat sich seitdem denn ein Kopplungswert ≠0 im Neutrino-Elektron-Dublett eingefunden, damit Neutrinos Masse erhalten?

    Herzliche Grüße.

  152. #152 MartinB
    23. Juli 2012

    @SCHWAR_A
    Soweit ich das sehe, gibt es im Moment unterschiedliche Möglichkeiten, wie die Neutrinos Masse bekommen können – das Original-SM ist für masselose Neutrinos entwickelt. Guckst du hier:
    https://en.wikipedia.org/wiki/Standard_Model_%28mathematical_formulation%29#Including_neutrino_mass

  153. #153 Lavie
    23. Juli 2012

    “Wann kann das Higgs-Boson als “gefunden” gelten? Meiner Meinung nach frühestens erst dann, wenn mindestens eine weitere Messung unter Zuhilfenahme eines möglichst vollständig anderen Messprinzips zum gleichen Ergebnis führt. Bis dahin ist Skepsis doch ganz unabhängig von ästhetischen Erwägungen durchaus angebracht, gelle?”

    @vastehnix
    Genau deshalb wurden ja zwei Detektoren am LHC konstruiert, die dasselbe Ziel verfolgen. CMS und ATLAS sind ausserdem völlig unterschiedlich aufgebaut und haben unterschiedliche Auswertungsverfahren. Zudem werden die Daten vor einer Auswertung absichtlich verblindet, d.h. der Hintergrund wird nicht anders ausgewertet als ein mutmassliches Signal. Auch zwischen CMS und ATLAS werden keine Daten im Vorfeld ausgetauscht (bis auf die üblichen “illegalen” Blogeinträgen von einigen schwarzen Schafen der Kollaborationen).

    “da Unmengen von Daten anfallen, muss man natürlich immer filtern und diese Filterung würde eventuell vollkommen unerwartete prozesse einfach rausfiltern.”

    @MartinB
    Diese Filter (sog. Trigger) wurden ja genau darauf programmiert bereits bekannte Prozesse herauszufiltern. Dennoch könnten bereits bekannte Prozesse herausgefiltert werden, welche ein Teil eines unbekannten Prozesses sind. Ich denke, hier wurden im Vorfeld sicher lange Diskussionen um die “Maschengrösse” der Filter geführt. Einerseits stützt man sich dabei auf das Standardmodell und andererseits auf Theorien die darüber hinausgehen. Eine Gratwanderung.

  154. #154 Lavie
    23. Juli 2012

    “Vielleicht lenken Sie mal nicht ständig von der Sache ab. Ich hab den Eindruck, eine Menge Leute mit spitzen Ohren können dieser einfachen Logik folgen.”

    @Name auf Verlangen entfernt
    Hihi. Einzig interessant für Leute mit spitzen Ohren sind Ihre kleinen Ablenkungsmanöver. Das sind nichts weiter als die typischen Phrasen eines Trolls nachdem ihn jemand etwas in die Ecke gedrängt hat.
    Sie haben noch nicht mal die Grösse zuzugeben, dass Sie hier absichtlich falsche Zitate zum Besten geben. Typisch Troll.

  155. #155 Name auf Verlangen entfernt
    23. Juli 2012

    @ Dr. W.B.: sehn Sie, da haben Sie´s, wie die Kinder freuen Sie sich jetzt, weil sie plötzlich nicht wissen wollen, dass die Allgemeine Relativitätstheorie für Gravitationstheorie steht – als wär die Ofenklappe zu. Ich ich soll jetzt die Rolle des Physik-Lehrers übernehmen – weil ich ein Wort vertauscht habe (vollkommen ohne Arglist) – das ohnehin in der Theorie synonym ist:

    Der versehentliche Satz:

    “Die Einsteinsche Relativitätstheorie zeigt denselben Mangel wie die Newtonsche: auch in ihr treten keine Relativ-größen, wie etwa die Relativgeschwindigkeit auf, sondern auf das lokale Feld bezogene Absolutgrößen.”

    hat denselben Inhalt bezogen auf Absolutgrößen, wie:

    “Die Einsteinsche Gravitationstheorie zeigt denselben Mangel wie die Newtonsche: auch in ihr treten keine Relativ-größen, wie etwa die Relativgeschwindigkeit auf, sondern auf das lokale Feld bezogene Absolutgrößen.”

    Nur darauf, mit Verlaub, kommt es ja wohl an. Die Absolutgrößen tauchen auch – um vollends topic zu werden, überall auf – hier zum Beispiel:

    Nein – das Higgsfeld ist keine Substanz, die immer dünner wird, wenn man das Universum weiter ausdehnt. Es hat (und behält) immer denselben Wert, unabhängig von der Expansion des Universums.”

    Kein Wunder, denn sie sind ja die Grundlage der Theorie, die ihrem Resultat grundlegend widerspricht.

  156. #156 MartinB
    23. Juli 2012

    @Lavie
    Soweit ich weiß, werden aber auch Ereignisse herausgefiltert, die eben nicht dem Suchmuster entsprechen. Ein vollkommen ungewöhnlicher und von niemandem voerhegeseher Prozess würde deshalb womöglich übersehen werden. Jedenfalls entspricht das meiner düsteren Erinnerung an das, was ich seinerzeit am DESY im Beschleunigerseminar (ich glaube es war das ZEUS-Seminar, ist lange her) gelernt habe.

  157. #157 junia
    25. Juli 2012

    sacht mir Bescheid, wenn das Anti-Higgs-Feld entdeckt wird. Dann schwimm ich da mal durch – zum Abnehmen. Ein paar Kilos weniger wäre echt super für mich.

  158. #158 MartinB
    26. Juli 2012

    @junia
    Pech gehabt, das Higgs ist erstens sein eigenes Antiteilchen und zweitens kommt der Großteil unserer Masse aus der Bindungsenergie der Protonen und Neutronen. Schluck lieber ein paar Kilo Antimaterie, die zerstrahlt deine überschüssigen Pfunde sofort 😉

  159. #159 vastehnix
    26. Juli 2012

    @MartinB
    Danke für die Antworten.

    Kann man das Postulat der immer gleichen Stärke des Higgsfeldes unabhängig vom eingenommenen Raum genauer erklären? Wenn das Feld durch massebehaftete Teilchen (Higgsbosonen) erzeugt wird, erschließt es sich zumindest mir nicht, wie das geht.

    Oder ist es so zu verstehen, dass zu einem Elektron stets auch ein (meinetwegen in einer Wahrscheinlichkeitswolke oder was auch immer) zuzuordnendes Higgsboson nebst des allein durch dieses erzeugten Feldes vorhanden ist?

  160. #160 vastehnix
    26. Juli 2012

    Und noch eine Frage:
    Was geschieht eigentlich mit dem Higgsboson/ Higgsfeld bei Umwandlung Elektron in Neutrino und umgekehrt?

  161. #161 MartinB
    26. Juli 2012

    @vastehnix
    “Wenn das Feld durch massebehaftete Teilchen (Higgsbosonen) erzeugt wird”
    Wird es ja nicht. Das Higgsboson ist eine Anregung des feldes, das Feld selbst ist überall vorhanden.

    “Was geschieht eigentlich mit dem Higgsboson/ Higgsfeld bei Umwandlung Elektron in Neutrino und umgekehrt?”
    Nicht,s warum sollte es. Das Elektron koppelt ans feld, das Neutrino nicht (oder nur sehr wenig), aber das ist dem Feld ziemlich egal.

  162. #162 SCHWAR_A
    27. Juli 2012

    @vastehnix & MartinB:
    “Oder ist es so zu verstehen, dass zu einem Elektron stets auch ein (meinetwegen in einer Wahrscheinlichkeitswolke oder was auch immer) zuzuordnendes Higgsboson nebst des allein durch dieses erzeugten Feldes vorhanden ist?”

    Haben nicht letztlich alle elementaren, also nicht zusammengesetzten Partikel im Kern eine Higgs-Anregung? Das darauf aufgesetzte, eigentliche Teilchen ist dann eine Art Resonanz zur Higgs-Anregung?

    Herzliche Grüße.

  163. #163 MartinB
    27. Juli 2012

    @SCHWAR_A
    “Das darauf aufgesetzte, eigentliche Teilchen ist dann eine Art Resonanz zur Higgs-Anregung?”
    Nein. Das hat mit Resonanz und so nichts zu tun. Das Quantenfeld eines massebehafteten Teilchens interagiert mit dem Higgsfeld so, dass im Propagator ein Term entsteht, der so aussieht wie ein Masseterm. Das ist alles.

  164. #164 SCHWAR_A
    27. Juli 2012

    @MartinB:
    “…interagiert mit dem Higgsfeld so…”

    ‘interagiert’ bedeutet doch eine Wechselwirkung, einen zeitlich-räumlichen Ablauf im Feld, bei dem das passiert, was man dann als Masseterm in einer Gleichung wiederfindet.
    Aber was passiert da tatsächlich? Es muß doch ein (salopp ausgedrückt) Energie-Hin-und-Her für eine bestimmte Zeit (sogar für immer bei stabilen Teilchen) erfolgen, oder nicht?

    Herzliche Grüße.

  165. #165 MartinB
    27. Juli 2012

    @SCHWAR_A
    “Aber was passiert da tatsächlich? ”
    Was da “tatsächlich passiert” hängt davon ab, was du damit meinst.
    Wenn du meinst, welches Modell-Bild ist dafür angemessen, dann hilft vermutlich das feynman-Diagramm von oben weiter. Du musst aber berücksichtigen, dass du über sämtliche feynmandiagramme bis zur unendlichsten Ordnung summieren musst,um das richtige Endergebnis zu bekommen.
    Aber du kannst es auch einfach mathematisch sehen: Der We-Wi-Term beeinflusst den propagator genauso wie ein Masseterm.
    Warum da aber Energie “fließen” soll, sehe ich nicht.

  166. #166 SCHWAR_A
    15. August 2012

    @MartinB:
    “Warum da aber Energie “fließen” soll, sehe ich nicht.”

    Ein Teilchen regt das Higgsfeld an, es “stört” das Higgsfeld.
    Jede “normale” Störung in einem Feld breitet sich einfach in alle Richtungen aus und “verschwindet” quasi.
    Bei stabilen Teilchen ist das aber nicht der Fall. Deren “Störung”, also ein Energieaufwand, verliert sich nicht; es sieht so aus, als ob die Störung über ein ganz bestimmtes Gebiet (Kugel→Querschnitt) verteilt ist und bleibt. Die dafür nötige Energie bleibt “gespeichert”. Da sich die Energie einerseits in alle Richtungen “verdünnisieren” will, ist dafür wohl ein Rückwärts-Energiefluß nötig – letztlich führen beide Energieflüsse überlagert zu dem “statischen” Energiefeld, das das Teilchen selbst ausmacht.

    Kann man das so sehen?

    Herzliche Grüße.

  167. #167 MartinB
    16. August 2012

    @SCHWAR_A
    Nein, ein Teilchen stört das Higgsfeld nicht. Da breitet sich auch nichts aus. Deine Anschauung ist soweit ich sehen kann, einfach fehlerhaft, deswegen kommst du immer zu falschen Schlüssen.
    Denk dir ein ruhendes massebehaftetes Teilchen. Diese Situation ist lorentz-invariant – jede Beschreibung der Masse, bei der sich irgendetwas verändert (aisbreitet, fließt was auch immer) muss deswegen falsch sein. Versuch doch mal, deine Anschauung an diesem Beispiel zu schärfen.

  168. #168 SCHWAR_A
    16. August 2012

    @MartinB:
    ” jede Beschreibung der Masse, bei der sich irgendetwas verändert…”

    Im Endeffekt ändert sich bei meinem Bild ja nichts. Es herrscht ein statisches Gleichgewicht, zumindest für stabile Teilchen.

    “ein Teilchen stört das Higgsfeld nicht.”

    Dann muß ich wohl was falsch verstanden haben, zB. von hier: “Um Teilchen Masse zu geben wird ein Hintergrund-Feld eingeführt, das lokal gestört wird, jedesmal wenn sich ein Teilchen hindurchbewegt. Die Störung – d.h. die Cluster-Bildung des Feldes um das Teilchen – erzeugt die Masse des Teilchens.”

    Herzliche Grüße.

  169. #169 MartinB
    18. August 2012

    @SCHWAR_A
    Die Masse der Teilchen ist direkt proportional zum Vakuumerwartungswert des Higgsfeldes. Die Teilchen koppeln also an das Higgsfeld, aber so weit ich sehen kann, ist (zumindest in niedrigster Ordnung) eine Änderung des Higgsfeldes für die Massenerzeugung nicht notwendig. Insofern halte ich die Internetseite, die du zitierst, in der Hinsicht für problematisch – wenn da eine Clusterbildung stattfände, dann müsste meiner Ansicht nach das entsprechende Feynmandiagramm eine Higgs-Higgs-Kopplung oder eine Vakluumschleife oder so etwas enthalten.

  170. #170 SCHWAR_A
    18. August 2012

    @MartinB:
    “…dann müsste meiner Ansicht nach das entsprechende Feynmandiagramm eine Higgs-Higgs-Kopplung oder eine Vakuumschleife oder so etwas enthalten.”

    Ergäbe “so etwas” denn einen Widerspruch zu den existierenden Beobachtungen?
    Falls nein, müßte dieser Fall bereits irgendwo ein Bestandteil der Analysen sein.
    Oder aber, ihr Effekt ist so klein, daß ihr Beitrag unerheblich und damit immer vernachlässigt wird. Eigentlich sollte er dann trotzdem in den Analysen erwähnt sein…

    Herzliche Grüße.

  171. #171 MartinB
    18. August 2012

    @SCHWAR_A
    Naja, das einzubauen ist wenn ich es richtig sehe eine Frage der Renormierungstheorie (man müsste ja überlegen, wie irgendwelche schleifen etc. die “nackten” und die “physikalischen” Konstanten beeinflussen) – ich vermute sehr stark, dass man das irgendwo nachlesen kann, habe aber im Moment nichts parat. Soweit ich micht entsinne, war der Beweis, dass die elektroschwache Theorie mit dem Higgsmechanismus renormierbar ist, ein ziemlich kniffliges Stück arbeit.

  172. #172 SCHWAR_A
    18. August 2012

    @MartinB:

    …das hört sich so an, als wäre die “Störung des Higgsfeldes durch ein Teilchen” bereits eingearbeitet…
    …und die von mir oben erwähnte gespeicherte “Störungs”-Energie ist bereits eingebaut…

    “ist…eine Frage der Renormierungstheorie…, wie irgendwelche Schleifen etc….die “physikalischen” Konstanten beeinflussen”

    Meinst Du damit die Beschreibung der Selbstwechselwirkung durch die ß-Funktion, bei der die QCD-Kopplung immer mehr abnimmt, je energiereicher der Teilchen-Beschuß wird?

    Herzliche Grüße.

  173. #173 MartinB
    18. August 2012

    @SCHWAR_A
    “und die von mir oben erwähnte gespeicherte “Störungs”-Energie ist bereits eingebaut…”
    Nein, da ist keine Störungsenergie (die würde nach der Logik mit-wegrenormiert). Warum beharrst du eigentlich so auf dieser Energie?

    Und mit QCD hat das nichts zu tun – Renormierung ist in jeder QFT notwendig, damit keine Unendlichkeiten rauskommen. und am Ende ales stimmt. Irgendwann schreibe ich auch noch mal was in der QFT-Serie dazu.

  174. #174 SCHWAR_A
    18. August 2012

    @MartinB:
    “die würde nach der Logik mit-wegrenormiert”

    Also kann sie existieren, verschwindet aber mathematisch quasi als Offset, um kein Unendlich zu erhalten, richtig?

    Herzliche Grüße.

  175. #175 MartinB
    18. August 2012

    @SCHWAR_A
    Nein, diese Energie existiert nicht, jedenfalls nicht so, dass da etwas fließen würde – kann nicht, wegen der Lorentzinvarianz (wo sollte denn das hinfließen?).
    Die Störung des Higgs-feldes, die du meinst, würde meiner Ansicht nach wegrenormiert werden.

  176. #176 SCHWAR_A
    19. August 2012

    @MartinB:
    “wegen der Lorentzinvarianz”

    ????
    Wieso soll die betroffen sein? Wenn es einen gleichmäßigen Energiefluß in die eine Richtung und einen gleichmäßigen zurück gibt, dann ergibt das insgesamt doch eine statische Energie-Verteilung, bei einem Teilchen eine kugelsymmetrische. Ist die denn nicht Lorentz-invariant? Wieso nicht?

    Herzliche Grüße.

  177. #177 MartinB
    19. August 2012

    @SCHWAR_A
    Verstehe ich nicht – es gibt bei dir gleichzeitig einen Energiefluss von A nach B und von B nach A – also netto eben keinen Energiefluss? Warum dann überhaupt einen Energiefluss postulieren, wenn sich am Ende alle Energieflüsse aufheben?

  178. #178 SCHWAR_A
    19. August 2012

    @MartinB:
    “Warum dann überhaupt einen Energiefluss postulieren, wenn sich am Ende alle Energieflüsse aufheben?”

    …und jetzt sind wir wieder hier

    Die Interaktion eines Teilchens mit dem Higgsfeld findet doch auch dann statt, wenn sich das Teilchen bzgl. des Beobachters nicht bewegt. Es existiert also eine permanente Störung des teilchenlokalen Higgsfeldes. Sowas stelle ich mir vor, geht nicht ohne Energiefluß. Daher schließe ich, daß der eine Energiestrom aus dem Teilchenzentrum nach außen fließt, mit dem Higgsfeld interagiert, durch das ständige Erzeugen-Vernichten (Selbstkopplung) peu-á-peu reflektiert wird und als anderer Energiestrom in der Summe aller Reflektion wieder im Zentrum fokussiert (mit konstruktiver Interferenz zur aktuellen Phasenlage dort). Die Energie ist quasi gespeichert.

    Ein unbeschleunigtes Teilchen bildet so ein statisches kugelsymmetrisches Energiefeld — bis man es beeinflußt und dadurch die beiden konstruierenden Energie-flüsse sichtbar, sprich meßbar macht.

    Ein unbeschleunigtes Teilchen hat auf diese Weise trotzdem Masse – messen kann man die aber erst, wenn man das Teilchen beschleunigt, weil dadurch das Feld von außen in seiner Symmetrie gestört wird: die beiden Energieflüsse heben sich dann nicht mehr komplett auf und das Teichen “stemmt” sich gegen die Beschleunigung von außen → Trägheit.

    Herzliche Grüße.

  179. #179 MartinB
    19. August 2012

    “Es existiert also eine permanente Störung des teilchenlokalen Higgsfeldes. Sowas stelle ich mir vor, geht nicht ohne Energiefluß.”
    Nein, es existiert keine “Störung” (siehe oben), und nein, nicht jede physikalische Wechselwirkung erfordert einen Energiefluss.

    Wenn du ernsthaft QFT betreiben willst, dann ist es wirklich nicht hilfreich, dir erst viele Anschauungen zu bauen, die alle nicht ganz stimmen – schau auf dei Formeln für den Massenterm und überlegen, ob da irgendwo kugelsymmetrische Energieflüsse oder so drin stecken. Wenn sie das nicht tun, dann passt deine Anschauung einfach nicht.

    “messen kann man die aber erst, wenn man das Teilchen beschleunigt”
    Nö, man könnte ja auch die Gravitation messen.

  180. #180 SCHWAR_A
    19. August 2012

    @MartinB;
    “Nö, man könnte ja auch die Gravitation messen.”

    Ein Testteilchen im Energiefeld meines Modells besitzt sein eigenes Energiefeld nach dem selben Modell.
    Beide erzeugen gegenseitig eine Überlagerung, beide Energiefelder verformen sich entsprechend und auf der einander zugewandten Seite sind die jeweiligen Rückflüsse nicht mehr so stark wie auf der abgewandten Seite.
    (Genau zwischen ihnen, bei gleichen Teilchenmassen, ist der Netto-Rückfluß sogar Null, also “Schwerelosigkeit”, wenn man ohne Störung messen könnte.)
    Dadurch erfahren beide Teilchen einen stärkeren Rückfluß aus der jeweils abgewandten Seite und werden aufeinander zu beschleunigt.
    Wieder eine Beschleunigung und daher kann die Masse gemessen werden.

    “nicht jede physikalische Wechselwirkung erfordert einen Energiefluss.”

    Mir fällt leider keine ein – an welche physikalische WW denkst Du dabei?

    Herzliche Grüße.

  181. #181 MartinB
    19. August 2012

    Nachtrag, habe nochmal nachgedacht:
    1. Dein Bild mit dem “Teilchen” krankt zunächst daran, dass wir es mit Quantenfeldern zu tun haben. Wenn du dir ein Teilchen wie in einem Feynman-Diagramm vorstellst, dann musst du im geiste immer über alle Möglichkeiten summieren. (Dein Bild mit dem Energiefluss stimmt aber auch dann nicht, s.u.)
    2. Durch den Higgsmechanismus wird die Masse in der Lagrangedichte durch den Vakuumerwartungswert des Higgsfeldes (HF) und die Kopplung an das HF ersetzt. Da das Terme in der Lagrangedichte ohne jede Ableitung sind, siehst du schon daran, dass hier zunächst nichts fließt – die Terme sind lokal.
    3. Auch wenn das Bild oben was anderes suggeriert – wenn ich mich nicht total irre, dann wird an dem HF-Vertex in dem Diagramm kein Impuls vom Elektron auf das HF übertragen – ebenso wie bei einem normalen massenterm auch kein Impuls übertragen wird (wohin auch).

  182. #182 MartinB
    19. August 2012

    Achtung, die mails haben sich überschnitten.

    “Mir fällt leider keine ein – an welche physikalische WW denkst Du dabei?”
    Ein Planet kreist um die Sonne z.B. – potentielle und kinetische Energie bleiben die ganze zeit konstant, kein Energiefluss. Ein Elektron kreist um den Atomkern – kein Energiefluss. usw usw.

  183. #183 SCHWAR_A
    21. August 2012

    @MartinB:
    “dass hier zunächst nichts fließt”

    Besser wäre “…NETTO nichts fließt”.

    “…wird … kein Impuls vom Elektron auf das HF übertragen”

    “kein NETTO-Impuls” wäre in meinem Modell der Fall, weil ja in Vorwärtsrichtung p-Δp abfließt und rückwärts -Δp dem ankommenden p überlagert wird…

    “…kein Impuls übertragen wird (wohin auch)”

    Dieses “wohin auch” ist genau das, warum es in meinem Modell den Energie-Rückfluß gibt. Die Suggestion im Bild oben erhält so eine echte physikalische Grundlage…

    “Ein Planet kreist um die Sonne”

    Hierbei “fällt” das Testteilchen um das zu vermessende Teilchen herum. Einer linearen tangentialen Vorwärtbewegung überlagert sich eine Beschleunigung.
    In dieser Wechselwirkung fließt NETTO-Energie, was man auch daran sieht, daß der Bahnradius immer kleiner wird — für uns leider erst bei riesigen Massen meßbar, aber dennoch ist der Effekt selbst auch bei kleinsten Massen vorhanden.

    Herzliche Grüße.

  184. #184 MartinB
    21. August 2012

    @SCHWAR_A
    “In dieser Wechselwirkung fließt NETTO-Energie, was man auch daran sieht, daß der Bahnradius immer kleiner wird ”
    Nein, ein zwei-Körper-problem (mit Punktmassen) ist stabil, da ändern sich die Bahnradien nicht (wäre auch schlecht für die erde). Was du da für energieflüsse siehst hat was it gezeiten/Reibungskräften zu tun, die sind im Erde-Sonne-System aber extrem klein.

    Bitte nicht böse sein, aber ich glaube, dass dein Physik-Verständnis an einigen Stellen böse Lücken hast und dass du dazu neigst, die Physik an deine vorgefassten Bilder anzupassen, statt umgekehrt.

  185. #185 Niels
    21. August 2012

    @SCHWAR_A
    Du gehst meiner Ansicht nach den falschen Weg.

    Bevor man für sich selbst eine wirklich tiefe Veranschaulichung eines physikalischen Sachverhaltes entwickeln kann, muss man leider zuerst den mathematischen Formalismus vollständig verstanden haben.

    Als “Abkürzung” kann man natürlich einfach die Veranschaulichungen echter Experten übernehmen.
    Das reicht im Allgemeinen auch völlig aus.
    Aber wenn man nicht selbst verstanden hat, wie diese Veranschaulichungen mit den Formeln zusammenhängen, sind eigene, weitreichende Schlüsse aus diesen “Bildern” mit größter Wahrscheinlichkeit falsch.
    Die “Sprache” der Physik ist nun mal die Mathematik, alle “Übersetungen” in die “Alltagssprache” sind prinzipiell immer mit Fehlern behaftet.

    Für ein echtes, tiefes Verständnis des Higgs-Mechanismus musst du dich mit der echten Physik und der zugrunde liegenden Mathematik auseinandersetzen.
    Da führt leider kein Weg dran vorbei.

  186. #186 MartinB
    21. August 2012

    @Niels
    ich denke, das größere problem ist, dass unterschiedliche Experten unterschiedliche veranschaulichungen verwenden, um Dinge auf dem jeweils angemessenen Niveau zu erklären. Das war ja auch der Kern des Textes oben. Diese Modelle tragen alle unterschiedlich weit und widersprechen sich zum teil (wie z.B. das Honig-Bild). Deswegen scheitert man zwangsläufig, wenn man versucht, sich seine Anschauung aus verschiedenen solcher Modelle zusammenzubasteln, und das ist genau das, was SCHWAR_A meiner Ansicht nach passiert.

  187. #187 Niels
    21. August 2012

    @MartinB
    Stimmt, das ist wirklich das Hauptproblem.

  188. #188 MartinB
    21. August 2012

    @Niels
    Ich glaube, darüber sollte ich mal etwas nachdenken und dann einen text dazu schreiben – ich vermute, da gibt es einen Zusammenhang damit, dass man als Laie immer und immer wieder Bücher z.B. über Quanten oder die SRT lesen kann und mag (ging mir jedenfalls vorm Studium so), immer in der Hoffnung, mehr zu verstehen…. grübel…

  189. #189 Niels
    22. August 2012

    @MartinB
    Mir gings vorm Studium jedenfalls auch so.
    Ein Zusammenhang erscheint mir auf auf den ersten Blick auch irgendwie logisch.
    Das ist mir aber bisher noch nie in den Sinn gekommen und darüber hab ich soweit ich mich erinnere auch noch nie etwas gelesen. Mir fällt spontan auch nichts Tiefgründiges dazu ein.

    Da könnte aber durchaus was dran sein, eine Untersuchung lohnt sich vielleicht.
    Wenn du da etwas im Stil und in der Qualität von “Denken in Modellen”, “Ist die Natur mathematisch?” oder “Kann die Physik die Welt erklären?” hinkriegst, wird das sicher extrem lesenswert.

    Bin mir auch nicht sicher, ob das Thema so viel hergibt…

  190. #190 MartinB
    22. August 2012

    @Niels
    Nein, sicher bin ich mir auch nicht. Also mach ich’s wie immer bei solchen Themen – ein paar Tage lang im Hinterkopf behalten und das Unterbewusstsein arbeiten lassen, und dann lostippen und sehen, ob was sinnvolles bei rauskommt. Bisher hat die Methode erstaunlich gut funktioniert.

  191. #191 SCHWAR_A
    28. August 2012

    @MartinB:
    hier hat Ethan Siegel gerade ein sehr ähnliches Thema angerissen:
    frei übersetzt:

    “Wie sag’ ich’s verständlich…”

    …oder, wie Feynman es einst sagte:
    “Erst, wenn Du es in normaler Sprache ‘rüberbringen kannst, hast Du das Thema wirklich verstanden”

    Herzliche Grüße.

  192. #192 MartinB
    28. August 2012

    @SCHWAR_A
    Ich lese Ethan Siegel ganz gern, aber manchmal vereinfacht er in meinen Augen zu stark. Manchmal ist “Jargon” auch vollkommen o.k. – niemand würde sich darüber aufregen, dass Fussballfans das Wort “Abseits” benutzen, auch wenn es jargon ist. Man muss die Fachbegriffe halt erklären, aber manchmal geht es nicht ohne.

  193. #193 Stephan Goldammer
    https://cspannagel.wordpress.com/2013/04/22/gastbeitrag-haben-schlechte-vorlesungen-eine-zukunft/
    10. Oktober 2013

    “Das ist eins von vielen Beispielen für die oft furchtbar schlechte Didaktik der Physik.”

    Was halten Sie in dem Zusammenhang von der recht hitzigen/emotionalen Debatte zwischen der DPG und dem sogenannten Karlsruher Physikkurs?

    https://www.physikdidaktik.uni-karlsruhe.de/kpk/Fragen_Kritik/DPG.html

    Eine der aufgeworfenen Fragen: Wer bestimmt was “richtige”, “gute”, “anschauliche” Erklärungen im Physikunterricht sind. Bestimmen das DPG-Gutachter?

  194. #194 MartinB
    10. Oktober 2013

    @StephanGoldammer
    Nach dem, was ich bisher vom Karlsruher Physikkurs gesehen habe, finde ich ihn schlicht und einfach furchtbar. Keine praktizierende Physikerin denkt so, wie dieser Kurs annimmt, und Größen wie “Impulsstromdichten” (oder waren das jetzt “stärken”) einzuführen, wenn man einfach Kräfte meint, ist in meinen Augen absurd. Das otwendige abstrakte Vorstellungsvermögen, um immer zwischen einer Dichte und einer Stärke zu unterscheiden, haben Achtklässler meist noch gar nicht, dann sind das einfach nur Begriffe, die man auswendig lernt.

    Ich glaube, so etwas entsteht, wenn jemand nach langem Nachdenken plötzlich merkt, dass ein anschauliches Konzept für ihn der Schlüssel zum Verständnis ist – man glaubt dann, dass man nur dieses eine Konzept beibringen muss und alle würden es genauso verstehen. Was Ähnliches habe ich mal in einem Elektrodynamik-Kurs gesehen, wo alles nur im Formalismus der Differentialformen beigebracht wurde. Auch axiomatische Thermodynamik geht in die Richtung (es gibt da ein Vorlesungsskript in dem so etwa steht “Die Entropie ist eine extensive Zustandgröße, über deren anschauliche Bedeutung wir uns keine Gedanken machen”.)
    Wenn man den Stoff schon verstanden hat, dann sind solche vereinheitlichenden Konzepte oft hilfreich, aber um Stoff beizubringen, halte ich das für verfehlt.

    Was der Physikunterricht (abgesehen von mehr Stunden) meiner Ansicht nach braucht, ist eine bessere Fundierung in anschaulichen Modellen; dafür kann man sich ein paar überflüssige Experimente sparen. (Was man nicht braucht sind 6 Monate Energieumwandlungsketten ohne zu erklären, wie man sich die Energieformen eigentlich vorstellen kann – zur Zeit lernt man ja in Klasse 7 laut Lehrplan erst den Energiebegriff und dann den Kraftbegriff. Und keiner sagt den Kleinen, dass niemand weiß, was Energie ist.)

    Aber das wäre ein Thema, über das man lange diskutieren kann (was ich gelegentlich auch tue, da ich eine Physik-Referendarin gut kenne…)

  195. #195 RJ
    18. Februar 2019

    Erstmal vielen Dank für das Blog und die Serien – ich habe diese Site erst kürzlich gefunden und schon einiges gelesen (allerdings mehr noch nicht) – vieles wird klarer, vielen Dank nochmal!

    Manche Grundsatzfragen erschließen sich mir aber weiterhin nicht. (Und ich meine anschaulich – die involvierte Mathematik ist mir ohnehin ein paar Nummern zu hoch.)

    Mit dem Higgs-Feld habe ich auch noch meine Schwierigkeiten und keines der Erklärungsmodelle, die ich bisher gesehen habe, helfen mir viel weiter.
    Es soll ja Teilchen Masse verleihen. Wir reden über träge Masse, richtig? (Ja, ich habe den Artikel über die verschiedenen Gesichter der Masse gelesen.)

    Wenn ich mich aus der Schule (lange her) richtig erinnere, kann man die träge Masse beschreiben als Widerstand gegen eine Änderung des Bewegungszustandes. (Genauer Änderung der Geschwindigkeit, ergo Beschleunigung.)

    Sämtliche angeführten Beispiele beschreiben aber – wenn ich sie nicht völlig mißverstehe – einen Widerstand gegen Bewegung an sich (Geschwindigkeit), also eine Dämpfung. Wenn eine prominente Person in einer Menschenmenge gestoppt wird, oder eine Kugel in Honig, ändert sich ja dabe gerade der Bewegungszustand, sie wird nämlich zum Stillstand gebracht. Und dazu verzögert, sprich hier wird eben nicht die Masse beschrieben, sondern die Dämpfung.

    Womöglich mißverstehe ich auch die physikalische Erklärung mit der Überlagerung der verschiedenen Wege, aber auch darin sehe ich letztlich einen Widerstand gegen die Bewegung (das Teilchen braucht länger, um von a nach b zu kommen) und nicht gegen seine Beschleunigung; schon gar nicht gegen Verzögerung. Oder was habe ich da falsch verstanden?
    (Ja, ich habe die Serie über das Teilchen von A nach B gelesen. Womöglich nicht alles im Detail verstanden, aber die Grundsätze, dachte ich, schon.)

    Was ich mir als Verdeutlichung des Beschleunigungsthemas vorstellen könnte, wäre: wenn die prominente Person sich umringt von ihrer Entourage bewegt, dann ist sie sicher schwerer zu verzögern oder zu beschleunigen als eine einzelne Person, weil eben viel mehr “dranhängt”. Das wäre schon ein Analogon zur Masse, aber das ist auch schon rein per Newton klar, weil es mehr Leute sind mit mehr Masse, und einen Zusammenhang mit einem Higgs-analogen Feld sehe ich da nicht.

    Masse ist ja äquivalent zu Energie, richtig? Also könnte man auch sagen, daß Fermionen ihre Energie aus dem Higgs-Feld beziehen?
    Nur in dem Modell haben sie ja keine Ruhemasse und bewegen sich mit Lichtgeschwindigkeit, also über was für eine Energie reden wir dann hier überhaupt? Wie sieht die denn zum Beispiel aus für ein Elektron, das sich in Ruhe befindet? (Das geht ja eigentlich gar nicht nach dem Modell, also was passiert dann da?) Oder ein sich linear bewegendes, auf das eine Kraft entgegen seiner Bewegungsrichtung einwirkt – wie erzeugt das Higgs-Feld da das Beharrungsvermögen (die Massenträgheit)?

    In Deinem langen Artikel hattest Du ja von Feynman geschrieben und seinen Leitsätzen, Zum Beispiel, daß man etwas nur dann vollständig verstanden hat, wenn man es einem Anfänger erklären kann. Ich denke, das stimmt. Heißt das aber, daß niemand (zumindest niemand, der/die sich öffentlich an Erklärungen versucht) das Higgs-Feld bisher wirklich verstanden hat?

    Auch zu dem langen Artikel und den Stufen des Verständnisses: ich bin der Ansicht, daß man Zusammenhänge qualitativ (ob man das nun intuitiv nennen mag oder nicht) verstehen kann, auch ohne die Mathematik dazu zu durchdringen.

    Letztlich ist Mathematik ja auch “nur” ein Vehikel zur Erklärung von Sachverhalten, wenn auch ein sehr präzises (und nützliches), mit dem man quantitative Aussagen machen kann. Wenn man das Modell grundsätzlich richtig verstanden hat, sollte man aber auch ohne die Mathematik korrekte qualitative Aussagen über das Modellverhalten machen können. (Die sich dann mathematisch oder/und experimentell nachvollziehen und präzisieren lassen. Wenn das nicht der Fall ist, hat man’s halt doch (noch) nicht verstanden.) Denke ich.

    Nur wo bekommt man dieses Verständnis her? Deine Erklärung ist einen Zacken besser als die meisten anderen, die ich gesehen habe, aber leider verstehe ich trotzdem nach wie vor nicht, wie das funktionieren soll.

    Nochmals vielen Dank und sorry für den langen Text – mal sehen, ob den überhaupt irgendwer bemerkt, über fünf Jahre nach dem letzten Kommentar.
    Über jegliche Hinweise zu möglichen Antworten würde ich mich freuen!

  196. #196 MartinB
    18. Februar 2019

    @RJ
    “Wenn eine prominente Person in einer Menschenmenge gestoppt wird, oder eine Kugel in Honig, ändert sich ja dabe gerade der Bewegungszustand, sie wird nämlich zum Stillstand gebracht.”
    Richtig. Diese Beispiele sind schlecht, deswegen nutze ich die hier auf dem Blog auch normalerweise nicht, außer, wenn ich eine extrem vereinfachte Darstellung brauche.

    “(das Teilchen braucht länger, um von a nach b zu kommen)”
    Und das ist ja wichtig, weil es sich sonst mit Lichtgeschwindigkeit bewegen würde.

    Eine ganz andere Erklärung des Mechanismus findest du hier:
    https://scienceblogs.de/hier-wohnen-drachen/2016/10/01/das-higgs-teilchen-ganz-anders/
    Die hat den Vorteil, eigentlich genau die Physik korrekt wiederzugeben.

    “Also könnte man auch sagen, daß Fermionen ihre Energie aus dem Higgs-Feld beziehen?”
    Jein. Wenn wir nur über Ruhemasse reden, dann kann man das wohl so sagen, aber die Ruhemasse ist in der Quantentheorie auch kein ganz einfaches Konzept (letztlich tut sie genau das, was die Bildchen hier zeigen, aber es ist nicht so ganz einfach, das wieder anschaulich mit dem Alltagsbegriff zusammenzubringen).

    “Heißt das aber, daß niemand (zumindest niemand, der/die sich öffentlich an Erklärungen versucht) das Higgs-Feld bisher wirklich verstanden hat?”
    Nein, siehe auch den anderen Text. Diese Erklärungen funktionieren, und werden von Leuten verstanden.
    Siehe auch hier:
    https://scienceblogs.de/hier-wohnen-drachen/2012/07/14/kann-man-das-higgsteilchen-verstehen/

    “Wenn man das Modell grundsätzlich richtig verstanden hat, sollte man aber auch ohne die Mathematik korrekte qualitative Aussagen über das Modellverhalten machen können.”
    Richtig, ich denke das geht durchaus. Meist ist es aber so, dass die Leute, die eine gute Intuition haben, erst die Mathematik verstehen und dann ihre Intuition daraus ableiten. Und dass es wenige wirklich brauchbare Erklärungen gibt, liegt eben daran, dass es kompliziert ist, dass die meisten Medien (zeitungen usw.) Erklärungen, die mehr als einen Absatz lang sind, scheuen (und dann bleibt man halt beim Honig-Bild), und dass die Leute, die es wirklich gut verstehen, nicht unbedingt die sind, bei denen die Vermittlung an Laien Priorität hat.

    Und wie gesagt, der andere Text mit der Seilwellenanalogie sollte eigentlich auch erlauben, Dinge weiterzudenken.

  197. #197 MartinB
    18. Februar 2019

    PS,
    Sorry, bei den vielen Artikeln zum Higgs hatte ich nicht gesehen, dass du den, den ich unten verlinke, ja gerade kommentierst, ich dachte, wir sind hier bei einem anderen Artikel…

  198. #198 RJ
    18. Februar 2019

    Wow, das war schnell, danke!

    Den Artikel mit dem Seil hatte ich kurz vorher gelesen und jetzt nochmal ganz langsam.

    Jetzt verstehe ich das noch weniger. Dort geht es ja darum, daß sich die Welle ohne die Federn (das Higgs-Feld) mit immer der gleichen Geschwindigkeit (c) bewegt. Die Federn sorgen dann dafür, daß die Bewegung des Wellenpakets verlangsamt wird. Soweit richtig?

    Heißt das dann, daß die Teilchen sich tatsächlich nicht mehr mit Lichtgeschwindigkeit bewegen und im gleichen Zuge (durch Energiezufuhr) mit einer Masse versehen werden?

    Kann man lichtschnelle und masselose Teilchen überhaupt abbremsen? Oder werden die langsamer, weil sie eben nicht mehr masselos sind? Oder ist das auch nur wieder ein Modell und in Wirklichkeit bewegen die sich weiterhin mit Lichtgeschwindigkeit, nur nicht auf dem direkten Weg?

    Das Seilmodell erweckt ja den Eindruck, daß irgendwo punktuell (die Hand) eine Energie eingeleitet wird, die dann in der Interaktion mit dem Higgs-Feld (den Federn) dazu führt, daß die Geschwindigkeit des Teilchens (Wellenpaket) sich verlangsamt. Wenn wir das mal so hinnehmen (siehe meine Frage oben), woraus bestimmt sich denn dann die resultierende Geschwindigkeit? Wohl aus der Steifigkeit der Federn, oder? Aber ist die (die Stärke des Higgs-Feldes) nicht überall gleich? Wie kann ein Teilchen denn dann weiter verzögert oder beschleunigt werden? Und woher kommt der Widerstand gegenüber dieser Geschwindigkeitsänderung, vulgo die Masse des Teilchens? Oder ist die Annahme, daß die Masse sich ja nun automatisch ergibt, weil sie ja nicht mehr lichtschnell sind, also quasi eine emergente Eigenschaft sind?

    Sorry, ich stehe einfach nach wie vor auf dem Schlauch und diesen Teil verstehe ich auch nicht:

    “Jein. Wenn wir nur über Ruhemasse reden, dann kann man das wohl so sagen […]”

    Ich dachte, lichtschnelle Teilchen haben keine Ruhemasse, oder ist das gerade, was das Higgs-Feld macht: den Teilchen eine Ruhemasse geben und sie abbremsen?

    Da drehe ich mich aber wieder im Kreis … – kann man lichtschnelle Teilchen abbremsen? Ist es dieser Prozeß, der ihnen eine Ruhemasse gibt?

    Leistet das Higgs-Feld quasi eine Arbeit an ihnen, wodurch sie Geschwindigkeitsenergie in Masse umwandeln? (Kann ein masseloses Teilchen Geschwindigkeitsenergie haben? Null (Masse) mal unendlich (Lorentz-Faktor) = endlicher Wert?)
    Aber das wäre ja auch wieder eine Verzögerung eine Gegenwirkung zur Geschwindigkeit, also eine Dämpfung. Grrr, ich habe das Gefühl, die Einsicht ist nicht weit weg, aber ich kann sie nicht greifen.

  199. #199 MartinB
    18. Februar 2019

    @Rj
    “Heißt das dann, daß die Teilchen sich tatsächlich nicht mehr mit Lichtgeschwindigkeit bewegen und im gleichen Zuge (durch Energiezufuhr) mit einer Masse versehen werden?”
    Jupp.

    “Kann man lichtschnelle und masselose Teilchen überhaupt abbremsen?”
    Durch diesen Mechanismus ja, dann sind sie eben nicht mehr lichtschnell und masselos.

    “Oder ist das auch nur wieder ein Modell und in Wirklichkeit bewegen die sich weiterhin mit Lichtgeschwindigkeit, nur nicht auf dem direkten Weg?”
    Ist ein bisschen ne Frage, welches Bild man verwendet. Im (in meinen Augen besseren) Bild der Seilwelle bewegt sich die Anregung des Seils langsamer. Im Bild der Feynman-Diagramme lässt man in jedem einzelnen Diagramm die Teilchen zwischen den Punkten mit Lichtgeschwindigkeit laufen (naja, eigentlich auch nicht wirklich, man kann beliebige Geschwindigkeiten nehmen, aber alle anderen mitteln sich raus), aber was “ein Elektron” tut, ergibt sich eben erst aus der Summe über alle Diagramme und da kommt dann im Endeffekt wieder die langsamere Welle bei raus.

    “daß irgendwo punktuell (die Hand) eine Energie eingeleitet wird, die dann in der Interaktion mit dem Higgs-Feld (den Federn) dazu führt, daß die Geschwindigkeit des Teilchens (Wellenpaket) sich verlangsamt. ”
    DIe Hand ist die Erzeugung des Teilchens – also beispielsweise eines Elektrons.

    “Wie kann ein Teilchen denn dann weiter verzögert oder beschleunigt werden? ”
    Mit mehr Energie läuft sie schneller, mit weniger langsamer. DIe Geschwindigkeit wird durchdie Anregung vorgegeben.

    ” ist das gerade, was das Higgs-Feld macht: den Teilchen eine Ruhemasse geben und sie abbremsen?”
    Ja.

    “kann man lichtschnelle Teilchen abbremsen?”
    Ist glaube ich einfach die falsche Vorstellung, Es ist ja nicht s, als wäre da erst das lichtschnelle Teilchen und dann kommt das Higgsfeld und plötzlich wird es abgebremst. Das Higgsfeld ist überall und das Teilchen hat damit sofort die Masse, die es dank Higgsfeld hat.

    “Leistet das Higgs-Feld quasi eine Arbeit an ihnen”
    Nein, das Higgsfeld leistet keine Arbeit.

  200. #200 RJ
    18. Februar 2019

    Ah so, sie *wären* lichtschnell und masselos, wenn da nicht das Higgs-Feld wäre? Schöner Kunstgriff, oder?

    Ist schon immer wieder der Hammer, wenn nach so einer theoretischen Überlegung dann tatsächlich das dafür notwendige und passende Teilchen nachgewiesen werden kann.

    “Mit mehr Energie läuft sie schneller, mit weniger langsamer. DIe Geschwindigkeit wird durchdie Anregung vorgegeben.”

    Aber so verhält sich doch eine Seilwelle nicht, oder? Soweit ich das parat habe, kann die Anregung die Frequenz und Amplitude vorgeben, aber nicht die Geschwindigkeit – die wird durch die Parameter des, ahem, “Mediums” (Seil, Federn, hier: Higgs-Feld) bestimmt. Oder wo liege ich da falsch?

    “Nein, das Higgsfeld leistet keine Arbeit.”

    Hm, da muß ich nochmal drüber nachdenken. Leistet netto (unter dem Strich, in der Summe) keine Arbeit, oder in keiner Weise? Es übt ja Kräfte aus, über Wege, und das ist Arbeit, oder nicht?

    Für mich sieht es so aus, als leiste das Teilchen (die Welle) Arbeit am Higg-Feld und würde dadurch an Energie (Masse) zunehmen. Paßt das ungefähr?

  201. #201 MartinB
    18. Februar 2019

    @RJ
    “Ist schon immer wieder der Hammer, wenn nach so einer theoretischen Überlegung dann tatsächlich das dafür notwendige und passende Teilchen nachgewiesen werden kann.”
    In der Tat, deswegen wurde das Higgs ja auch 2012 so gefeiert.

    “Aber so verhält sich doch eine Seilwelle nicht, oder? ”
    Eine normale nicht, eine Seilwelle mit lauter Federn dran aber schon. Siehe das in der Fußnote des Artikels verlinkte Vorlesungsskript zur Dispersion (ist allerdings mathematisch). Das ist ja gerade der witz hier.

    Mit der Arbeit bin ich mir in der Analogie nicht sicher. Klar, in der Seilwelle werden die Federn lokal gespannt und es wird entsprechend nergie auf die Federn und dann wieder zurück übertragen. Bin mir momentan nicht sicher, ob das im Higgsfeld eine Entsprechung hat.

  202. #202 RJ
    18. Februar 2019

    Oh mei, da muß ich wohl noch ein bißchen lesen!
    Das Prinzip wird mir jetzt langsam klarer. Glaube ich. Vielen Dank für die Erklärungen!

    Was die Arbeit angeht, dachte ich, daß das gerade der Witz an der Sache ist. Die Welle zu bremsen erfordert Arbeit. Wenn ich das richtig deute, wird ein Teil der Anregungsenergie über diesen Mechanismus – also Arbeit am Higgs-Feld, das dann genausoviel Arbeit an der Welle leistet – von Bewegungsenergie in Masse konvertiert. Wie, verstehe ich noch nicht. Und vielleicht stimmt das ja auch gar nicht, erscheint mir nur gerade so.

  203. #203 MartinB
    18. Februar 2019

    “Die Welle zu bremsen erfordert Arbeit”
    Bin nicht sicher, ob das ein guter Ansatz ist. Die Welle ist ja nicht erst schnell und wird dann langsamer, sondern sie läuft gleich langsamer los und verbreitert sich.

  204. #204 RJ
    18. Februar 2019

    “Die Welle ist ja nicht erst schnell und wird dann langsamer, sondern sie läuft gleich langsamer los und verbreitert sich.”

    Ja, nur warum tut sie das?
    Wenn das Feld vom ersten Augenblick an bremst, bremst es doch trotzdem. Und wenn es das nicht tun würde, würde die Welle schon schneller loslaufen. Oder?

  205. #205 MartinB
    18. Februar 2019

    Wenn du ein solches Seil in die Han dnimmst und dann dran wackelst, so dass sich die Störung ausbreitet, was wird da “gebremst”? Dass die Geschwindigkeit der welle ohne die Federn größer wäre, “weiß” das Seil ja nicht.
    Vielleicht ist das ein Problem der Definition. Für mich bedeutet “bremsen”: die Geschewindigkeit verringern.
    Du verwendest die Alltagsdefinition, nach der mein Auto auch dann “gebremst” wird, wenn ich mit angezogener Handbremse losfahre.

    Dass die Ausbreitungsgeschwindigkeit mit den Federn kleiner ist, habe ich ja in dem anderen Artikel zu erklären versucht.

  206. #206 RJ
    18. Februar 2019

    Muß das Seil das denn wissen, daß es ohne die Federn schneller wäre? Hat das irgendeinen Einfluß auf die Tatsache, daß das der Fall ist?

    In dem Beispiel mit der Handbremse weiß der Motor auch nicht, daß die angezogen ist – man könnte auch bergauf fahren, starken Gegenwind haben, oder er ist noch nie ohne angezogene Handbremse gefahren – und trotzdem wäre das Fahrzeug bei gleicher Motorleistung schneller ohne. Beziehungsweise würde weniger Leistung reichen für die gleiche Geschwindigkeit.

    Oder wenn Du Dich auf einen Heimtrainer setzt – der hat auch eine Bremse, und ob Du das weißt oder nicht, spielt doch keine Rolle dabei, ob die wirksam ist.

    Eine Bremskraft ist eine Kraft, die der Bewegung entgegenwirkt – unabhängig davon, ob sie nun in der Summe aller Einflüsse zu einer Verzögerung führt oder nicht. Ich kann auch auf der Straße im Gefälle bremsen und trotzdem beschleunigen.

    Wenn sich die Geschwindigkeit verringert, nenne ich das Verzögerung, und die kann durch Bremsen eintreten, muß aber nicht. Bremsarbeit wird aber trotzdem geleistet. (Sorry, bin Ingenieur.)

    Ich sehe aber nach wie vor bei allen Beispielen nach wie vor das Problem, daß die Kraft/Leistung/Arbeit geschwindigkeitsabhängig ist (ich schließe mal konstant über der Geschwindigkeit hier ein – das verschwindet ja in der Ableitung zur Beschleunigung), und daher formeltechnisch als Dämpfung beschrieben werden müßte und nicht als Masse, die ja beschleunigungsabhängige Kräfte hervorruft.

    Oder habe ich jetzt komplett das Thema verfehlt?

  207. #207 MartinB
    18. Februar 2019

    @RJ
    Das ist ja, was ich sage, du verwendest den Begriff “bremsen” anders als ich. Für mich ist bremsen= Geschwindigkeit verringern (so ist es in der Physik definiert), für dich ist bremsen=eine der Geschwindigkeit entgegengesetzte Kraft.

    Formeltechnisch wird das wie gesagt detailliert in dem Skriptkapitel vorgerechnet, das ich verlinkt habe, wenn du es also vorgerechnet haben willst, findest du es da. Und ja, in vieler Hinsicht wirkt das wie eine Dämpfung.

  208. #208 RJ
    19. Februar 2019

    @MartinB
    Ok, unterschiedliche Terminologie, sorry.

    (Ich könnte argumentieren, daß es von Deinem Bezugssystem abhängt – wenn Du den beschriebenen Fall der Bremsbetätigung im Gefälle von einem sonst identischen Fahrzeug in der Parallelspur aus beobachtest, in dem die Bremse nicht betätigt wird, wirst Du das beobachtete Fahrzeug auch in Deinem Sinne bremsen, also seine Geschwindigkeit verringern, sehen. Insofern verwundert es mich, daß die Physik da so eine schwammige und nicht allgemein anwendbare Begriffsdefinition verwendet. Aber ich versuche mal, das Wort “bremsen” nicht mehr zu verwenden.)

    Also anders formuliert:
    Im Seilmodell mit Federn sind bei gleicher Anregungsenergie die Ausbreitungsgeschwindigkeit und [Amplitude oder Frequenz] geringer als im Modell ohne Federn. Das legt aus meiner Sicht nahe, daß ein Teil der Anregungsenergie von den Federn aufgenommen wird. Auch wieder abgegeben, aber mein Punkt ist, daß sie überhaupt in das Feld reingeht, was ja Arbeit am Feld erfordert.

    Beim Higgsfeld ist ja gerade der Witz, daß es ebenfalls die Ausbreitungsgeschwindigkeit verringert.

    Ok, bin ich jetzt wieder unpräzise? Es verringert sie ja nicht, denn es gibt ja keine Ausbreitungsgeschwindigkeit vor Anwendung des Higgsfeldes – richtiger wäre also zu sagen, daß die Ausbreitungsgeschwindigkeit mit Higgsfeld geringer ist, als sie es im theoretischen Fall seines Nicht-Vorhandenseins wäre. So oder so ist zu fragen, wie das gehen soll, wenn kein Energieaustausch stattfindet, sprich Arbeit geleistet wird.

    Den Artikel werde ich noch lesen, habe ich aber noch nicht. Ich hoffe, die Mathematik ist verdaulich – an sich möchte ich es nicht wirklcih “vorgerechnet” haben, sondern anschaulich erklärt, denn bei der Mathematik steige ich, wie gesagt, ziemlich schnell aus.

    Das Problem mit Dämpfung ist halt, daß sie eine Gegenkraft zu jeglicher Bewegung erzeugt, sprich dann stellt sich wirklich die Frage, warum die Teilchen nicht kontinuierlich immer langsamer werden und irgendwann komplett stehenbleiben. (Außer natürlich bei negativer Dämpfung, die sie immer weiter beschleunigen würde, so wie die dunkle Energie es ja tut – da gibt es aber keinen Zusammenhang, oder?)

    Masse erzeugt bei (positiver oder negativer) Beschleunigung eine Kraft, die der Beschleunigung entgegengesetzt ist, sprich jeder unbeschleunigte Zustand wird beibehalten, solange keine externen Kräfte wirken.
    Dämpfung hingegen erzeugt eine Kraft, die jeglicher Bewegung entgegenwirkt, sprich nur die Massenträgheit sorgt dafür, daß die so gedämpfte Masse nicht sofort stehenbleibt.

    Da paßt einfach die Erklärung noch nicht, denn Dämpfung ist ja nicht, was wir in der Realität beobachten, sonst würde ohne ständige Energiezufuhr alles stehenbleiben. Oder wird die Dämpfungsenergie dem System ständig in anderer Form wieder zugeführt, um eben den Vortrieb zu erzeugen, der die Dämpfung überwindet?

    Ok ok, ich werde den anderen Artikel lesen, habe nur nicht allzuviel Hoffnung, daß mir die Mathematik darin meine Fragen beantworten wird.

  209. #209 MartinB
    19. Februar 2019

    @RJ
    “Ich könnte argumentieren, daß es von Deinem Bezugssystem abhängt”
    Nein, solange wir über gleichförmig bewegte Bezugssysteme reden, ist immer eindeutig, ob ein Objekt konstante Geschwindigkeit hat oder nicht. Das Auto am Gefälle beschleunigt ja, beschleunigte Bezugssysteme sind immr schwierig (dazu habe ich in meinem Buch ganz schön viel geschrieben…)

    ” Das legt aus meiner Sicht nahe, daß ein Teil der Anregungsenergie von den Federn aufgenommen wird”
    Ja, das hatte ich ja auch oben geschrieben. Ich bin mir nur nicht sicher, ob die mathematische Analogie soweit geht, dass es auch im Higgsfeld eine Energie ist, die da ausgetauscht wird; ich glaube eigentlich eher nicht, dass man es so sehen kann.

    Und nein, in dem Sinne, dass alles immer weiter gebremst wird und schließlich stehen bleibt, gibt es weder beim Higss noch bei der Seilwelle eine Dämpfung.

  210. #210 RJ
    20. Februar 2019

    @MartinB
    Tja, da muß ich wohl noch weiterlesen, vielleicht klickt es ja irgendwann. Vielen Dank nochmal für die Richtigstellungen und Denkanstöße!